Tap chi Epsilon so 3

237 20 0
Tap chi Epsilon so 3

Đang tải... (xem toàn văn)

Tài liệu hạn chế xem trước, để xem đầy đủ mời bạn chọn Tải xuống

Thông tin tài liệu

Với gần 50 chủ đề chia về 5 nhóm kiến thức số học, đại số, hình học, giải tích, tổ hợp, và lý thuyết đồ thị, THE BOOK chứa đựng một cách dày đặc các định lý, tính chất, và các chứng minh[r]

(1)Vĩnh thức Tự học là tốt có thầy tốt Ngô Quang Hưng Ma trận ngẫu nhiên Vũ Hà Văn Thư Kapitsa khoa học Đàm Thanh Sơn (dịch) Nguyễn Tiến Dũng VÀ CÁC CHUYÊN MỤC KHÁC Định lí bướm kép tứ giác Nguyễn Ngọc Giang Trịnh Huy Vũ Cubic Rubik Trần Nam Dũng (dịch và tổng hợp) 90 x x x 90 90 45 x HENRYK MINC ghi hồi ông nộp số bài báo vĩnh thức kỷ 20 thì có bình duyệt viên nói “chế cái tên gì lố bịch thế”? 1/3x No n g - Qua Vĩnh t c - N gô 135 Nếu bạn muốn bạn thông minh, hãy đọc cho chúng nghe truyện cổ tích Nếu bạn muốn bạn thông minh hơn, hãy đọc cho chúng nhiều truyện cổ tích Hưng Albert Einstein 13 Jun 2015 (2) ngày 13 tháng 06 năm 2015 Số Tạp chí online cộng đồng người yêu Toán Chủ biên: TRẦN NAM DŨNG Biên tập viên: VÕ QUỐC BÁ CẨN TRẦN QUANG HÙNG LÊ PHÚC LỮ NGUYỄN TẤT THU ĐẶNG NGUYỄN ĐỨC TIẾN (3) LỜI NGỎ Ban biên tập Epsilon Epsilon số mắt bạn đọc đúng hạn vào ngày 13/4 đã làm cho Epsilon không thuộc vào đội ngũ các Tạp chí số Tức là tạp chí dồn công sức số đầu hay, tốt là Có số tức là hy vọng có số Và bạn đọc đọc số tạp chí Epsilon – tạp chí online người yêu toán Qua số đầu tiên, chúng ta vui mừng vì người đọc Epsilon nhiều lên, người biết đến và ủng hộ Epsilon nhiều lên, người viết bài cho Epsilon nhiều lên Trong buổi uống bia trưa 10/6 quán Hải Xồm gần Viện Toán học, chúng tôi cảm thấy vui vui người nhắc đến Epsilon “Hôm trước gặp hội anh Nguyễn Thành Nam, có nói chú vừa tờ Epsilon Trước đó Ngô Bảo Châu có nói anh rảnh thì viết bài cho Epsilon” – GS Phạm Hữu Tiệp, người vừa trình bày chuyên đề Viện toán nói với tôi “Anh rảnh viết cho em luôn bài Random walk anh”, “Chưa biết anh có viết bài đó không, sau trận bia này anh với chú ramdon walk viện Toán” Thấy người bàn tán rôm rả Epsilon, PGS trẻ Phạm Hoàng Hiệp, người vừa giải thưởng Tạ Quang Bửu, còn nói đùa “Có anh Dũng phải đổi tên tạp chí thành 1/epsilon nên” Nói vui là thế, công việc Epsilon thực là công việc gom li ti để tạo nên sản phẩm nhỏ mà ý nghĩa, để người đọc luôn có thể tìm điều gì đó cho mình qua số tạp chí Với suy nghĩ vậy, chúng tôi mong muốn, trân trọng và ghi nhận đóng góp đến từ các tác giả Epsilon hy vọng có mặt mình làm cho người chịu khó viết bài Các GS thì cố gắng nghiên cứu cách viết đơn giản, dễ hiểu để có thể đến với số đông Các bạn học sinh, sinh viên cố gắng viết bài có chất lượng hơn, có định hướng hơn, bước đầu làm quen (4) Tạp chí Epsilon, Số 03, 06/2015 với phong cách nghiên cứu và trình bày bài báo khoa học Các thầy giáo có động lực việc tổng kết các chuyên đề cách hệ thống hơn, thay vì các bài viết nhỏ với các ý tưởng đơn lẻ Epsilon số lần này quy tụ 13 bài viết chính với các chủ đề phong phú: Các bài viết Vũ Hà Văn “Ma trận ngẫu nhiên” và Ngô Quang Hưng “Vĩnh thức và định thức Pfaff” giới thiệu với bạn đọc vấn đề nóng hổi toán học đại Mục lịch sử toán học dành các trang viết mình cho các nhà Vật lý qua bài “Những triết lý sống Einstein” BBT sưu tầm và “Thư Kapitsa khoa học” GS Đàm Thanh Sơn dịch và giới thiệu Mục Giảng dạy toán học lần này có bài GS Nguyễn Tiến Dũng “Tự học là tốt có thầy tốt hơn” Nguyễn Quốc Khánh tiếp tục chuyên mục điểm sách đầy hấp dẫn mình qua bài “Đêm trước thảo in” Đặng Nguyễn Đức Tiến thì tạm gác chủ đề mũ để giới thiệu nhà truyền bá toán học tiếng giới – Martin Gardner Chuyên mục Các vấn đề cổ điển và đại giới thiệu chuỗi bài toán đếm tam giác (dành cho học sinh tiểu học và THCS) và chuỗi bài toán khối vuông Rubik (từ trò chơi đến lý thuyết nhóm) Mảng toán sơ cấp, thường lệ rôm rả với bài viết công phu “Cực trị tập hợp” thầy Trần Minh Hiền, tiểu phẩm xinh xinh thầy Nguyễn Duy Liên xung quanh bài toán thi IMO 2001 Các học sinh chuyên toán chuẩn bị cho các kỳ thi HSG năm học sau chắn tìm nhiều điều bổ ích qua bài bình luận bài còn lại kỳ thi chọn đội tuyển Việt Nam 2015 thầy Trần Nam Dũng Đặc biệt mảng hình học lần này giới thiệu chùm hoa đẹp gồm ba bài viết các thành viên nhóm Bài toán hay – Lời giải đẹp – Đam mê toán học (Vũ Thanh Tùng, Nguyễn Chương Chí, Nguyễn Ngọc Giang) phối hợp cùng biên tập viên Trần Quang Hùng và các học trò (Nguyễn Bảo Ngọc, Trịnh Huy Vũ) Những bài toán và định lý xinh xắn, chứng minh ảo thuật chắc làm hài lòng độc giả yêu thích hình học, còn người chưa thích bắt đầu thích Làm cho người đã thích toán thêm thích toán Làm cho người chưa thích toán bắt đầu thấy thích toán Nếu hoàn (5) Tạp chí Epsilon, Số 03, 06/2015 thành 10% nhiệm vụ đó thì người làm Epsilon quá đỗi vui mừng Và lại có lượng để bước tiếp Đi nhiều người, bạn xa (6) Tạp chí Epsilon, Số 03, 06/2015 (7) MỤC LỤC Lời ngỏ Ma trận ngẫu nhiên Vũ Hà Văn Vĩnh thức 29 Ngô Quang Hưng Tự học là tốt có thầy tốt 61 Nguyễn Tiến Dũng Thư Kapitsa khoa học 71 Đàm Thanh Sơn Những triết lý sống Einstein 81 Ban biên tập Epsilon Lời giải và bình luận bài toán đề thi chọn đội tuyển Việt Nam 2015 89 Trần Nam Dũng Martin Gardner - Người làm vườn toán học 105 Đặng Nguyễn Đức Tiến Cực trị tập hợp 123 Trần Minh Hiền (8) Tạp chí Epsilon, Số 03, 06/2015 10 Một bài toán số học hay với nhiều cách giải 175 Nguyễn Duy Liên 11 Định lý carnot đồng quy các đường vuông góc với các cạnh tam giác và ứng dụng 181 Vũ Thanh Tùng, Nguyễn Chương Chí 12 Về bài toán hình học từ diễn đàn AoPS 193 Trần Quang Hùng, Nguyễn Bảo Ngọc 13 Định lý bướm kép tứ giác 205 Nguyễn Ngọc Giang (TP Hồ Chí Minh) Trịnh Huy Vũ (THPT Chuyên KHTN Hà Nội) 14 Đêm trước thảo in 211 Nguyễn Quốc Khánh 15 Các vấn đề cổ điển và đại 223 Trần Nam Dũng (9) MA TRẬN NGẪU NHIÊN VŨ HÀ VĂN (Đại học Yale, Mỹ) Lời giới thiệu Lý thuyết ma trận ngẫu nhiên có mục tiêu chính là đưa hiểu biết sâu sắc các tính chất đa dạng các ma trận mà các thành phần chúng chọn ngẫu nhiên từ các phân phối xác suất khác Từ đời đến nay, lý thuyết ma trận ngẫu nhiên đã có phát triển mạnh mẽ, thúc đẩy ứng dụng Thống kê và Giải tích số, Khoa học máy tính, Điều khiển tối ưu, và đặc biệt là các ứng dụng Vật lý hạt nhân Ở Việt Nam, lý thuyết ma trận ngẫu nhiên là khái niệm tương đối Năm 2009, người viết lời giới thiệu này, dạy cho đội tuyển Olymic Toán Việt Nam chuẩn bị cho kỳ thi toán quốc tế IMO 2009, đã dẫn toàn đội tuyển đến dự bài nói chuyện GS Vũ Hà Văn Ma trận ngẫu nhiên Thú thực là mặc dù thầy và trò hiểu lõm bõm điều GS Văn nói, tất ấn tượng hàng loạt các giả thuyết đã Vũ Hà Văn cùng Terence Tao chứng minh với tốc độ chóng mặt Trong Epsilon số này, đồng ý tác giả, chúng tôi trích giới thiệu nội dung chương đầu bài báo cáo GS Vũ Hà Văn Đại hội Toán học Thế giới 2014 (ICM 2014) Để giúp độc giả có thể nắm bắt nội dung chính, chúng tôi cố gắng chú giải chi tiết có thể, đồng thời đăng nguyên tiếng Anh để đối chiếu Vì đây là lĩnh vực mới, ít có tài liệu tiếng Việt nên dịch thuật có thể có chỗ chưa chuẩn, mong nhận ý kiến đóng góp bạn đọc để các phần sau dịch tốt Ban Biên tập (10) Tạp chí Epsilon, Số 03, 06/2015 Tóm tắt nội dung Trong bài viết này, chúng ta trao đổi số bài toán lý thuyết ma trận ngẫu nhiên có chất tổ hợp Mở đầu Lý thuyết ma trận ngẫu nhiên là mảnh đất màu mỡ toán học Bên cạnh vấn đề nội thú vị, ma trận ngẫu nhiên đóng vai trò quan trọng nhiều lĩnh vực Thống kê, Vật lý Toán, Tổ hợp, Khoa học Máy tính Trong khảo sát này, chúng tôi tập trung vào các bài toán có chất tổ hợp Các bài toán này đặc biệt thú vị các ma trận lấy mẫu từ phân phối rời rạc Các mô hình thông dụng là: • (Bernoulli) Mn : ma trận ngẫu nhiên bậc n mà các thành phần là các biến ngẫu nhiên độc lập đồng theo phân phối Bernoulli (nhận các giá trị ±1 với xác suất 1/2) Ma trận này đôi còn gọi là ma trận dấu ngẫu nhiên1 Tổng cộng có tất N = 2n ma trận với tất các thành phần là ±1, ma trận có xác suất 1/N • (Bernoulli đối xứng2 ) Mnsym : ma trận đối xứng ngẫu nhiên bậc n mà các thành phần trên và đường chéo là các biến ngẫu nhiên độc lập đồng theo phân phối Bernoulli Số ma trận đối xứng với thành phần ±1 là M = 2n(n+1)/2 và ma trận có xác suất 1/M • Ma trận kề đồ thị ngẫu nhiên Với đồ thị ta có ma trận kề định nghĩa sau: Giả sử đồ thị G có n đỉnh {1, , n} Ma trận kề G là ma trận đối xứng và vị trí ij ta viết ij là cạnh G và trường hợp ngược lại Về các mô hình đồ thị ngẫu nhiên Trong bài viết này, chúng tôi xét trên hai mô hình: Erdös-Rényi và đồ thị ngẫu nhiên Chi tiết các mô hình này, xem [6, 35] Random sign matrix Toàn chú thích bài này Ban Biên tập Symmetric Bernoulli 10 (11) Tạp chí Epsilon, Số 03, 06/2015 • (Erdös-Rényi) Ta ký hiệu G(n, p) là đồ thị ngẫu nhiên trên n đỉnh, sinh cách vẽ cạnh nối hai điểm với xác suất p cách độc lập • (Đồ thị ngẫu nhiên3 ) Đồ thị ngẫu nhiên có n đỉnh với bậc d thu cách chọn ngẫu nhiên với xác suất trên tập hợp tất các đơn đồ thị bậc d4 trên tập các đỉnh {1, 2, , n} Ta ký hiệu đồ thị này là Gn,d Có chú ý quan trọng là các cạnh Gn,d không độc lập5 Vì vậy, mô hình này thường khó nghiên cứu mô hình G(n, p) Ta ký hiệu A(n, p) là ma trận kề đồ thị ngẫu nhiên ErdösRényi G(n, p), và An,d là ma trận kề Gn,d tương ứng Về ký hiệu: Trong suốt bài này, n luôn giả sử là lớn Các ký hiệu tiệm cận6 o, O, Θ hiểu n tiến tới vô cùng Ta viết A  B A = o(B) c ký hiệu cho số chung7 Tất các logarit là logarit tự nhiên không nói khác Xác suất suy biến Bài toán tổ hợp tiếng ma trận ngẫu nhiên có lẽ là bài toán suy biến8 Gọi pn là xác suất ma trận Mn suy biến (một ma trận vuông suy biến định thức 0) Hiển nhiên là: pn ≥ 2−n vì vế phải là xác suất để hai dòng đầu ma trận nhau9 Vì ta có thể chọn hai dòng (cột) (thay vì chọn dòng đầu) và có thể thay dấu dấu cùng nhân với Random regular graph Đồ thị bậc d (d-regular graph) là đồ thị mà đỉnh có bậc d Ví dụ đồ thị bậc có đỉnh là đỉnh cô lập, đồ thị đầy đủ Kn là đồ thị bậc n − Nghĩa là xác suất tồn các cạnh Gn,d không độc lập với nhau, xác suất các cạnh G(n, p) là độc lập Asymptotic notation Universal constant Đôi dịch là số phổ dụng hay số độc lập Singularity problem Ma trận có dòng có định thức 11 (12) Tạp chí Epsilon, Số 03, 06/2015 ±110 , ta có cận tốt chút:   n −n pn ≥ (4 − o(1)) = ( + o(1))n 2 (2.1) Một giả thuyết đặt là: Giả thuyết 2.1 [Giả thuyết suy biến] pn = ( 12 + o(1))n Giả thuyết 2.1 là bài toán mở, ta có thể phát biểu giả thuyết chính xác (xem [4]), dựa vào "niềm tin" sau: Hiện tượng I.11 Lý chủ yếu để ma trận ngẫu nhiên suy biến là phụ thuộc số ít hàng/cột Thực thì việc chứng minh pn = o(1) đã là không đơn giản Kết này lần đầu tiên chứng minh Komlós [38] vào năm 1967 (trong phần bài này, chúng tôi đưa chứng minh ngắn gọn cho định lý Komlós) Sau đó Komlós (xem [6]) tìm chứng minh cho cận trên: pn = O(n−1/2 ) Trong bài báo quan trọng, Kahn, Komlós và Szemerédi [37] lần đầu tiên đã đưa cận trên theo hàm mũ: Định lý 2.2 pn ≤ 999n Lập luận Kahn, Komlós và Szemerédi đã đơn giản hóa Tao và Vũ bài báo [66] năm 2004, dẫn đến cận trên tốt chút O(.958n ) Sau đó ít lâu, các tác giả này [67] kết hợp cách tiếp cận [37] với ý tưởng các định lý đảo (xem [71, chương 7] hay [53]) đã đạt kết quan trọng: Định lý 2.3 pn ≤ (3/4 + o(1))n Không dừng lại đó, Bourgain, Vũ và Wood bài báo [9] đã dùng thêm ý tưởng không gian có chiều phân số để tiếp tục cải thiện cận trên: Định lý 2.4 pn ≤ ( √12 + o(1))n 10 Nghĩa là ta có thể chọn dòng (hoặc cột) có cùng trị tuyệt đối thay vì 11 Ở đây tác giả dùng "Phenomenon" nên chúng tôi đối dịch là "Hiện tượng" Đây là cách dùng lạ, vì nó mang tính trực giác nhiều cho vấn đề xét 12 (13) Tạp chí Epsilon, Số 03, 06/2015 Hai phương pháp trên [67, 9] cho phép chúng ta thu các cận trên pn trực tiếp từ các ước tính lượng giác đơn giản Ví dụ cận trên 3/4 có từ: | cos x| ≤ + cos 2x, 4 | cos x|2 = 1 + cos 2x 2 √ cận 1/ thu từ Định lý 2.2 [9] đã đưa mối liên hệ hình thức các ước lượng suy biến và các ước lượng lượng giác Các liên hệ này mặc dù chưa thể dùng để giải bài toán suy biến tổng quát, có thể sử dụng để ước tính khá chính xác các cận trên số trường hợp, chẳng hạn xác suất suy biến ma trận ngẫu nhiên với các thành phần (0, ±1) Để kết thúc mục này, chúng tôi đề cập đến công cụ hữu ích: định lý Littlewood-Offord-Erdös Gọi v = {v1 , , } là tập hợp gồm n số thực khác và ξ1 , , ξn là các biến ngẫu Pnhiên Bernoulli độc lập phân bố đồng Định nghĩa S := ni=1 ξi vi và pv (a) = P(S = a) và pv = supa∈Z pv (a) Vào năm 1940, Littlewood và Offord đã đưa cách ước tính pv (ở [45]) thành tố kỹ thuật chính các nghiên cứu họ nghiệm thực đa thức ngẫu nhiên Erdös, cách cải thiện kết Littlewood và Offord, đã chứng minh định lý sau mà chúng ta gọi là bất đẳng thức bóng nhỏ Erdös-Littlewood-Offord (xem [53] để rõ cái tên này) Định lý 2.5 (Bất đẳng thức bóng nhỏ) Giả sử v1 , , là các số thực khác và ξ1 , , ξn là các biến ngẫu nhiên Bernoulli độc lập phân bố đồng Khi đó:  n pv ≤ bn/2c 2n = O(n−1/2 ) Định lý 2.5 là kết kinh điển tổ hợp và có nhiều mở rộng và hệ sâu xa (xem [7, 34, 53], [71, Chương 7] và các tài liệu tham khảo đó) 13 (14) Tạp chí Epsilon, Số 03, 06/2015 Để độc giả có thể cảm nhận làm bất đẳng thức bóng nhỏ có thể có ích việc đánh giá pn , ta xếp các dòng Mn dòng từ trên xuống Giả sử n − dòng đầu là độc lập và tạo thành siêu phẳng với véc-tơ pháp tuyến v = (v1 , , ) Khi đó, xác suất để Mn suy biến là: P(X · v = 0) = P(ξ1 v1 + · · · + ξn = 0), đó X = (ξ1 , , ξn ) là dòng cuối cùng Trong phần 3, độc giả thấy ứng dụng định lý 2.5 dẫn đến kết gốc Komlós: pn = o(1) Để thu các đánh giá mạnh các kết định lý 2.3 và 2.4, ta cần thiết lập các định lý Littlewood-Offord đảo, dựa trên nguyên lý tổng quát sau: Hiện tượng II Nếu P(X · v = 0) là tương đối lớn thì các hệ số v1 , , có cấu trúc cộng tính mạnh Các định lý này thúc đẩy các định lý đảo kiểu Freiman tổ hợp cộng tính12 mà việc thảo luận nằm ngoài phạm vi khảo sát này Độc giả quan tâm có thể xem chi tiết [53] Một chứng minh đơn giản định lý Komlós pn = o(1) Ta hãy tính chất đơn giản Từ đây sau véc-tơ Bernoulli hiểu là véc-tơ với tọa độ ±1 Tính chất 3.1 Cho H là không gian ≤ d ≤ n chiều Khi đó H chứa nhiều 2d véc-tơ Bernoulli Để thấy điều này, ta chú ý không gian d chiều, tồn tập hợp d tọa độ xác định các tọa độ còn lại13 Tính chất này suy ra: pn ≤ n−1 X i=1 P(Xi+1 ∈ Hi ) ≤ 12 n−1 X i=1 2i−n ≤ − 2n Additive Combinatorics Trong không gian d chiều, có thể dùng d véc-tơ sở để biểu diễn véc-tơ không gian đó 13 14 (15) Tạp chí Epsilon, Số 03, 06/2015 Rất không hay là điều này đối nghịch với kết chúng ta muốn chứng minh, đừng vội nản chí, màn hay hãy còn phần sau! Để thu cận trên o(1) mong muốn, ta cần chứng minh tổng số hạng tử cuối, chẳng hạn log log n, không Để chứng minh điều này, ta sử vượt quá (chẳng hạn) log1/3 n dụng tính chất Hi sinh các véc-tơ ngẫu nhiên Bổ để sau suy định lý Komlós thông qua định lý cận hợp14 : Bổ đề 3.1 Cho H là không gian sinh d véc-tơ ngẫu nhiên, đó d ≥ n − log log n Khi đó với xác suất ít − n1 , n H chứa nhiều log21/3 n véc-tơ Bernoulli Ta nói tập hợp S gồm d véc-tơ là k-phổ dụng với tập k số khác ≤ i1 , , ik ≤ n và tập dấu 1 , , n (i = ±1) nào, tồn véc-tơ v thuộc S cho dấu tọa độ thứ ij v j , với ≤ j ≤ k Tính chất 3.2 Nếu d ≥ n/2 thì − n1 là xác suất thấp cho tập hợp gồm d véc-tơ ngẫu nhiên là k-phổ dụng với k := log n/10 Để chứng minh điều này, ta chú ý xác suất thất bại, theo định lý cận hợp, không vượt quá   n 1 (1 − k )d ≤ nk (1 − k )n/2 ≤ n−1 k 2 Nếu S là k-phổ dụng, thì véc-tơ v khác phần bù trực giao không gian sinh S có nhiều k véctơ khác (nếu không, có véc-tơ S có tích trong15 14 Union bound Còn gọi bất đẳng thức Boole, phát biểu với tập hữu hạn đếm thì xác suất để ít kiện xảy nhỏ tổng xác suất tất các kiện, nghĩa là E1 , E2 , En , là các kiện thì: P {∃i : Ei xảy ra} = P { ∞ [ i=1 Ei } ≤ ∞ X i=1 P {Ei } Ở số tài liệu union bound còn gọi là định lý tổng xác suất 15 Inner product 15 (16) Tạp chí Epsilon, Số 03, 06/2015 dương với v) Nếu ta cố định véc-tơ v này và giả sử X là véc-tơ ngẫu nhiên Bernoulli thì theo định lý 2.5 P(X ∈ Span(S)) ≤ P(X · v = 0) = O( k 1/2 )≤ log 1/3 n , và bổ đề 3.1 cùng định lý chứng minh *** Phần bài viết tiếp tục giới thiệu các số Epsilon Chúng tôi đính kèm gốc tiếng Anh để bạn đọc tiện theo dõi phần sau 16 (17) (Van H Vu)∗ Keywords General mathematics, collection of articles Abstract In this survey, we discuss several problems in Random Matrix theory of combinatorial nature Introduction The theory of random matrices is a very rich topic in mathematics Beside being interesting in its own right, random matrices play fundamental role in various areas such as statistics, mathematical physics, combinatorics, theoretical computer science, etc In this survey, we focus on problems of combinatorial nature These problems are most interesting when the matrix is sampled from a discrete distribution The most popular models are: • (Bernoulli) Mn : random matrix of size n whose entries are i.i.d Bernoulli random variables (taking values ±1 with probability 1/2) This is sometimes referred to as the random sign matrix • (Symmetric Bernoulli) Mnsym : random symmetric matrix of size n whose (upper triangular) entries are i.i.d Bernoulli random variables • Adjacency matrix of a random graph This matrix is symmetric and at position ij we write if ij is an edge and zero otherwise • Laplacian of a random graph Model of random graphs We consider two models: Erdös-Rényi and random regular graphs For more information about these models, see [6, 35] • (Erdös-Rényi) We denote by G(n, p) a random graph on n vertices, generated by drawing an edge between any two vertices with probability p, independently ∗ Yale University (18) • (Random regular graph) A random regular graph on n vertices with degree d is obtained by sampling uniformly over the set of all simple d-regular graphs on the vertex set {1, , n} We denote this graph by Gn,d It is important to notice that the edges of Gn,d are not independent Because of this, this model is usually harder to study, compared to G(n, p) We denote by A(n, p) (L(n, p)) the adjacency (laplacian) matrix of the Erdös-Rényi random graph G(n, p) and by An,d (Ln,d ) the adjacency (laplacian) matrix of Gn,d , respectively Notation In the whole paper, we assume that n is large The asymptotic notation such as o, O, Θ is used under the assumption that n → ∞ We write A  B if A = o(B) c denotes a universal constant All logarithms have natural base, if not specified otherwise The singular probability The most famous combinatorial problem concerning random matrices is perhaps the ”singularity” problem Let pn be the probability that Mn is singular Trivially, pn ≥ 2−n , as the RHS is the probability that the first two rows are equal By choosing any two rows (columns) and also replacing equal by equal up to sign, one can have a slightly better lower bound   n −n (1) pn ≥ (4 − o(1)) = ( + o(1))n 2 It has been conjectured, for quite sometime, that Conjecture 2.1 [Singularity Conjecture] pn = ( 12 + o(1))n Conjecture 2.1 is still open, but one can formulate even more precise conjectures (see [4]), based on the following belief Phenomenon I The dominating reason for singularity is the dependency between a few rows/columns It is already non-trivial to prove that pn = o(1) This was first done by Komlós [38] in 1967 and in Section 3, we will give a short proof of this fact Later, Komlós (19) (see [6]) found a new proof which gave quantitative bound pn = O(n−1/2 ) In an important paper, Kahn, Komlós and Szemrédi [37] proved the first exponential bound Theorem 2.2 p(n) ≤ 999n Their arguments were simplified by Tao and Vu in 2004 [66], resulting in a slightly better bound O(.958n ) Shortly afterwards, these authors [67] combined the approach from [37] with the idea of inverse theorems (see [71, Chapter 7] or [53] for surveys) to obtained a more significant improvement Theorem 2.3 p(n) ≤ (3/4 + o(1))n With an additional twist, Bourgain, Vu and Wood [9] improved the bound further Theorem 2.4 p(n) ≤ ( √12 + o(1))n The method from [67, 9] enables one to deduce bounds on pn directly from simple trigonometrical estimates For instance, the 3/4-bound comes from the fact that | cos x| ≤ + cos 2x, 4 | cos x|2 = 1 + cos 2x 2 √ while the 1/ bound come from [9, Theorem 2.2] provides a formal connection between singularity estimates and trigonometrical estimates of this type, which, while not yet solve the Singularity Conjecture, does lead to sharp bounds in other situations, such as singularity of random matrices with (0, ±1) entries) To conclude this section, let us mention a very useful tool, the Littlewood-OffordErdös theorem Let v = {v1 , , } be a set of n non-zero Pn real numbers and ξ1 , , ξn be i.i.d random Bernoulli variables Define S := i=1 ξi vi and pv (a) = P(S = a) and pv = supa∈Z pv (a) The problem of estimating pv came from a paper of Littlewood and Offord in the 1940s [45], as a key technical ingredient in their study of real roots of random polynomials Erdös, improving a result of Littlewood and Offord, proved the following theorem, which we will refer to as the Erdös-Littlewood-Offord small ball inequality; see [53] for an explanation of this name Theorem 2.5 (Small ball inequality) Let v1 , , be non-zero numbers and ξi be i.i.d Bernoulli random variables Then  n pv ≤ bn/2c 2n = O(n−1/2 ) (20) Theorem 2.5 is a classical result in combinatorics and have many non-trivial extensions with far reaching consequences (see [7, 34, 53], [71, Chapter 7] and the references therein) To give the reader a feeling about how small ball estimates can be useful in estimating pn , let us expose the rows of Mn one by one from top to bottom Assume that the first n−1 rows are independent and form a hyperplane with normal vector v = (v1 , , ) Conditioned on these rows, the probability that Mn is singular is P(X · v = 0) = P(ξ1 v1 + · · · + ξn = 0), where X = (ξ1 , , ξn ) is the last row In Section 3, the reader will see an application of Theorem 2.5 that leads to Komlós’ original result pn = o(1) In order to obtain the stronger estimates in Theorems 2.3 and 2.4, one needs to ebstablish Inverse (or structural) Littlewood-Offord theorems, based on the following general principle Phenomenon II If P(X ·v = 0) is relatively large, then the coefficients v1 , , posses a strong additive structure These theorems are motivated by inverse theorems of Freiman type in Additive Combinatorics, the discussion of which is beyond the scope of this survey The interested reader is referred to [53] for a detailed discussion A simple proof of Komlós’ Theorem Let us start with a simple fact Here and later Bernoulli vectors mean vectors with coordinates ±1 Fact 3.1 Let H be a subspace of dimension ≤ d ≤ n Then H contains at most 2d Bernoulli vectors To see this, notice that in a subspace of dimension d, there is a set of d coordinates which determine the others This fact implies pn ≤ n−1 X i=1 P(Xi+1 ∈ Hi ) ≤ n−1 X i=1 2i−n ≤ − 2n While this bound is quite the opposite of what we want to proof, notice that the loss comes at the end Thus, to obtain the desired upper bound o(1), it suffices to (21) show that the sum of the last (say) log log n terms contribute at most (say) log1/3 n To this, we will exploit the fact that the Hi are spanned by random vectors The following lemma implies the theorem via the union bound Lemma 3.1 Let H be the subspace spanned by d random vectors, where d ≥ n n − log log n Then with probability at least − n1 , H contains at most log21/3 n Bernoulli vectors We say that a set S of d vectors is k-universal if for any set of k different indices ≤ i1 , , ik ≤ n and any set of signs 1 , , n (i = ±1), there is a vector v in S such that the sign of the ij th coordinate of v matches j , for all ≤ j ≤ k Fact 3.2 If d ≥ n/2, then with probability at least 1− n1 , a set of d random vectors is k-universal, for k := log n/10 To prove this, notice that the failure probability is, by the union bound, at most   n (1 − k )d ≤ nk (1 − k )n/2 ≤ n−1 2 k It S is k-universal, then any non-zero vector v in the orthogonal complement of the subspace spanned by S should have more than k non-zero vectors (otherwise, there would be a vector in S having positive inner product with v) If we fix such v and let X be a random Bernoulli vector, then by Theorem 2.5, P(X ∈ Span(S)) ≤ P(X · v = 0) = O( proving Lemma 3.1 and the theorem k 1/2 )≤ log 1/3 n , (22) Tạp chí Epsilon, Số 03, 06/2015 Tài liệu tham khảo [1] N Alon, Eigenvalues and expanders, Combinatorica 6(1986), no 2, 83-96 [2] N Alon and V Milman, λ1 - isoperimetric inequalities for graphs, and supercon- centrators, J Combin Theory Ser B 38 (1985), no 1, 73-88 [3] N Alon and J Spencer, The probabilistic method, 3rd ed., John Wiley & Sons Inc., Hoboken, NJ, 2008 [4] R Arratia and S DeSalvo, On the singularity of random Bernoulli matricesÑnovel integer partitions and lower bound expansions, Ann Comb 17 (2013), no 2, 251-274 [5] Z Bai and J Silverstein, Spectral analysis of large dimensional random matrices Second edition Springer Series in Statistics Springer, New York, 2010 [6] B Bollobás, Random graphs Second edition, Cambridge Studies in Advanced Mathematics, 73 Cambridge University Press, Cambridge, 2001 [7] B Bollobás, Combinatorics Set systems, hypergraphs, families of vectors and combinatorial probability Cambridge University Press, Cambridge, 1986 [8] C Bordenave, M Lelarge, and J Salez, The rank of diluted random graphs, Ann Probab 39 (2011), no 3, 1097-1121 [9] J Bourgain, V Vu and P M Wood, On the singularity probability of discrete random matrices, J Funct Anal 258 (2010), no 2, 559–603 [10] S Brooks and E Lindenstrauss, Non-localization of eigenfunctions on large regular graphs, Israel J Math 193 (2013), no 1, 1–14 [11] Chung, F R K.; Graham, R L.; Wilson, R M Quasirandom graphs Combinatorica (1989), no 4, 345–362 [12] F Chung, Spectral graph theory, CBMS series, no 92 (1997) 22 (23) Tạp chí Epsilon, Số 03, 06/2015 [13] K Costello, Bilinear and quadratic variants on the Littlewood-Offord problem,Israel J Math 194 (2013), no 1, 359–394 [14] K Costello and V Vu, The ranks of random graphs Random Structures and Algorithm 33 (2008), 269-285 [15] K Costello and V Vu, The rank of sparse random matrices, Combin Probab Comput 19 (2010), no 3, 321–342 [16] K Costello, T Tao and V Vu, Random symmetric matrices are almost surely singular, Duke Math J 135 (2006), no 2, 395–413 [17] Y Dekel, J Lee, and N Linial Eigenvectors of random graphs: Nodal domains.Approx- imation, Randomization, and Combinatorial Optimization Algorithms and Techniques, pages 436-448, 2008 [18] I Dimitriu and S Pal, Sparse regular random graphs: spectral density and eigenvectors, Ann Probab 40 (2012), no 5, 2197–2235 [19] A Edelman, E Kostlan and M Shub, How many eigenvalues of a random matrix are real? J Amer Math Soc (1994), no 1, 247–267 [20] A Edelman, Eigenvalues and condition numbers of random matrices, SIAM J Matrix Anal Appl (1988), 543– 560 [21] P Erdös, On a lemma of Littlewood and Offord, Bull Amer Math Soc 51 (1945), 898–902 [22] L Erdös, A Knowles, H-T Yau and J Yin, Spectral statistics of Erd?s-RvZnyi graphs I: Local semicircle law, Ann Probab 41 (2013) [23] L Erdös, A Knowles, H-T Yau and J Yin, Spectral statistics of Erd?s-RvZnyi Graphs II: Eigenvalue spacing and the extreme eigenvalues, Comm Math Phys 314 (2012), no 3, 587–640 23 (24) Tạp chí Epsilon, Số 03, 06/2015 [24] L Erdös, B Schlein and H-T Yau, Wegner estimate and level repulsion for Wigner random matrices, Int Math Res Not IMRN 2010, no 3, 436–479 [25] L Erdös and H-T Yau, Universality of local spectral statistics of random matrices, Bull Amer Math Soc (N.S.) 49 (2012), no 3, 377–414 [26] J Friedman On the second eigenvalue and random walks in random d-regular graphs Technical Report CX-TR-17288, Princeton University, August 1988 [27] J Fiedman, A proof of Alon’s second eigenvalue conjecture and related problems (English summary)Mem Amer Math Soc 195 (2008), no 910, viii+100 pp [28] J Friedman and D-E Kohler, The Relativized Second Eigenvalue Conjecture of Alon, preprint [29] Z Füredi and J Komlós, The eigenvalues of random symmetric matrices,Combinatorica (1981), no 3, 233-241 [30] V L Girko, A refinement of the central limit theorem for random determinants (Russian) Teor Veroyatnost i Primenen 42 (1997), no 1, 63–73; translation in Theory Probab Appl 42 (1997), no 1, 121–129 (1998) [31] V L Girko, A central limit theorem for random determinants Teor Veroyatnost i Mat Statist 21 (1979), 35–39, 164 [32] A Guionnet and O Zeitouni, Concentration of the spectral measure for large matrices, Electron Comm Probab (2000), 119–136 [33] H Golstein and J von Neumman, Numerical inverting of matrices of high order, Bull Amer Math Soc 53 (1947), 1021-1099 [34] G Halász, Estimates for the concentration function of combinatorial number theory and probability, Period Math Hungar (1977), no 3-4, 197–211 [35] S Janson, T Luczak and A Graphs,Wiley-Interscience (2000) 24 Rucinski, Random (25) Tạp chí Epsilon, Số 03, 06/2015 [36] J Kahn and E Szemerédi, STOC 1989 [37] J Kahn, J Komlós, E Szemerédi, On the probability that a random ±1 matrix is singular, J Amer Math Soc (1995), 223–240 [38] J Komlós, On the determinant of (0, 1) matrices, Studia Sci Math Hungar (1967) 7-22 [39] J Komlós, On the determinant of random matrices,Studia Sci Math Hungar (1968) 387–399 [40] M Krivelevich and B Sudakov, Pseudo-random graphs.More sets, graphs and numbers, 199-262, Bolyai Soc Math Stud., 15, Springer, Berlin, 2006 [41] A Lubotzky, R Phillips, and P Sarnak Ramanujan graphs, Combinatorica, 8(3):261-277, 1988 [42] G.A Margulis , Explicit group-theoretical constructions of combinatorial schemes and their application to the design of expanders and superconcentrators [in Russian] Problemy Peredachi Informatsii 24 (1988), pp 51-60 [43] B.D McKay The expected eigenvalue distribution of a large regular graph.Linear Algebra and its Applications, 40:203216, 1981 [44] P Mitra, Entrywise bounds for eigenvectors of random graphs Electron J Combin 16 (2009), no 1, Research Paper 131, [45] J E Littlewood and A C Offord, On the number of real roots of a random algebraic equation III Rec Math [Mat Sbornik] N.S 12 , (1943) 277–286 [46] A Litvak, A Pajor, M Rudelson, N Tomczak-Jaegermann, Smallest singular value of random matrices and geometry of random polytopes, Adv Math 195 (2005), no 2, 491– 523 [47] A Marcus, D Spielman and N Srivastava, Interlacing Families I: Bipartite Ramanujan Graphs of All Degrees, preprint 25 (26) Tạp chí Epsilon, Số 03, 06/2015 [48] K Maples, Symmetric random matrices over finite fields announcement, April 15, 2013, preprint [49] A Nilli, On the second eigenvalue of a graph, Discrete Mathematics 91 (1991), 207-210 [50] A Nilli, Tight estimates for eigenvalues of regular graphs, Electronic J Combinatorics 11 (2004), N9, 4pp [51] H Nguyen, On the least singular value of random symmetric matrices, Electron J Probab 17 (2012), no 53 [52] H Nguyen, Inverse Littlewood-Offord problems and the singularity of random symmetric matrices, Duke Math J 161 (2012), no 4, 545–586 [53] H Nguyen and V Vu, Small probability, inverse theorems, and applications, Erdos’ 100th Anniversary Proceeding, Bolyai Society Mathematical Studies, Vol 25 (2013) [54] H Nguyen and V Vu, Random matrices: Law of the determinant, Annals of Probability (2014), Vol 42, No 1, 146167 [55] M Rudelson, Invertibility of random matrices: norm of the inverse, Ann of Math (2) 168 (2008), no 2, 575–600 [56] M Rudelson, Lecture notes on non-aymptotic random matrix theory, notes from the AMS Short Course on Random Matrices, 2013 [57] M Rudelson and R Vershynin, The Littlewood-Offord problem and invertibility of random matrices, Adv Math 218 (2008), no 2, 600–633 [58] M Rudelson and R Vershynin, Delocalization of eigenvectors of random matrices with independent entries, preprint [59] O N Feldheim and S Sodin, A universality result for the smallest eigenvalues of certain sample covariance matrices, Geom Funct Anal 20 (2010), no 1, 88–123 [60] A Sárközy and E Szemerédi, Uber ein Problem von Erdös und Moser, Acta Arithmetica, 11 (1965) 205-208 26 (27) Tạp chí Epsilon, Số 03, 06/2015 [61] D Spielman and S-H Teng, D Spielman, S.-H Teng, Smoothed analysis of algorithms, Proceedings of the International Congress of Mathematicians, Vol I (Beijing, 2002), 597–606, Higher Ed Press, Beijing, 2002 [62] B Sudakov and V Vu, Local resilience of graphs, Random Structures Algorithms 33 (2008), no 4, 409–433 [63] T Tao and V Vu, A central limit theorem for the determinant of a Wigner matrix, Adv Math 231 (2012), no 1, 74–101 [64] T Tao and V Vu, Random matrices: universal properties of eigenvectors, Random Matrices Theory Appl (2012), no [65] T Tao and V Vu, Random matrices: Universality of the local eigenvalues statistics pdf file Acta Math 206 (2011), no 1, 127–204 [66] T Tao and V Vu, On random ±1 matrices: Singularity Determinant,Random Structures Algorithms 28 (2006), no 1, 1–23 [67] T Tao and V Vu, On the singularity probability of random Bernoulli matrices, J Amer Math Soc 20 (2007), no 3, 603–628 [68] T Tao and V Vu, Inverse Littlewood-Offord theorems and the condition number of random matrices, Annals of Math 169 (2009), 595-632 [69] T Tao and V Vu, On the permanent of random Bernoulli matrices,Advances in Mathematics 220 (2009), 657-669 [70] T Tao and V Vu, Random matrices: Universality of local spectral statistics of non-Hermitian matrices, to appear in Annals of Probability [71] T Tao and V Vu, Additive Combinatorics,Cambridge Univ Press, 2006 [72] T Tao and V Vu, Random matrices: The Universality phenomenon for Wigner ensembles, preprint, to appear in AMS lecture notes on Random Matrices, 2013 27 (28) Tạp chí Epsilon, Số 03, 06/2015 [73] T Tao and V Vu, Random matrices: the distribution of the smallest singular values, Geom Funct Anal 20 (2010), no 1, 260–297 [74] T Tao and V Vu, Random matrices: universal properties of eigenvectors, Random Matrices Theory Appl (2012), no [75] L Tran, V Vu and K Wang, Sparse random graphs: Eigenvalues and Eigenvectors, Random Structures Algorithms 42 (2013), no 1, 110–134 [76] R Vershynin, Invertibility of symmetric random matrices, Random Structures and Algorithms 44 (2014), 135–182 [77] E.P Wigner On the distribution of the roots of certain symmetric matrices.Annals of Mathematics, 67(2):325-327, 1958 [78] N.C Wormald, Models of random regular graphs,In Surveys in Combinatorics, 1999, J.D Lamb and D.A Preece, eds, pp 239-298 [79] V Vu and K Wang, Random projection, random quadratic forms, and random eigenvectors, to appear in Random Structures and Algorithms [80] M Wood, The distribution of sandpile groups of random graphs, preprint 28 (29) VĨNH THỨC NGÔ QUANG HƯNG (Đại học Buffalo, Mỹ) Tóm tắt nội dung Vĩnh Thức, định thức, và định thức Pfaff là các đa thức đa biến trên các ma trận Chúng có liên hệ mật thiết với nhau, và có ứng dụng Vật Lý thống kê, Kinh tế học, toán Tổ hợp, độ phức tạp tính toán, lý thuyết đồ thị, và thuật toán Bài viết này điểm qua lịch sử và chứng minh số kết liên kết các đối tượng Tổ hợp kỳ thú này Vĩnh Thức Gọi A = (aij ) là ma trận vuông n × n Vĩnh Thức1 A định nghĩa sau: Perm(A) = n XY aiπ(i) , π∈Sn i=1 đó Sn là tập tất các hoán vị [n] Như vậy, công thức tính vĩnh thức giống công thức Leibniz để tính định thức A, khác điểm là ta không nhân sgn(π) vào số hạng tổng trên Hàm vĩnh thức có nhiều ứng dụng Ví dụ, vấn đề toán Tổ hợp là tìm số cách lát hình chữ nhật với các quân đô-mi-nô kích thước × Mỗi cách lát hoàn hảo tương ứng với bắt cặp hoàn hảo2 đồ thị lưới tương ứng (Xem hình 3.1.) Làm nào để ta đếm tổng số cách bắt cặp hoàn hảo đồ thị lưới? Dễ thấy đồ thị lưới là đồ thị hai phần3 Giả Permanent Sau thảo luận với các anh Phạm Hi Đức và Phùng Hồ Hải, tôi định chọn từ vĩnh thức để dịch permanent Perfect matching Còn gọi là cặp ghép hoàn hảo (ban Biên tập) Bipartite graph 29 (30) Tạp chí Epsilon, Số 03, 06/2015 sử đồ thị hai phần này có n đỉnh bên trái và n đỉnh bên phải (Nếu bên có số đỉnh ít thì ta thêm vào cách đỉnh đơn lẻ cho hai bên nhau.) Sau đó, ta xây dựng ma trận A = (aij ) đó aij = đỉnh i bên trái có cạnh đến đỉnh j bên phải; và aij = không có cạnh ij đồ thị Khi đó, Perm(A) đúng tổng số các cách bắt cặp hoàn hảo đồ thị – và nó là số cách lát đô-mi-nô mà ta cần tìm Hình 3.1: Lợp hình chữ nhật × các hình đô-mi-nô × 1, và bắt cặp hoàn hảo tương ứng đồ thị lưới Vấn đề tìm số cách lát đô-mi-nô không phải là bài toán giải trí Tổ hợp thông thường Đây là vấn đề Vật lý thống kê và Hoá học trạng thái rắn [17, 16, 18] Chúng ta quay lại với mô hình dimer Vật lý đây Vĩnh Thức khởi nguyên năm 1812, các nhà Toán học người Pháp Jacques Philippe Marie Binet và Augustin-Louis Cauchy định nghĩa Trong kỷ 19 đã có khoảng chục nhà toán học nghiên cứu các đẳng thức và bất đẳng thức liên quan đến vĩnh thức, bao gồm Arthur Cayley và Thomas Muir Vương Quốc Anh Cái tên “permanent” có lẽ bắt nguồn từ Cauchy (1812), người đầu tiên thật dùng và làm “chết tên” nó là Muir (1882) Tuy nhiên, các nghiên cứu vĩnh thức thật “nóng” lên từ năm 1960 các đóng góp toán học Hà Lan 1.1 Giả định van der Waerden Năm 1926, nhà Toán học người Hà Lan Bartel Leendert van der Waerden đặt câu hỏi là vĩnh thức nhỏ ma 30 (31) Tạp chí Epsilon, Số 03, 06/2015 trận ngẫu nhiên kép4 là bao nhiêu [35] Trực quan cho thấy ma trận Jn = n1 J có lẽ là ma trận có vĩnh thức cực tiểu (Ma trận J là ma trận vuông n × n gồm toàn các số 1.) Từ đó, bất đẳng thức sau đây gọi là giả định van der Waerden5 : Perm(A) ≥ n! nn (3.1) với ma trận ngẫu nhiên kép A kích thước n × n Van Lint [37] kể rằng, năm 1969 có lần van der Waerden đến dự buổi hội thảo toán Tổ hợp Ông vốn là dân Đại số, ít làm toán tổ hợp Một diễn giả trẻ lên báo báo vấn đề liên quan đến giả định van der Waerden Van der Waerden giơ tay lên hỏi “giả định đó nói gì vậy?” Cuối buổi diễn giả xuống xem bảng tên người đặt câu hỏi Van Lint, vốn là nhà toán học cừ khôi người Hà Lan, đã truy vấn van der Waerden xem giả định này có xuất phát điểm từ đâu Van der Waerden nhớ lại hồi 1926, ông nói chuyện với O Schreier, và Schreier cho ông biết G A Miller có chứng minh có hệ đại diện chung6 cho các lớp kề7 trái và các lớp kề phải nhóm H nhóm hữu hạn G Van der Waerden quan sát các lớp kề trái là phân hoạch (R1 , , Rn ) G, đó R1 ∪ · · · ∪ Rn = G, và |Ri | = |G|/n với i ∈ [n] Bộ các lớp kề phải là phân hoạch (C1 , , Cn ) G, thoả |Cj | = |G|/n với j ∈ [n] Hệ đại diện chung cho hai phân hoạch này là các phần tử S = {g1 , , gn } ⊆ G cho |Ri ∩S| = |Cj ∩S| = 1, với i, j ∈ [n] Bây ta xây dựng ma trận A = (aij ) đó aij = |Ri ∩ Cj | thì tồn hệ đại diện chung tương đương với phát biểu Perm(A) > Do tổng các hàng và các cột A |Ri ∩Cj | Khi đó A |G|/n, ta “thường hoá”8 A cách đặt aij = |G|/n là ma trận ngẫu nhiên kép Ta có thể chứng minh Perm(A) > dễ dàng định lý König-Hall Nhưng câu hỏi khác Doubly stochastic matrix Một ma trận vuông gọi là “ngẫu nhiên kép” nó không âm, và tổng hàng và cột Van der Waerden conjecture Common system of representatives Coset Normalize 31 (32) Tạp chí Epsilon, Số 03, 06/2015 tự nhiên không kém là giá trị nhỏ Perm(A) có thể đạt tới là bao nhiêu Đây chính là nguồn gốc giả định van der Waerden Trong “Permanents” (1978 [23]), Henryk Minc ghi hồi ông nộp số bài báo vĩnh thức kỷ 20 thì có bình duyệt viên nói “chế cái tên gì lố bịch thế”? Giả thiết van der Waerden cuối cùng chứng minh, năm 1981, độc lập hai bài báo khác G P Egorychev [8] và D I Falikman [9] Cả hai dùng bất đẳng thức hình học gọi là bất đẳng thức Alexandroff-Fenchel [1] Egorychev và Falikman giải Fulkerson năm 1982 bài này Năm 2006, Leonid Gurvits [11] chứng minh bất đẳng thức tổng quát với chứng minh ngắn gọn, bao gồm hệ là Định lý van der Waerden 1.2 Giả định Minc Giả định van der Waerden nói chặn vĩnh thức lớp các ma trận định Về chặn trên thì năm 1963 Minc [22] có giả định sau Gọi A là ma trận 01, nghĩa là aij ∈ {0, 1} với i, j ∈ [n] Q Gọi mi là số số trên hàng i ma trận, thì ta có Perm(A) ≤ ni=1 (mi !)1/mi Năm 1973 Lev M Brègman [4] chứng minh giả định này, và bây nó thường gọi là Định lý Brègman.9 Năm 1977, nhà toán học lỗi lạc khác người Hà Lan, Alex Schrijver, trình bày chứng minh cực gọn và lịch [28] Ngoài ra, Schrijver tổng quát hoá định lý cho các ma trận nguyên không âm Dưới đây chúng ta ghi lại chứng minh cho trường hợp ma trận 01 Schrijver Định lý 1.1 (Định lý Brègman) Gọi A là ma trận 01 kích thước n × n Gọi mi là số các số trên hàng i A Ta có, Perm(A) ≤ n Y (mi !)1/mi (3.2) i=1 Chứng minh Cho cặp i, j ∈ [n], gọi Aij là ma trận đạt cách bỏ hàng i và cột j khỏi A Từ định nghĩa (và từ giả Đây chính là Brègman phân kỳ Brègman (Bregman divergence) xác suất thống kê 32 (33) Tạp chí Epsilon, Số 03, 06/2015 thiết A là ma trận 01), dễ thấy X Perm(A) = Perm(Aij ), với i ∈ [n] (3.3) j:aij =1 (Khai triển này giống khai triển Laplace tính định thức.) Do vế phải (3.2) là cái tích, ta thử chặn Perm(A) cách chuyển vế phải (3.3) thành cái tích các hàm Perm(Aij ), sau đó áp dụng quy nạp Cách tự nhiên để chặn trên tổng tích là dùng bất đẳng thức Jensen Cụ thể hơn, từ tính lồi hàm x ln x trên miền x > 0, bất đẳng thức Jensen cho ta biết     t1 + · · · + tm t1 ln t1 + · · · tm ln tm t1 + · · · + tm ln ≤ , m m m miễn là ti > với i ∈ [m] Từ đó, ta chặn tổng tích: (t1 + · · · + tm )t1 +···+tm ≤ mt1 +···+tm tt11 · · · ttmm Và dễ thấy bất đẳng thức này đúng với ti ≥ 0, không cần ti > trước Áp dụng vào (3.3), ta chặn trên cho Perm(A) mà vế phải là tích: Y Perm(A) Perm(A)Perm(A) ≤ mi Perm(Aij )Perm(Aij ) (3.4) j:aij =1 Bất đẳng thức (3.4) đúng với i ∈ [n] Bước tự nhiên là ta nhân chúng với để có vế phải đối xứng, và các vế phải “trung hoà” lẫn kích thước chúng khác Nói cách khác, ta dùng trung bình nhân các vế phải:  !  n n Y Y Y Perm(A) Perm(A)nPerm(A) ≤ mi · Perm(Aij )Perm(Aij )  i=1 j:aij =1 i=1 (3.5) Phát triển tự nhiên là áp dụng bất đẳng thức (3.2) cho Perm(Aij ) bên vế phải Tuy nhiên, điều phiền phức là có Perm(Aij ) trên số mũ – áp dụng vào làm loạn vế phải Cho nên, ta tìm cách viết lại thừa số thứ hai bên vế phải (3.5) chút Định nghĩa,  S = π ∈ Sn | aiπ(i) = 33 (34) Tạp chí Epsilon, Số 03, 06/2015  π ∈ S | π(i) = j Sij = Từ định nghĩa vĩnh thức, dễ thấy |S| = Perm(A) và |Sij | = Perm(Aij ) aij = Đây là “diễn giải tổ hợp” hàm vĩnh thức Ngoài ra, aij = thì |Sij | = Do đó, ta có thể viết lại thừa số thứ hai bên vế phải (3.5) sau: n Y Y n Y Y Perm(Aij )Perm(Aij ) = i=1 j:aij =1 Perm(Aij )|Sij | i=1 j:aij =1 n Y Y = Perm(Aij )|Sij | = Perm(Aij )|Sij | j:aij =0 i=1 j:aij =1 n Y n Y Y Perm(Aij )|Sij | i=1 j=1 Đến đây ta dùng cái mẹo “định trị đại lượng hai cách” phổ dụng toán Tổ hợp Trong vế phải đẳng thức trên, với cặp (i, j) thì Perm(Aij ) xuất |Sij | lần Bây tưởng tượng ta xây dựng đồ thị hai phần mà các đỉnh bên trái là các cặp (i, j), và các đỉnh bên phải là các hoán vị π ∈ S Sau đó, ta nối đỉnh (i, j) bên trái với đỉnh π bên phải và π(i) = j Với “cân nặng” Qn ta|Scho Qncạnh ij | chính là tích Perm(Aij ), thì cái tích i=1 j=1 Perm(Aij ) cân nặng các cạnh tưởng tượng nhóm theo các đỉnh bên trái đồ thị hai phần Thế thì, cách khác để nhóm cái tích các cân nặng này lại với là nhóm theo các đỉnh π ∈ S bên phải Với đỉnh π ta lấy n cạnh kề với nó, đó là các cạnh nối π với cách đỉnh (i, π(i)) bên trái Như ta suy n Y n Y Perm(Aij )|Sij | = i=1 j=1 n YY Perm(Aiπ(i) ) π∈S i=1 Còn thừa số đầu tiên vế phải (3.5) thì dễ viết lại thành n Y Perm(A) mi = i=1 n Y |S| mi i=1 = n YY mi π∈S i=1 Đến đây, ta có thể viết lại toàn bất đẳng thức (3.5) dạng dễ chịu nhiều: Perm(A) nPerm(A) ≤ n YY π∈S i=1 34 mi · Perm(Aiπ(i) ) (3.6) (35) Tạp chí Epsilon, Số 03, 06/2015 Vế phải (3.6) đã cân đối, đến lúc ta áp dụng giả thiết quy nạp rồi:   n Y i=1 Perm(Aiπ(i) ) ≤ n Y Y  Y  1/(mk −1) 1/mk   ((m − 1)!) (m !) k k   k6=i akπ(i)=1 i=1 k6=i akπ(i)=0 (3.7) Vế phải bất đẳng thức này trông có vẻ kinh dị, lại có dạng đơn giản Nó là tích “cân nặng” các cặp i 6= k Cân nặng này ((mk − 1)!)1/(mk −1) akπ(i) = 1, và (mk !)1/mk akπ(i) = Vế phải (3.7) nhóm các tích này theo i Ta có thể nhóm nó theo k (nhớ dùng cái đồ thị hai phần tưởng tượng – bên trái là i, bên phải là k, và nối (i, k) i 6= k) Nhóm theo k thì có cái lợi là cân nặng cặp i 6= k phụ thuộc vào k Lưu ý π ∈ S, đó akπ(k) = Do đó, số các i cho i 6= k và akπ(i) = chính là mk − 1, và số các i cho i 6= k và akπ(i) = chính là n − mk Do đó, vế phải (3.7) n  Y k=1 (mk − 1)! · (mk !) n−mk mk  Kết hợp với (3.6) thì ta có " n # " n #! Y Y Y n−mk  Perm(A)nPerm(A) ≤ mi · (mk − 1)! · (mk !) mk i=1 n Y π∈S = Y = n/mk (mk !) π∈S n Y k=1 k=1 (mk !)1/mk k=1 ! !nPerm(A) Đó là chứng minh tuyệt vời Alex Schrijver 1.3 Độ phức tạp tính toán Từ kỷ 18, Gauss đã cho chúng ta biết cách tính định thức thời gian O(n3 ) Chỉ khác cái dấu, vĩnh thức khó tính định thức nhiều Năm 1979, Leslie Valiant định nghĩa lớp độ phức tạp #P và chứng minh vấn đề 35 (36) Tạp chí Epsilon, Số 03, 06/2015 tính Perm(A) ma trận 01 là vấn đề #P-khó [33] Đây là công trình cột mốc mang đến giải Turing năm 2010 cho Valiant Ông có hai trai là Greg Valiant (giáo sư khoa Máy tính Stanford), và Paul Valiant (giáo sư khoa Máy tính Brown) Ở đây chúng ta mô tả nôm na kết Valiant Một vấn đề định10 là vấn đề mà câu trả lời là có không tồn lời giải Lớp P là lớp các vấn đề định mà ta có thể tìm lời giải trả lời không tồn lời giải thời gian đa thức Ví dụ, cho đồ thị G = (V, E), vấn đề bắt cặp lớn nhất11 hỏi G có tập cạnh không giao có kích thước ít k hay không Đây là vấn đề định, và thuật toán trổ hoa12 Edmonds năm 1965 giải nó thời gian O(|V |4 ) [7] Lớp NP là lớp các vấn đề định mà việc xác minh lời giải có thật là lời giải hay không có thể làm nhanh thời gian đa thức, việc tìm lời giải thì chưa Ví dụ, đó tô màu đồ thị G và nói “đây là tô màu hợp lệ dùng màu” thì ta dễ dàng kiểm tra thời gian đa thức xem người đó có thành thật hay không Nhưng cho trước đồ thị G thì ta không biết cách nào để trả lời câu hỏi “có tồn cách tô G dùng màu hay không?” thời gian đa thức Dễ thấy P ⊆ NP, chứng minh P 6= NP là câu hỏi triệu đô Hiện nay, để chứng minh vấn đề định nào đó là khó (nghĩa là khó có khả tồn thời gian đa thức để giải nó) thì thường là ta chứng minh nó khó tất các vấn đề NP Cụ thể chút, ta có thể chứng minh trả lời câu hỏi “G có tô màu không?” cách hiệu thì tất các vấn đề NP có thuật toán hiệu để giải Vì thế, vấn đề tô màu là vấn đề NP-khó Các vấn đề định là các vấn đề tính toán bit thông tin: bit (trả lời không) bit (trả lời có) Đây là giải pháp kỹ thuật để chứng minh độ khó Nhưng tất nhiên các vấn đề thực tế không là các vấn đề mà output có bit thông tin Ví dụ ta cần tính xác suất mà mạng máy tính là liên thông, cho biết trước xác suất lỗi các kết nối Bài toán này gọi là 10 Decision problem Maximum matching Còn gọi là cặp ghép cực đại (ban Biên tập) 12 Blossom algorithm 11 36 (37) Tạp chí Epsilon, Số 03, 06/2015 vấn đề độ tin cậy13 mạng Hoặc, ta cần tính Perm(A) ma trận A cho trước Rõ ràng là trên thực tế có hàng tỉ các vấn đề Lớp #P Valiant định nghĩa để nắm bắt độ khó việc tính số (thay vì bit thông tin): #P là lớp các vấn đề mà câu trả lời là số lời giải vấn đề NP Ví dụ, vấn đề tìm số cách tô đồ thị dùng màu hay vấn đề tìm số tập cạnh không giao với kích thước n/2 là các vấn đề lớp #P Rõ ràng là ta đếm số lời giải, thì ta định là có lời giải hay không Do đó, các vấn đề #P khó các vấn đề tương ứng NP Đếm số cách tô màu khó xác minh xem có cách tô màu hay không Đếm số cách bắt cặp hoàn hảo khó xác minh xem có cách bắt cặp hoàn hảo hay không Tương tự NP-khó, #P-khó là lớp các vấn đề mà – ta giải vấn đề nào lớp này cách hiệu quả, thì ta giải tất các vấn đề lớp #P cách hiệu quả, và đó ta giải tất các vấn đề NP cách hiệu Tóm lại, thuật toán thời gian đa thức cho vấn đề #P-khó nào dẫn đến hệ là P = NP Trong ngữ cảnh bài viết này thì điều này có nghĩa là khó có khả tồn thuật toán tính Perm(A) thời gian đa thức – cho dù ma trận A là ma trận 01 Không tính hiệu thì ta có hai chọn lựa tự nhiên: là tìm thuật toán xấp xỉ nó (trong thời gian đa thức), hai là tìm lớp các ma trận A mà tồn thuật toán hiệu Chọn lựa thứ hai dẫn ta đến câu hỏi Pólya và khái niệm định thức Pfaff Tuy nhiên, trước thảo luận câu hỏi Pólya và định thức Pfaff, ta cần chuẩn bị ít kiến thức hoán vị Vài quan sát hoán vị Cho hoán vị π ∈ Sn Ta có thể biểu diễn π thành dạng đồ thị có hướng, với tập đỉnh [n], và cạnh (i, π(i)) Đồ thị này là tập hợp các chu trình không giao nhau, và các chu trình này phủ toàn [n] (Xem hình 3.2.) 13 Reliability 37 (38) Tạp chí Epsilon, Số 03, 06/2015 π = (8 4) Hình 3.2: Hoán vị và cấu trúc chu trình nó Hoán vị này có chu trình chẵn Một nghịch thế14 π là cặp (i, j) cho i < j và π(i) > π(j) Gọi Inv(π) là tổng số các nghịch π Dấu π định nghĩa là sgn(π) = (−1)Inv(π) Một chuyển vị kề15 là hoán vị π = (1 · · · i − i + i i + · · · n) với i ∈ [n − 1] nào đó; hoán vị này thường viết là (i i + 1) Nếu ta hợp thành π và chuyển vị kề (i i + 1) thì ta hoán vị σ = π ◦ (i i + 1) = (π(1) · · · π(i − 1) π(i + 1) π(i) π(i + 2) · · · π(n)) Mọi hoán vị σ tuỳ ý có thể đạt đến từ hoán vị π tuỳ ý cách áp dụng nhiều chuyển vị kề Điều này tương đương với thuật toán xếp bong bóng16 Trong thuật toán xếp bong bóng, ta dùng chuyển vị kề để phần tử nhỏ dãy số cho trước “nổi” lên đầu tiên, phần tử nhỏ nhì lên vị trí thứ nhì, vân vân Nghĩa là hoán vị chuyển hoán vị đơn vị chuyển vị kề Và thì hoán vị chuyển hoán vị khác dùng chuyển vị kề Dễ thấy sgn(σ) = −sgn(π) σ là hợp thành π và chuyển vị kề Còn chuyển vị kề thay đổi cấu trúc chu trình giao hoán π nào? Nếu i và i + nằm trên cùng chu trình π, thì sau chuyển vị chu trình này bị bẻ thành hai chu trình Nếu i và i + nằm trên hai chu trình khác thì hai chu trình này trộn thành chu trình 14 Inversion Còn dịch là cặp trật tự ngược (ban Biên tập) Adjacent transposition 16 Bubble sort Còn gọi là xếp bọt (ban Biên tập) 15 38 (39) Tạp chí Epsilon, Số 03, 06/2015 Các khẳng định vừa đúng cho chuyển vị (i j), không thiết là chuyển vị kề Từ đó, ta chứng minh bài tập sau đây, là bài tập toán Tổ hợp đếm Bài tập 2.1 Chứng minh sgn(π) = (−1)e(π) , đó e(π) là số các chu trình chẵn cấu trúc chu trình π (Chu trình chẵn là chu trình có số chẵn các đỉnh.) Một tính chất thú vị các hoán vị là hai hoán vị liên hợp có cấu trúc chu trình giống hệt Cụ thể hơn, hoán vị π và σ gọi là hai hoán vị liên hợp17 π = τ ◦ σ ◦ τ −1 với τ là hoán vị nào đó Ví dụ, π = (i i + 1) ◦ σ ◦ (i i + 1)−1 thì cấu trúc chu trình π và cấu trúc chu trình σ giống y nhau, đổi chỗ i và i + Hình 3.3 minh hoạ điều này Hình 3.3: Cấu trúc chu trình σ = (5 7) ◦ π ◦ (5 7)−1 , π là hoán vị Hình 3.2 Câu hỏi Pólya Năm 1913, George Pólya [24] quan sát     a b a b det = Perm , c d −c d nghĩa là với ma trận × thì có cách đổi dấu (vài) phần tử ma trận để biến vĩnh thức thành định thức Pólya thắc mắc là điều này có đúng với ma trận hay không? Cụ thể hơn, câu hỏi Pólya sau Cho trước ma trận A = (aij ) Có tồn ma trận B = (bij ) cho bij = ±aij , với i, j ∈ [n], và, với hoán vị π ∈ Sn ta có n Y aiπ(i) = sgn(π) i=1 17 n Y i=1 Conjugate permutations 39 biπ(i) (3.8) (40) Tạp chí Epsilon, Số 03, 06/2015 Trong đó, sgn(π) ∈ {+1, −1} là dấu hoán vị π, đã định nghĩa phần trước Theo khai triển Leibniz thì det(B) = X sgn(π) π∈Sn n Y bij i=1 Do đó, tồn ma trận B trên thì Perm(A) = det(B) Từ nay, ma trận B thoả tính chất này gọi là ma trận Pólya ma trận A Cũng năm đó, Gábor SzegHo18 [31] trả lời là không, vì với n ≥ luôn tồn ma trận n × n không có ma trận Pólya tương ứng Bài tập 3.1 Gọi J3 là ma trận × gồm toàn các số Chứng minh J3 không có ma trận Pólya Dựa vào đó, chứng minh với n ≥ 3, tồn ma trận n × n không có ma trận Pólya tương ứng Câu hỏi tự nhiên tiếp theo, là câu hỏi chính phần này bài viết, sau: Câu hỏi 3.2 (Câu hỏi Pólya) Ma trận A phải nào thì có ma trận Pólya cho nó? Giả sử A có ma trận Pólya B Thì ta có thể viết B = A ◦ L là tích Hadamard A và ma trận L = (`ij ) Trong đó bij = aij `ij với i, j ∈ [n], `ij ∈ {−1, 0, 1}, và `ij 6= aij 6= Từ (3.8) ta Qn suy ma trận L phải thoả mãn tính chất sau đây Nếu i=1 aiπ(i) 6= thì n Y sgn(π) `iπ(i) = 1, (3.9) i=1 nghĩa là tất các số hạng khác khai triển Leibniz định thức L Một ma trận L với các phần tử tập {−1, 0, 1} gọi là ma trận không kỳ dị dấu19 nó thoả điều kiện là tất các số hạng khác khai triển định thức L có cùng dấu, và tồn ít số hạng khác Tập các cặp (i, j) cho aij 6= gọi là giàn tựa20 A Phân tích vừa cho thấy câu hỏi Pólya tương đương với câu hỏi sau đây: 18 Pólya thì không cần phải giới thiệu Bản thân SzegHo là nhà Toán học lớn người Hungary, đã kèm thêm cho John von Neumann 19 Sign nonsingular 20 Support 40 (41) Tạp chí Epsilon, Số 03, 06/2015 Câu hỏi 3.3 Cho trước ma trận A, nào thì tồn ma trận L có cùng giàn tựa A, và L là ma trận không kỳ dị dấu Một điều thú vị là đây là câu hỏi có ứng dụng Kinh Tế học, chính Paul Samuelson (Nobel Kinh tế, 1970) đặt “Nền tảng phân tích Kinh Tế” năm 1947 [27] Để tìm cách trả lời câu hỏi 3.3, ta xếp nó lại thành bài toán Tổ hợp Trước hết, xây dựng đồ thị hai phần G = (R ∪ C, E), đó R = [n] là tập các hàng A, C = [n] là tập các cột A, và có cạnh (i, j) ∈ E đồ thị G và aij 6= Mỗi ma trận L có cùng giàn tựa A tương ứng với phép gán các hệ số {−1, +1} vào các cạnh đồ thị G Bất kể ta gán hệ số nào, khai triển định thức L có số hạng khác và tồn bắt cặp hoàn hảo đồ thị G; vì số hạng khác khai triển định thức tương ứng với bắt cặp hoàn hảo Như vậy, không tính tổng quát ta có thể giả sử là G có ít bắt cặp hoàn hảo Bắt cặp hoàn hảo (nếu có) đồ thị hai phần G có thể tính nhiều thuật toán, ví dụ 21 thuật toán nới dài đường dẫn p thuật  toán Hopcroft–Karp  [12] với thời gian chạy là O |V (G)||E(G)| = O n5/2 Gọi π ∈ Sn là bắt cặp hoàn hảo G, nghĩa là các cạnh (i, π(i)) thuộc bắt cặp này Nếu ta xáo trộn các hàng các cột ma trận L thì ta không thay đổi tính chất không kỳ dị dấu nó Do đó, không tính tổng quát ta có thể giả sử π(i) = i cách đánh số lại các đỉnh tập R Còn nữa, ta nhân hàng cột L với giá trị −1 thì ta không thay đổi tính chất không kỳ dị dấu L Do đó, ta có thể giả sử phép gán hệ số vào các cạnh G gán số cho tất các cạnh (i, i) G Tiếp tục với hành trình xếp lại bài toán Bây ta xây dựng đồ thị có hướng D = ([n], E(D)) hai bước: (1) định hướng tất các cạnh (i, j) ∈ E(G), i ∈ R, j ∈ C, cách đổi nó thành mũi tên từ i đến j; và (2) sau đó nhập đỉnh i ∈ R và đỉnh i ∈ C G làm Sau làm điều này thì đồ thị D có n đỉnh, đỉnh i có cái vòng22 là mũi tên trỏ 21 Augmenting path algorithm Còn gọi là Thuật toán tìm đường tăng (luồng) (ban Biên tập) 22 Loop 41 (42) Tạp chí Epsilon, Số 03, 06/2015 từ i đến i Tại vì, G thì có mũi tên từ i ∈ R vào i ∈ C Ngoài ra, hoán vị π ∈ Sn cho (i, π(i)) ∈ E(G), ∀i ∈ [n], tương ứng với các chu trình không giao đồ thị D, và các chu trình này phủ toàn các đỉnh D Một tập các chu trình (có hướng) D gọi là phủ chu trình23 D Tóm lại, bài toán ta đã chuyển từ gán hệ số {−1, 1} vào các cạnh vô hướng G thành việc gán hệ số w : E(D)õ{−1, 1} vào các cạnh có hướng D Với phủ chu trình π, định nghĩa “cân nặng” nó là w(π) = sgn(π) n Y w(i, π(i)) i=1 (Ở đây ta lạm dụng ký hiệu, và dùng luôn w để ký hiệu hàm cân nặng các phủ chu trình.) Và ta muốn tìm phép gán hệ số cho w(π) = 1, với phủ chu trình π D Thật bài toán đòi hỏi các số hạng này cùng dấu, vì π(i) = i là các phủ chu trình, và cân nặng nó 1, nên ta biết tất các cân nặng các phủ chu trình phải Mặc dù đã chuyển vấn đề từ ma trận vấn đề đồ thị phần nào dễ hình dung hơn, chúng ta tuyệt nhiên không biết cách nào để (bằng thuật toán) kiểm tra xem có phép gán hệ số mong muốn hay không Vazirani và Yannakakis [40] lại xếp tiếp bài toán này Bổ đề 3.4 (Vazirani-Yannakakis, 1988) Cho đồ thị có hướng D = ([n], E) với n vòng (i, i) ∈ E Tồn phép gán hệ số w : E õ{−1, 1} cho tất các phủ chu trình D có cân nặng và tồn phép gán hệ số w : E õ{−1, 1} cho, với chu trình C D, thì có số lẻ các cạnh C với hệ số Chứng minh Giả sử tồn phép gán hệ số cho chu trình C D có số lẻ các cạnh với hệ số Như vậy, chu trình lẻ có số chẵn các hệ số −1; và chu trình chẵn có số lẻ các hệ số −1 Gọi π là phủ chu trình 23 Cycle cover 42 (43) Tạp chí Epsilon, Số 03, 06/2015 D Từ Bài tập 2.1, ta biết sgn(π) = (−1)c đó c là số các chu trình chẵn π Từ đó, dễ thấy w(π) = Ngược lại, giả sử tồn phép gán hệ số cho tất các phủ chu trình có cân nặng là Gọi C là chu trình D Nếu ta lấy C, cùng với các vòng (i, i) với i ∈ / C, thì ta có phủ chu trình π Nếu C là chu trình lẻ, thì sgn(π) = – π không có chu trình chẵn Vì thế, phải có số chẵn các hệ số −1 trên chu trình C Ngược lại, C là chu trình chẵn thì sgn(π) = −1 Vì thế, phải có số lẻ các hệ số −1 trên C Đến đây thì vấn đề rõ ràng chút Tuy nhiên, kể đó cho ta phép gán hệ số thì ta không biết cách nào để kiểm tra cách hiệu xem phép gán hệ số đó có tốt hay không Tại vì, tổng số các chu trình đồ thị cho trước có thể làm hàm mũ Ta không thể kiểm tra chu trình Đó là chưa nói đến chuyện phải qua tất các phép gán hệ số {−1, 1} Định nghĩa 3.5 Một đồ thị có hướng D gọi là đồ thị chẵn nếu, với phép gán hệ số {−1, 1} vào các cạnh D, luôn tồn chu trình D có số chẵn các hệ số Bổ đề Vazirani-Yannakakis trên đã cho ta biết câu hỏi Pólya tương đương với câu hỏi kiểm tra xem đồ thị có hướng có phải đồ thị chẵn hay không Năm 1987, Paul Seymour và Carsten Thomassen [29] nghiên cứu các ma trận không kỳ dị dấu, và đã chứng minh rằng, tồn đồ thị có hướng H cho D là đồ thị chẵn và H có chu trình với số chẵn các cạnh Và H có thể xây dựng từ D thời gian đa thức Như vậy, ta có Định lý 3.6 Câu hỏi Pólya tương đương mặt thuật toán với bài toán xác minh xem đồ thị có hướng H có chu trình với sỗ chẵn các cạnh hay không Chứng minh định lý trên không tầm thường, mặc dù không phải quá khó Xem thêm phần Chú Thích cuối bài và các tham khảo từ đó Câu hỏi Pólya đã trở thành câu hỏi rõ ràng mặt tổ hợp, ta hoàn toàn không biết cách trả lời câu hỏi thời gian đa thức Cuối cùng, đến năm 1999 thì Robertson, Seymour, và Thomas [26] thiết kế thuật toán trả lời câu hỏi này thời gian đa 43 (44) Tạp chí Epsilon, Số 03, 06/2015 thức Bài báo này tờ Annals of Mathematics Như là sau 86 năm, câu hỏi Pólya đã trả lời thoả đáng Không thế, đề cập phần tới bài, câu hỏi này có liên quan mật thiết đến đối tượng đại số tuyến tính khác – định thức Pfaff – quyến rũ không kém định thức và vĩnh thức Định thức Pfaff Cũng vĩnh thức và định thức, định thức Pfaff là đa thức đa biến, với các biến là các phần tử ma trận vuông Trong phần này bài viết, tôi giải thích Định thức Pfaff liên quan đến định thức, đến tổng số cách bắt cặp hoàn hảo đồ thị cho trước, và đo đó liên đới đến vĩnh thức 4.1 Định thức Pfaff Gọi n là số nguyên dương chẵn Gọi Fn là tập tất các phân hoạch [n] thành n/2 cặp số Như vậy, thành viên F ∈ Fn là tập n/2 cặp (i, j), i < j, cho tất các số [n] điều thuộc cặp nào đó F Hai cặp (i, j), (i0 , j ) ∈ F gọi là hai cặp cắt nhau24 i < i0 < j < j Đơn giản là ta vẽ hai cung từ i đến j và từ i0 đến j thì chúng cắt nhau, hình sau đây i0 i j0 j Gọi χ(F ) là tổng số cặp cắt F Định thức Pfaff định nghĩa sau Gọi A = (aij ) là ma trận phản xứng25 , nghĩa là A = −AT , thì định thức Pfaff A, ký hiệu là Pf(A), định nghĩa là Pf(A) = X (−1)χ(F ) F ∈Fn 24 25 Y (i,j)∈F Crossing Anti-symmetric, gọi là skew-symmetric 44 aij (3.10) (45) Tạp chí Epsilon, Số 03, 06/2015 Bài tập 4.1 Có cách khác để định nghĩa “dấu” thành viên F ∈ Fn Giả sử F = {i1 , j1 }, · · · {in/2 , jn/2 } , đó i` < j` với ` ∈ [n/2] Gọi π là hoán vị sau đây   ··· n − n π= i1 j1 i2 j2 · · · in/2 jn/2 Chứng minh (−1)χ(F ) = sgn(π) (Trong số sách bạn đọc thấy họ dùng sgn(π) thay vì (−1)χ(F ) để định nghĩa định thức Pfaff.) Theo “Chứng minh và khẳng định”26 David M Bressoud [5] thì các tổng dạng định thức Pfaff xuất hiên đầu tiên từ bài báo Johann Friedrich Pfaff từ năm 1815 Ông mô tả phương pháp giải hệ 2n phương trình vi phân bậc cách dùng biến và phương trình phụ trợ Các phương trình phụ trợ này gồm tổng số tỉ lệ mà tử số là các định thức Pfaff Lúc đó Pfaff đã gần 50 tuổi, và công trình ông không đọc rộng rãi Carl Gustav Jacob Jacobi viết bài phương pháp Pfaff Định thức Pfaff liên hệ mật thiết đến các ma trận phản xứng Các người khổng lồ Poisson, Lagrange, Laplace, và Monge đã làm việc với các ma trận này nửa đầu kỷ 18 Nhưng phải đến Jacobi nhận liên hệ định thức Pfaff và định thức ma trận Mà kể vậy, thì phải chờ đến Arthur Cayley (1847) thì người ta biết đến đẳng thức quan trọng det(A) = Pf(A)2 Trên Wikipedia thì nói đẳng thức này Thomas Muir chứng minh sách ông định thức năm 1882 Nhưng tôi tin Bressoud đúng! Duyệt nhanh qua sách Muir thì khó khẳng định vì ông không viết theo kiểu đã chứng minh cái gì Chúng ta chứng minh đẳng thức này đây dùng phương pháp tổ hợp John Stembridge [30]; đây là bài báo thật là tuyệt hảo toán tổ hợp đếm Định lý 4.2 (Cayley, 1847) Gọi A là ma trận phản xứng n × n với n chẵn Ta có, det(A) = Pf(A)2 26 Proofs and confirmations 45 (46) Tạp chí Epsilon, Số 03, 06/2015 Chứng minh John Stembridge Trước hết, chúng ta viết lại vế trái đẳng thức trên dùng thêm kiến thức là A là ma trận phản xứng Khi ta khai triển định thức det(A) = X sgn(π) n Y aiπ(i) i=1 π∈Sn thì có nhiều cặp số hạng bù trừ lẫn vì A là ma trận phản xứng Cụ thể hơn, giả sử có chu trình C π là chu trình lẻ (như chu trình (1 4) Hình 3.2) Gọi π là hoán vị có các chu trình giống hệt π, ngoại Q trừ chu trình Q lẻ C bị đảo chiều Khi đó, sgn(π) = sgn(π ), và ni=1 aiπ(i) = − ni=1 aiπ0 (i) Như hai số hạng tương ứng với π và π bù trừ lẫn Ta có thể bắt cặp các hoán vị Sn cách này Nếu π có chu trình lẻ, thì ta lấy chu trình lẻ có chứa số bé các chu trình lẻ, đảo chiều chu trình lẻ này để lấy π Đây là bắt cặp hoàn hảo các hoán vị có (ít một) chu trình lẻ Do đó, gọi En ⊆ Sn là tập tất các hoán vị [n] với toàn chu trình chẵn, ta có det(A) = X (−1)e(π) n Y (3.11) aiπ(i) i=1 π∈En (Nhớ rằng, đã định nghĩa Bài tập 2.1, e(π) là số chu trình chẵn π.) Bây ta viết lại vế phải đẳng thức cần chứng minh     X Y X Y Pf(A)2 =  (−1)χ(F ) aij  ·  (−1)χ(F ) j  F ∈Fn = (i,j)∈F X (F,F )∈Fn ×Fn = X  (−1)χ(F )+χ(F )  F ∈Fn Y (i,j)∈F (−1)χ(F )+χ(F (F,F )∈Fn ×Fn 0) Y   aij  ·  (i0 ,j )∈F Y (i0 ,j )∈F aij  j  (i,j)∈F ∪F Kế hoạch là tìm song ánh En và Fn × Fn cho, với π ∈ En có cặp (F, F ) tương ứng (và ngược lại) để có đẳng thức sau đây là xong: e(π) (−1) n Y aiπ(i) = (−1)χ(F )+χ(F 0) Y (i,j)∈F ∪F i=1 46 aij (3.12) (47) Tạp chí Epsilon, Số 03, 06/2015 Tìm song ánh En và Fn × Fn là điều dễ hàng Một cặp (F, F ) ∈ Fn × Fn là cặp (có thứ tự) bắt cặp hoàn hảo các số [n] Ta gọi các cạnh F là cạnh xanh và cạnh F là cạnh đỏ Có tất n/2 cạnh xanh, n/2 cạnh đỏ, và tập các cạnh cùng màu là bắt cặp hoàn hảo Khi ta vẽ các cạnh xanh và đỏ vào cùng đồ thị với [n] là tập đỉnh, thì đỉnh có bậc đúng Cụ thể thì đỉnh kề với cạnh xanh và cạnh đỏ Do đó, tập n cạnh này tạo thành phủ chu trình [n] Mỗi chu trình phủ chu trình này có chiều dài chẵn, vì ta vòng theo chu trình ta gặp các màu xanh và đỏ luân phiên Cho đến đây thì ta chưa hoán vị vì phủ chu trình này chưa có hướng Để định hướng cho chu trình thì ta đỉnh nhỏ chu trình, và theo cạnh xanh trước Dễ thấy đây là song ánh, vì cho trước phủ chu trình π ∈ En , ta làm ngược lại: từ đỉnh nhỏ chu trình, và tô màu xanh đỏ thay phiên cho các cạnh Hình 3.4 minh họa song ánh này Ta gọi song ánh này là song ánh Stembridge Tương ứng với hoán vị π = (5 3) Hình 3.4: Song ánh En và Fn × Fn Như vậy, điều còn lại để chứng minh là chứng minh sgn(π) có cùng “dấu” với cặp ảnh (F, F ) nó song ánh Stembridge Với π ∈ En , gọi r(π) = {i | π(i) < i} thì đẳng thức cần chứng minh (3.12) tương đương với e(π) + r(π) = χ(F ) + χ(F ) (mod 2) (3.13) Ví dụ, đẳng thức này tất nhiên là đúng với π = (2 n n − 1) vì đó e(π) = n/2 và r(π) = n/2 Bây giả sử (3.13) đúng với hoán vị π nào đó, ta chứng minh là nó đúng với hoán vị liên hợp σ = (i i + 1) ◦ π ◦ (i i + 1)−1 Gọi (F, F ) là ảnh π Ta phân biệt hai trường hợp: 47 (48) Tạp chí Epsilon, Số 03, 06/2015 • Nếu i và i+1 nằm kề cùng chu trình π, thì hoán chuyển i và i + ta thay đổi r(π) mod 2, giữ nguyên e(π) Không khó để thấy ta thay đổi là χ(F ) mod là χ(F ) mod 2, không thay đổi hai Ví dụ, i và i+1 có cạnh màu xanh, thì hoán chuyển vị trí i và i + không thay đổi số điểm cắt các cạnh màu xanh Còn i và i + kề với hai cạnh màu đỏ khác Nếu chúng không cắt thì cắt sau hoán chuyển, và chúng đã cắt thì không cắt sau hoán chuyển • Nếu i và i+1 không nằm kề cùng chu trình π thì hoán chuyển i và i + giữ r(π) (và e(π)) nguyên trạng; lần này χ(F ) và χ(F ) thay đổi Như vậy, để chứng minh (3.13) đúng cho π, ta cần chứng minh (3.13) đúng cho σ – miễn là ta có thể chuyển π thành σ (nhiều lần) đổi chỗ hai số nguyên liên tiếp Mà cách đổi chỗ hai số nguyên liên tiếp thì ta có thể chuyển π thành hoán vị mà chu trình thứ C1 chứa các số nguyên nhỏ {1, 2, , |C1 |}, chu trình thứ hai chứa các số nguyên kế tiếp, theo cùng thứ tự, vân vân Dễ thấy (3.13) đúng với σ thoả tính chất này 4.2 Định hướng Pfaff Gọi A = (aij ) là ma trận kề27 đồ thị vô hướng G = Q(V, E), χ(F ) nghĩa là aij = ij là cạnh Mỗi số hạng (−1) ij∈F aij khai triển định thức Pfaff A khác và F là bắt cặp hoàn hảo G Tiếc là A là ma trận đối xứng không phải ma trận phản xứng Năm 1961, các nhà Vật lý Pieter Kasteleyn [14, 15] (người Hà Lan), Harold Neville Vazeille Temperley và Michael Fisher [32] (người Anh) nhận ý tưởng đơn giản Giả sử ta định hướng các cạnh ij G, và gán aij = iõj còn aij = −1 j õi, thì ma trận A định hướng G là ma trận phản xứng Khi đó, tồn định hướng G cho tất các số hạng khác khai triển định thức Pfaff A có cùng dấu, thì đó Định Lý 4.2 cho ta cách đếm nhanh số các bắt cặp hoàn hảo đồ thị G dùng định thức! Một định hướng G thoả tính chất 27 Adjacency matrix 48 (49) Tạp chí Epsilon, Số 03, 06/2015 này gọi là định hướng Pfaffian đồ thị Và, thuật toán này bây ta gọi là thuật toán FKT Hai câu hỏi ta cần trả lời là: • Khi nào thì tồn định hướng Pfaff cho G? • Nếu biết định hướng Pfaff tồn tại, thì làm nào để tìm nó cách hiệu quả? Để tìm cách trả lời các câu hỏi này, ta tìm miêu tả mang tính tổ hợp định hướng Pfaff Một chu trình C G là chu trình tốt nó có số chẵn các đỉnh và có bắt → − cặp hoàn hảo đồ thị G − C Cho định hướng G G thì chu trình C gọi là chu trình định hướng lẻ vòng quanh C theo chiều định thì số mũi tên xuôi chiều ta thấy là số lẻ Tất nhiên, vì C chẵn nên ta theo chiều ngược lại thì số mũi tên xuôi chiều là số lẻ → − → − Bổ đề 4.3 Cho G là định hướng G, thì G là định hướng Pfaff và tất các chu trình tốt G → − G định hướng lẻ → − Chứng minh Gọi A là ma trận kề G , nghĩa là aij = → − → − (i, j) ∈ E( G ), aij = −1 (j, i) ∈ E( G ), ngoài thì aij = Gọi → − F và F là hai bắt cặp hoàn hảo G, thì G là định hướng Pfaff và Y Y (−1)χ(F ) aij = (−1)χ(F ) j , (i0 ,j )∈F (i,j)∈F với cặp (F, F ) Đẳng thức này tương đương với Y (−1)χ(F )+χ(F ) aij = (3.14) (i,j)∈F ∪F Lưu ý F, F ∈ Fn , và (i, j) ∈ F ∪ F thì i < j Gọi π là ảnh (F, F ) song ánh Stembridge Từ đẳng thức (3.13), ta suy (3.14) tương đương với Y (−1)e(π)+r(π) aij = (3.15) (i,j)∈F ∪F Đến đây, ta bắt đầu chứng minh chiều thuận bổ đề Giả → − sử G là định hướng Pfaff G Gọi C là chu trình 49 (50) Tạp chí Epsilon, Số 03, 06/2015 tốt Gọi M là bắt cặp hoàn hảo G − C Ta có thể tách C thành hai bắt cặp C1 và C2 cách vòng theo C, bỏ cạnh vào C1 , cạnh kế vào C2 , luân phiên Định nghĩa F = M ∪ C1 và F = M ∪ C2 , thì F và F là hai bắt cặp hoàn hảo G Gọi π là ảnh cặp (F, F ) qua song ánh Stembridge, thì phân tích trên đẳng thức (3.15) thoả mãn Gọi V (C) là tập các đỉnh C, và E(C) là tập các cạnh Định nghĩa r(C) = |{i ∈ V (C) | π(i) < i}| Dễ thấy e(π) = |M | + r(π) = |M | + r(C) Định nghĩa ( i<j s(i, j) = −1 j ≥ i Do A là ma trận phản xứng, i < j ta có aij = s(i, j)aij và j < i ta có aji = s(i, j)aij Vì thế, Y Y = (i,j)∈F ∪F ij∈E(C) i<j Y = i∈V (C) aij · Y a2ij ij∈M s(i, π(i)) · aiπ(i) = (−1)r(C) Y aiπ(i) i∈V (C) Các đẳng thức vừa rồi, cộng với đẳng thức (3.15) dẫn đến −1, nghĩa là C định hướng lẻ Q i∈V (C) Cho chiều ngược lại, giả sử chu trình tốt định hướng lẻ Ta chứng minh đẳng thức (3.15) đúng với cặp bắt cặp hoàn hảo F và F Dễ thấy F ∪ F là hội k chu trình tốt C1 ∪ · · · ∪ Ck Tương tự trên, k X e(π) + r(π) ≡ (r(C` ) + 1) `=1 50 (mod 2) aiπ(i) = (51) Tạp chí Epsilon, Số 03, 06/2015 Và Y (i,j)∈F ∪F = k Y `=1  (−1)r(C` ) Y i∈V (C)  aiπ(i)  = k Y   (−1)r(C` ) · (−1) `=1 Do đó (3.15) đã chứng minh Bạn đọc hẳn đã nhận tương đồng bài toán tìm định hướng Pfaff đồ thị và câu hỏi Pólya Điều này không phải là ngẫu nhiên Hai bài toán này tương đương với (về mặt thuật toán) G là đồ thị hai phần; xem chứng minh bài Vazirani và Yannakakis [40] Tuy nhiên, kết FKT bước theo hướng khác, đẹp Họ chứng minh luôn tồn định hướng Pfaff G là đồ thị phẳng,28 nghĩa là ta có thể vẽ G trên mặt phẳng cho không có cạnh nào giao cạnh nào Định lý 4.4 (FKT, 1961) Nếu G là đồ thị phẳng thì luôn tồn định hướng Pfaff cho nó Chứng minh Ta xét cách vẽ G trên mặt phẳng tuỳ ý.29 Không tính tổng quát ta giả sử G là đồ thị liên thông Đồ thị G chia mặt phẳng thành nhiều mặt30 , đó phần vô hạn ngoài G gọi là mặt vô hạn G Các mặt còn lại là các mặt hữu hạn Nếu G không phải là cây thì có ít mặt hữu hạn, và mặt vô hạn ráp gianh với số mặt hữu hạn Xem Hình 3.5 Trước hết, ta chứng minh tồn cách định hướng các cạnh G cho điều sau đây đúng: ta theo chiều kim đồng hồ vòng quanh mặt hữu hạn bất kỳ, thì tổng số mũi tên xuôi chiều ta gặp là số lẻ Tạm gọi tính chất này là tính chất lẻ địa phương các mặt Lưu ý rằng, ta làm việc với cách vẽ G cụ thể trên mặt phẳng, ta có thể dùng “thuận chiều kim đồng hồ” mà không ngượng mồm Với đồ thị chưa vẽ trên mặt phẳng, khái niệm “thuận chiều kim đồng hồ” không có định nghĩa rõ ràng 28 Planar graph Embedding of G 30 Face 29 51 (52) Tạp chí Epsilon, Số 03, 06/2015 F2 F3 Mặt vô hạn F5 e F4 F1 Công thức Euler: 12 − 15 + = Hình 3.5: Một đồ thị phẳng, 12 đỉnh, 15 cạnh, mặt Ta chứng minh khẳng định này quy nạp Nếu G là cây thì tất nhiên điều này đúng vì không có mặt hữu hạn nào Nếu G không phải là cây Gọi e là cạnh nằm biên giới mặt vô hạn và mặt hữu hạn (ví dụ cạnh e Hình 3.5) Theo quy nạp thì ta có thể định hướng tất các cạnh G cho thoả tính chất “lẻ địa phương” trên Bây giờ, ta ráp cạnh e vào lại và định hướng e cho mặt hữu hạn chứa e lẻ địa phương Kế đến, ta chứng minh định hướng lẻ địa phương là định hướng Pfaff Theo Bổ Đề 4.3, ta cần chứng minh → − chu trình tốt G định hướng lẻ Gọi C là chu trình Giả sử C phủ f mặt hữu hạn bên nó Gọi wC là số cạnh thuận chiều kim đồng hồ vòng theo C Với mặt hữu hạn Fi , i ∈ [f ], bên C, gọi wi là số cạnh thuận chiều P kim đồng hồ trên mặt đó Thì ta cóP wi là lẻ với i ∈ [f ]; f đó i=1 wi ≡ f (mod 2) Trong cái tổng i wi thì cạnh bên C đếm đúng lần, vì nó thuận chiều với mặt kề nó thì ngược chiều với mặt bên Do đó, ta gọi eC là tổng số cạnh trên chu trình C, và eI là tổng số cạnh bên C, thì ta có f X i=1 wi ≡ wC + eI ≡ wC + (eI + eC ) (mod 2) (Nhớ là C là chu trình tốt, nên nó là chu trình chẵn.) Gọi v là tổng số đỉnh nằm trên C nằm C, thì v là số chẵn 52 (53) Tạp chí Epsilon, Số 03, 06/2015 vì G − C có bắt cặp hoàn hảo; nghĩa là các đỉnh bên C có thể bắt cặp hoàn hảo với Theo công thức Euler thì v − (eC + eI ) + (f + 1) = (Vì ta phải đếm mặt vô hạn công thức Euler.) Do đó wC ≡ f X i=1 wi − (eI + eC ) ≡ f − (eI + eC ) ≡ v − ≡ (mod 2) Công thức sau đây là các công thức toán Tổ hợp đếm mà số nguyên (mình cần đếm) lại biểu diễn thành hàm các số vô tỉ Công thức Fibonacci là ví dụ khác Nhưng công thức “ngầu” có lẽ là công thức Ramanujan đếm tổng số phân hoạch số nguyên – nó bao gồm tích phân Cauchy trên miền số phức Định lý 4.5 Gọi G là đồ thị lưới m × n với mn là số chẵn Thì, tổng số cách bắt cặp hoàn hảo G là     n  m Y Y πk π` 2 cos + cos m+1 n+1 k=1 `=1 mn Chứng minh Không tính tổng quát, ta giả sử n chẵn Định hướng Hình 3.6 thoả mãn tính chất là các mặt hữu hạn định hướng lẻ Do đó, đây là định hướng Pfaff đồ thị lưới Ta đánh số các đỉnh hình; 33 34 35 36 37 38 39 40 25 26 27 28 29 30 31 32 17 18 19 20 21 22 23 24 10 11 12 13 14 15 16 Hình 3.6: Định hướng Pfaff đồ thị lưới × 53 (54) Tạp chí Epsilon, Số 03, 06/2015 hàng có n = đỉnh và cột có m = đỉnh Ma trận kề định hướng này có thể viết dạng sau đây   X In −In −X In      −In X In   A=  −I −X I n n       In m−1 −In (−1) X đó, X là ma trận n × n có  −1   −1  X=    dạng: 1 −1         1 −1 Ma trận X là ma trận kề các hàng đồ thị lưới Hàng thứ các mũi tên trỏ từ nhỏ đến lớn, ta có X Hàng thứ hai có các mũi tên trỏ ngược lại, ta có −X, và luân phiên Ma trận I là ma trận đơn vị, kề các đỉnh các hàng cạnh trên ma trận lưới Chúng ta cần tính det(A) Để đơn giản hoá bài toán, ta viết lại A chút Nếu ta nhân hàng nào đó A với −1 cột nào đó với −1 thì ta thay đổi dấu det(A) không thay đổi trị tuyệt đối Ta tìm cách nhân ma trận A đối xứng Các ma trận đối xứng thì dễ tìm trị đặc trưng Và định thức thì tích các trị đặc trưng Để biến A thành ma trận đối xứng, ta đổi dấu khối n cột thứ A, đổi dấu khối n hàng thứ ba và thứ tư, đổi dấu khối n cột thứ tư và thứ năm, đổi dấu khối hàng thứ và thứ 8, vân vân Ta đạt ma trận   −X In  In −X In      I −X I n n   M=  In −X In    In    In −X 54 (55) Tạp chí Epsilon, Số 03, 06/2015 Định nghĩa Y là ma trận m × m   1     1    Y=  1      1 Thì dễ thấy M = −Im ⊗ X + Y ⊗ In đó ⊗ là tích Kronecker hai ma trận Ma trận M còn gọi là tổng Kronecker hai ma trận −X và Y Nhờ Bài tập 4.6, tác vụ ta là tìm trị đặc trưng X và Y Cả X và Y là các ma trận Toeplitz có ba đường chéo Theo kπ , k ∈ [m], và X có bài tập 4.7 thì Y có trị đặc trưng là cos m+1 `π trị đặc trưng là 2i cos n+1 , ` ∈ [n] Như vậy, với k ∈ [m] ta có trị đặc trưng cho M  và ` ∈ [n] `π `π kπ , nhân tất là cos m+1 − i cos n+1 Do cos n+1 = − cos (n+1−`)π n+1 các trị đặc trưng này với ta mn m Y n  Y cos2 k=1 `=1 `π kπ + cos2 m+1 n+1 1/2 Bài tập 4.6 Gọi A là ma trận n × n, B là ma trận m × m, α là trị đặc trưng A, và β là trị đặc trưng B Chứng minh α + β là trị đặc trưng tổng Kronecker A ⊕ B := Im ⊕ A + B ⊕ In Bội trị đặc trưng (α + β) là gì? Bài tập 4.7 Chứng minh kết đại số tuyến tính, dùng phổ biến các phương trình vi phân và 55 (56) Tạp chí Epsilon, Số 03, 06/2015 phương trình sai phân Ma  a b    Z=    trận n × n c a c b a c b a       c   c b a √ kπ , ≤ k ≤ n Gợi ý: giả sử có trị đặc trưng là a + bc cos n+1 v = (v1 , , ) là vector đặc trưng với trị đặc trưng λ Từ phương trình Zv = λv, ta có phương trình sai phân vk+1 + vk−1 = λvk , với k ∈ [n], đó các điều kiện biên là v0 = 0, vn+1 = Từ đó tìm n nghiệm λ Hỏi: các vectors đặc trưng là gì? Có phụ thuộc vào a, b, c không? Chú thích Nhiều bài báo và sách chú giải các chứng minh khác giả thiết van der Waerden [39, 19, 36, 37, 38] Các chứng minh này khá ngắn gọn và dễ hiểu Có hai chứng minh khác định lý Brègman, dựa trên chứng minh Schrijver Một chứng minh dùng phương pháp xác suất sách Alon và Spencer [2] Chứng minh dùng entropy có điều kiện Radhakrishnan [25] Tuy nhiên các chứng minh này không tự nhiên chứng minh Schrijver Quyển Arora và Barak [3] là tham khảo đại độ phức tạp tính toán Đọc thêm Mark Jerrum [13] để biết thêm #P và các cách xấp xỉ các vấn đề #P-khó Quyển sách Lovász và Plummer [20] là tham khảo tốt định thức Pfaff toán Tổ hợp William McCuaig [21] liệt kê khoảng 40 câu hỏi tương đương với câu hỏi Pólya Quyển “Chứng minh và khẳng định” David M Bressoud [5] là sách tuyệt hảo bài toán tiếng tổ hợp đếm Trong đó có mô tả vai trò định thức Pfaff chứng minh giả định gọi là giả định ma trận đảo dấu31 31 Alternating sign matrix conjecture 56 (57) Tạp chí Epsilon, Số 03, 06/2015 Kasteleyn là chữ K bất đẳng thức FKG có nhiều ứng dụng toán Tổ hợp mà chúng ta tìm dịp viếng thăm Như đã viết, chúng ta biết cách kiểm tra thời gian đa thức xem đồ thị hai phần có định hướng Pfaff hay không Nhưng bài toán này là bài toán mở cho đồ thị tổng quát: chúng ta không biết vấn đề này nằm P hay nó là NP-khó Định thức Pfaff đóng vai trò quan trọng Holographic algorithms [34, 6], và các mô hình đồ thị xác suất phẳng học máy [10] Tài liệu tham khảo [1] ALEXANDROFF, A Über die Oberflächenfunktion eines konvexen Körpers (Bemerkung zur Arbeit “Zur Theorie der gemischten Volumina von konvexen Körpern”) Rec Math N.S [Mat Sbornik] 6(48) (1939), 167–174 [2] ALON, N., AND SPENCER, J H The probabilistic method, second ed Wiley-Interscience Series in Discrete Mathematics and Optimization Wiley-Interscience [John Wiley & Sons], New York, 2000 With an appendix on the life and work of Paul ErdHos [3] ARORA , S., AND BARAK, B Computational complexity Cambridge University Press, Cambridge, 2009 A modern approach [4] BRÈGMAN, L M Certain properties of nonnegative matrices and their permanents Dokl Akad Nauk SSSR 211 (1973), 27–30 [5] BRESSOUD, D M Proofs and confirmations MAA Spectrum Mathematical Association of America, Washington, DC; Cambridge University Press, Cambridge, 1999 The story of the alternating sign matrix conjecture [6] CAI, J., AND LU, P Holographic algorithms: From art to science J Comput Syst Sci 77, (2011), 41–61 [7] EDMONDS, J Paths, trees, and flowers Canad J Math 17 (1965), 449–467 57 (58) Tạp chí Epsilon, Số 03, 06/2015 [8] EGORYCHEV, G P The solution of van der Waerden’s problem for permanents Adv in Math 42, (1981), 299–305 [9] FALIKMAN, D I Proof of the van der Waerden conjecture on the permanent of a doubly stochastic matrix Mat Zametki 29, (1981), 931–938, 957 [10] GLOBERSON, A., AND JAAKKOL A , T Approximate inference using planar graph decomposition In Advances in Neural Information Processing Systems 19, Proceedings of the Twentieth Annual Conference on Neural Information Processing Systems, Vancouver, British Columbia, Canada, December 4-7, 2006 (2006), B Schölkopf, J C Platt, and T Hoffman, Eds., MIT Press, pp 473–480 [11] GURVITS, L Hyperbolic polynomials approach to Van der Waerden/Schrijver-Valiant like conjectures: sharper bounds, simpler proofs and algorithmic applications In STOC’06: Proceedings of the 38th Annual ACM Symposium on Theory of Computing ACM, New York, 2006, pp 417– 426 [12] HOPCROFT, J E., AND KARP, R M An n5/2 algorithm for maximum matchings in bipartite graphs SIAM J Comput (1973), 225–231 [13] JERRUM, M Counting, sampling and integrating: algorithms and complexity Lectures in Mathematics ETH Zürich Birkhäuser Verlag, Basel, 2003 [14] KASTELEYN, P W The statistics of dimers on a lattice : I The number of dimer arrangements on a quadratic lattice Physica 27 (Dec 1961), 1209–1225 [15] KASTELEYN, P W Dimer statistics and phase transitions J Mathematical Phys (1963), 287–293 [16] KENYON, C., RANDALL, D., AND SINCL AIR, A Approximating the number of monomer-dimer coverings of a lattice J Statist Phys 83, 3-4 (1996), 637–659 [17] KENYON, R An introduction to the dimer model In School and Conference on Probability Theory, ICTP Lect Notes, 58 (59) Tạp chí Epsilon, Số 03, 06/2015 XVII Abdus Salam Int Cent Theoret Phys., Trieste, 2004, pp 267–304 (electronic) [18] KENYON, R The dimer model In Exact methods in lowdimensional statistical physics and quantum computing Oxford Univ Press, Oxford, 2010, pp 341–361 [19] KNUTH, D E A permanent inequality Amer Math Monthly 88, 10 (1981), 731–740, 798 [20] L OVÁSZ, L., AND PLUMMER, M D Matching theory NorthHolland Publishing Co., Amsterdam, 1986 Annals of Discrete Mathematics, 29 [21] MCCUAIG, W Pólya’s permanent problem Electron J Combin 11, (2004), Research Paper 79, 83 pp (electronic) [22] MINC, H Upper bounds for permanents of (0, 1)-matrices Bull Amer Math Soc 69 (1963), 789–791 [23] MINC, H Permanents, vol of Encyclopedia of Mathematics and its Applications Addison-Wesley Publishing Co., Reading, Mass., 1978 With a foreword by Marvin Marcus [24] PÓLYA , G Aufgabe 424 Arch Math Phys 20 (1913), 271 [25] RADHAKRISHNAN, J Entropy and counting Computational Mathematics, Modelling and Algorithms (2003) [26] ROBERTSON, N., SEYMOUR, P D., AND THOMAS, R Permanents, Pfaffian orientations, and even directed circuits Ann of Math (2) 150, (1999), 929–975 [27] SAMUEL SON, P A Foundations of Economic Analysis, first edition ed Harvard University Press Cambridge, Mass ; London, England, 1947 [28] SCHRIJVER, A A short proof of Minc’s conjecture J Combinatorial Theory Ser A 25, (1978), 80–83 [29] SEYMOUR, P., AND THOMASSEN, C Characterization of even directed graphs J Combin Theory Ser B 42, (1987), 36–45 [30] S TEMBRIDGE, J R Nonintersecting paths, Pfaffians, and plane partitions Adv Math 83, (1990), 96–131 59 (60) Tạp chí Epsilon, Số 03, 06/2015 [31] SZEGHO, G Lösung zu 424 Arch Math Phys 21 (1913), 291–292 [32] TEMPERLEY, H N V., AND FISHER, M Dimer problem in statistical mechanics-an exact result Philosophical Magazine (Aug 1961), 1061–1063 [33] VALIANT, L G The complexity of computing the permanent Theoret Comput Sci 8, (1979), 189–201 [34] VALIANT, L G Holographic algorithms SIAM J Comput 37, (2008), 1565–1594 [35] WAERDEN, B B Aufgabe 45 Jahresber Dtsch Math.-Ver 35 (1926), 117 [36] LINT, J H Notes on Egoritsjev’s proof of the van der Waerden conjecture Linear Algebra Appl 39 (1981), 1–8 [37] VAN L INT, J H The van der Waerden conjecture: two proofs in one year Math Intelligencer 4, (1982), 72–77 [38] LINT, J H The van der Waerden conjecture In Combinatorics ’81 (Rome, 1981), vol 18 of Ann Discrete Math North-Holland, Amsterdam-New York, 1983, pp 575–580 [39] VAN L INT, J H., AND W IL SON, R M A course in combinatorics, second ed Cambridge University Press, Cambridge, 2001 VAN DER VAN VAN [40] VAZIRANI, V V., AND YANNAKAKIS, M Pfaffian orientations, 0-1 permanents, and even cycles in directed graphs Discrete Appl Math 25, 1-2 (1989), 179–190 Combinatorics and complexity (Chicago, IL, 1987) 60 (61) TỰ HỌC LÀ TỐT NHƯNG CÓ THẦY TỐT HƠN NGUYỄN TIẾN DŨNG (Đại học Toulouse, Pháp) Giới thiệu Nguyễn Tiến Dũng là giáo sư Đại học Toulouse, Pháp Thời còn học sinh, Nguyễn Tiến Dũng đã tham gia kỳ thi Olymic Toán học quốc tế dành cho học sinh phổ thông (IMO) năm 1985 và đạt huy chương vàng 15 tuổi GS Nguyễn Tiến Dũng có nhiều bài viết toán học và giảng dạy toán học đăng trên trang web cá nhân zung.zetamu.net Vừa rồi, thông qua công ty Sputnik, Nguyễn Tiến Dũng đã phát hành miễn phí điện tử sách “Học toán và dạy toán nào?” để chia sẻ quan điểm anh vấn đề thực tế này Được đồng ý GS Nguyễn Tiến Dũng, chúng tôi xin trích đăng mục chương sách Toàn văn sách với nhiều quan điểm thẳng thắn và thú vị này bạn đọc có thể tải các địa sau: • Trang bán hàng trên mạng Sputnik Education • Tại http://sputnikedu.com/ Toán học có nghĩa và toán học vô nghĩa Vì có người, trường học toán toàn bị điểm kém, chợ hay bán hàng lại tính nhẩm nhanh gió, xác định giỏi các thứ làm ăn lỗ lãi sao, v.v.? Đó là vì, cái thứ toán mà họ phải dùng là “toán có nghĩa”, và nó có nghĩa với họ, thì họ trở nên quen thuộc với nó Còn thứ toán trường học họ nhiều là toán “vô nghĩa”, “thừa”, “không dùng vào đâu cả”, và đó học không vào 61 (62) Tạp chí Epsilon, Số 03, 06/2015 Một ví dụ là phép tính tích phân (có chương trình toán PTTH) Trong thăm dò ý kiến trên trang facebook Sputnik Education vào đầu năm 2014, hầu hết người trả lời nói họ chẳng cần dùng đến tích phân nào Câu hỏi đặt là: dạy tích phân, số phức, v.v chương trình phổ thông làm gì, chẳng sau này dùng đến chúng? Trong thảo luận cải cách giáo dục, đã có nhiều người nêu ý kiến nên bỏ thứ này Không Việt Nam, mà trên giới có nhiều người, kể trưởng giáo dục, cho chương trình toán phổ thông nước họ quá nặng, quá thừa Họ muốn cắt giảm bớt chương trình và số học toán phổ thông đi, chí đến nửa, và thay vào đó là môn học khác, ví dụ môn chăn ngựa Trong số các lý họ đưa ra, ngoài chuyện nhiều thứ toán dạy phổ thông là không cần thiết, còn có thêm lý là thời đại máy tính, các tính toán đã có máy tính là cho rồi, cần học toán nhiều làm gì Cả hai lý trên (đã có máy tính làm toán thay, và chương trình toán chứa nhiều thứ “vô dụng”), nhìn có vẻ có lý, thực không hợp lý Học toán không đơn là học phép tính, mà còn là học nhiều kiến thức và kỹ quan trọng khác, là khả suy luận lô-gích, chiến lược, phân biệt đúng sai, mô hình hóa các vấn đề, v.v đã bàn phía trên Máy tính có thể giúp chúng ta tính toán, tra cứu, v.v., không thể hiểu thay chúng ta Chúng ta cần phải hiểu toán, để giao đúng đầu bài cho máy tính thực hiện, và hiểu đúng ý nghĩa kết mà máy tính đưa Và lúc nào phải ỷ lại vào máy tính thì người ngày càng ngu đần đi, trở thành thứ nô lệ Các khái niệm toán học chương trình phổ thông nói chung thực là khái niệm kinh điển, và vạn năng, không “vô nghĩa” tẹo nào Việc chúng trở nên “vô nghĩa” không phải lỗi thân các khái niệm đó, mà là cách dạy và cách học quá hình thức thiên mẹo mức tính toán, mà không chú ý đến chất và và ứng dụng các khái niệm Thậm chí, theo tôi biết, có 62 (63) Tạp chí Epsilon, Số 03, 06/2015 người học toán đến bậc tiến sĩ chưa hiểu chất khái niệm tích phân Việc dạy và học toán theo lối “toán vô nghĩa” (không thấy công dụng đâu) có tác hại là làm cho nhiều người trở nên chán ghét môn toán, còn người mà “thích nghi” với lối học đó thì lại dễ bị tự kỷ hình thức chủ nghĩa Bởi vậy, cần tăng cường tìm hiểu chất và ý nghĩa các khái niệm học toán, hình thành chúng và các ứng dụng chúng, lý vì chúng tồn Câu hỏi “nó dùng để làm gì” quan trọng là câu hỏi “nó định nghĩa nào” Có thì các kiến thức toán học trở nên có nghĩa và hữu dụng Quay lại ví dụ khái niệm tích phân Einstein có nói: “Chúa không quan tâm đến các khó khăn toán học người, vì Chúa tính tích phân cách thực nghiệm” Trong sống hàng ngày, nhiều chúng ta “tính tích phân theo cách Chúa”, không phải là dùng công thức toán học viết cách chi li hình thức, mà là quan sát, ước lượng trực giác, v.v Ví dụ như, chúng ta ước lượng diện tích cái nhà, thể tích thùng rượu, thời gian để làm việc gì đó, v.v., là chúng ta “tính tích phân” Tích phân chẳng qua là tổng nhiều thành phần lại với nhau, với số thành phần có thể là vô hạn (chia nhỏ thành tổng các thành phần “nhỏ li ti”), và là công cụ để tính toán hay ước lượng độ lớn vạn vật: thể tích, diện tích, độ dài, vận tốc, trọng lượng, thời gian, tiền bạc, tăng trưởng dân số, bệnh dịch, v.v Bản thân cái ký hiệu phép lấy tích phân chính là chữ S kéo dài ra, mà S đây có nghĩa là summa (tổng) Khi học tích phân, quan trọng là hiểu ý tưởng tích phân chẳng qua là tổng và là công cụ để tính toán ước lượng các thứ qua các phép biến đổi Đấy là ý tưởng sáng, chẳng có gì khó khăn để hiểu nó Nắm ý tưởng đó, và biết vài nguyên tắc để biến đổi tích phân, là có thể coi là hiểu tích phân, không cần phải học hàng trăm công thức tính các tích phân rắm rối loằng ngoằng (như có số sách cho học sinh phổ thông Việt Nam) Những công thức quá phức tạp đó dùng, và lúc nào cần dùng có thể tra 63 (64) Tạp chí Epsilon, Số 03, 06/2015 cứu, phải học chúng thì đúng là dễ có cảm giác học phải cái vô dụng Khi mà không nắm ý nghĩa việc lấy tích phân, thì việc tính tính phân các phân thức là cái máy, nhớ đống các công thức tính tích phân hoàn toàn là phí thời gian vô ích Đấy chính là điều không may mà nhiều người gặp phải: học phép tính tính phân là thứ “thánh bảo thì nó phải vậy”, giáo điều mà không dùng vào đâu Trong đó, ngành tài chính trên giới dùng tích phân “như cơm bữa” Các mô hình tài chính đại dùng toán đại, không là tính tích phân theo nghĩa thông thường nhiều người biết, mà là còn tính các tích phân ngẫu nhiên, là thứ toán học phát triển từ kỷ 20 Chính vì mà nhiều người gốc toán trở thành các “át chủ bài” thị trường tài chính, và chương trình cao học tài chính các nơi có toán khá nặng Có sinh viên Việt Nam sau tốt nghiệp xuất sắc các trường kinh tế hay tài chính, học bổng sang Pháp học cao học, bị “gẫy cầu” không theo được, phần chính vì không thể nhai phép tính tích phân ngẫu nhiên này, không chuẩn bị tốt kiến thức toán Tự học là tốt có thầy tốt Ở Việt Nam có tình trạng học sinh phải học quá nhiều, từ sáng đến đêm, hết học chính thức trên lớp lại học thêm Việc đến lớp quá nhiều phản tác dụng: nó có nguy làm cho trẻ trở nên mụ mẫm, thụ động, không có thời gian để tự suy nghĩ và tiêu hóa kiến thức, và không có thời gian cho các hoạt động khác ngủ, vui chơi, thể thao, âm nhạc, học làm việc nhà, v.v quan trọng cho phát triển Một phần chính vì để tránh vấn nạn học thêm này mà nhiều người muốn cho vào trường quốc tế hay “tị nạn giáo dục” Ở thái cực khác, thay vì xu hướng “nghe giảng quá nhiều, tự học quá ít” là xu hướng “để học sinh tự học là chính” mà số người muốn đưa cho cải cách giáo dục Theo xu hướng này thì giáo viên không còn vai trò giảng bài nữa, mà còn vai trò “hướng dẫn học sinh tự sáng tạo khám phá tìm 64 (65) Tạp chí Epsilon, Số 03, 06/2015 các kiến thức” Xu hướng này tương đối Việt Nam đã xâm nhập vào các chương trình cải cách giáo dục các nước tiên tiến trên giới Anh, Pháp từ nửa kỷ nay, dựa trên chủ thuyết “constructivisme” (“tự xây dựng kiến thức”) Jean Piaget, người “làm mưa làm gió” giáo dục Theo chủ thuyết “constructivisme” Piaget, thì học trò “tự xây dựng” các kiến thức mình, thầy nói chung không giảng kiến thức mà gợi ý cách tìm Chủ thuyết này, cùng với câu nghe bùi tai “lấy học trò làm trung tâm”, và khoác áo “khoa học, đổi mới”, đã nhanh chóng làm “mủi lòng” các quan chức giáo dục và các chuyên gia giáo dục nhiều nước Tuy nhiên, kết nó đem lại hoàn toàn trái ngược với mong đợi: đầu tư cho giáo dục nhiều lên trình độ học sinh giảm Ví dụ, Pháp, theo báo cáo Viện Hàn lâm Khoa học năm 2004 nhà bác học lớn ký tên (xem đây), 30 năm kể từ thời điểm bắt đầu cải cách giáo dục Pháp theo hướng “constructivisme” vào năm 1970, chương trình môn toán đã bị thụt 1,5 năm, tức là tính trung bình thì học sinh học đến lớp 12 ngày còn trình độ môn toán học sinh học lớp 10 thời năm 1970! Trong đó thì phát triển khoa học và công nghệ ngày càng đòi hỏi nhiều hiểu biết toán Nhà toán học tiếng Laurent Lafforgue cùng các tác giả khác có viết sách bi kịch giáo dục Pháp vào năm 2007: Laurent Lafforgue, Liliane Lurcçat et Collectif, La débâcle de l’école: une tragédie incomprise, 09/2007 (Sự “đổ vỡ” trường học: bi kịch không thấu hiểu) Một các nguyên nhân chủ chốt mà Lafforgue đưa để giải thích tình trạng suy sút giáo dục Pháp chính là: chủ thuyết “constructivisme” Jean Piaget các nhà chức trách ép sử dụng đã phá hoại hệ thống giáo dục Những người theo “constructivisme” quá chú trọng khía cạnh “tìm tòi sáng tạo” mà coi nhẹ khía cạnh “truyền đạt, luyện tập, tiếp thu cách bắt chước làm theo”, dẫn đến hậu là học sinh bị hổng kiến thức, thiếu tảng, và kiến thức đơn giản biến thành phức tạp 65 (66) Tạp chí Epsilon, Số 03, 06/2015 Không môn toán, mà các môn học khác Pháp hứng chịu hậu nghiêm chủ thuyết Piaget, người mà thời nhiều nơi tung hô là nhà cải cách giáo dục lớn giới Ví dụ, môn tiếng Pháp, thay vì dạy chia động từ ngày ưa, với chủ thuyết “constructivisme” người ta bắt học sinh “quan sát thay đổi dạng động từ” Hệ quả: tỷ lệ khá lớn học sinh Pháp đến vào đại học không biết chia động từ cho đúng Trong môn lịch sử, kiến thức lịch sử trang bị cho học sinh thì hạn chế, lại đòi hỏi học sinh bình luận các tài liệu y là các học sinh đó là các nhà sử học Kết là các “bình luận tư do” đó thực là các câu giáo điều đã viết trước (bởi học sinh có biết gì đâu để mà bình luận) Môn lịch sử dạy hời hợt đến mức học sinh lẫn lộn thứ tự thời gian (chronology) các kiện, kể các học sinh “khá” PTTH không biết các hoàng đế Napoleon và Louis XIV sinh trước sinh sau Ở các nước khác chịu ảnh hưởng Piaget, tình hình tồi tương tự Một nghiên cứu thống kê Canada (xem đây) cho thấy các phương pháp giảng dạy khác thì phương pháp theo “constructivisme” là phương pháp luôn cho kết tồi tệ Ở Thuỵ Sĩ, quê hương Piaget, người ta phải kêu trời rằng, giáo dục phổ thông Thuỵ Sĩ trước năm 1970 coi là mẫu mực thế, mà từ bị nhiễm “constructivisme” đã trở nên suy sút nặng Ở Việt Nam, có số người muốn cải cách giáo dục theo hướng “constructivisme” Piaget, coi nó là “kinh thánh”, tô điểm thêm cho nó thành lý thuyết với cái tên kêu là “công nghệ giáo dục”, thì tương tự là cái mà các nước khác người ta đã trải qua và đã và phải hứng chịu hậu Các bậc phụ huynh và người làm ngành giáo dục nên đề phòng chuyện này, không phải cái gì khoác áo “khoa học, công nghệ” là khoa học, công nghệ, kể giáo dục Vì học theo kiểu “tự xây dựng kiến thức” lại chậm nhiều so với có nghe thầy giảng? Thực đây là điều mà từ xưa người ta đã biết, có điều số học thuyết “lang băm” đã làm nhiều người quên điều này Người Việt Nam có câu “Không thầy đố mày làm nên” Người 66 (67) Tạp chí Epsilon, Số 03, 06/2015 Trung Quốc có câu “Nghe thầy tự mò tháng” Theo ước tính, trung bình học có thầy giảng giải nhanh gấp lần là tự học Đặc biệt là các học sinh có học lực trung bình, việc nghe giải thích kiến thức cách rõ ràng là cần thiết, có tỷ lệ nhỏ các học sinh thông minh đặc biệt là có thể dễ dàng tự tìm các qui luật Có người lầm tưởng rằng, kiến thức đã có sẵn sách hết rồi, thì cần gì thầy giảng cho nữa, đọc là xong Kiến thức sách không có nghĩa là nó có thể nhảy vào đầu mình cách dễ dàng Nếu không có người hướng dẫn, thì hoàn toàn có thể đọc mà không hiểu, là tưởng mình đã hiểu thực chưa hiểu gì, dẫn đến tẩu hoả nhập ma Tệ nữa, thì có thể đọc phải sách nhảm nhí, sách viết sai, v.v Quá trình tự tìm kiến thức, tìm chân lý khó khăn lâu dài là quá trình tiếp thu lại từ người đã nắm nó Những kiến thức mà học sinh học bây là thứ mà các nhà bác học kỷ trước phải đời người để tìm Không thể bắt học sinh lại đường khám phá đó, vì quá nhiều thời gian Và tất nhiên lãng phí quá nhiều thời gian tâm trí vào việc “phát minh lại cái bánh xe”, học sinh không còn đủ thời gian để tiếp cận các kiến thức cần thiết khác Điều trên không có nghĩa là học sinh không nên nghiên cứu và sáng tạo Nghiên cứu và sáng tạo là cần thiết, phải dựa trên tảng sở đã có không thể rỗng ruột mà nghiên cứu sáng tạo cái hay ho Để học nghiên cứu sáng tạo, thì người thầy tốt cho việc đó chính là người đã nghiên cứu và sáng tạo: các nhà khoa học, nhà sáng chế, nghệ sĩ, v.v (chứ không phải là người dạy nghiên cứu mà thân chưa có công trình nghiên cứu quan trọng nào) Cũng chính mà để học văn và học tiếng, thì học sinh nên đọc các bài thơ, mẩu truyện hay các nhà văn để nhờ đó mà thích học đọc, thấy cái hay cái đẹp sáng tạo ngôn ngữ không phải từ lớp đã học các khái niệm ngôn ngữ phức tạp trên các ví dụ nhạt nhẽo vô nghĩa “Constructivisme” là kiểu “tự học là chính có thầy đứng 67 (68) Tạp chí Epsilon, Số 03, 06/2015 bên cạnh” Còn kiểu tự học “oách hơn”, là tự học không cần thầy Ở đây tôi không nói đến việc ôn bài hay làm bài tập nhà, vì đó là lúc tự học theo chương trình có thầy hướng dẫn Tôi muốn nói đến việc tự học mà không có thầy hướng dẫn, thì kết sao? Có vị giáo sư Việt Nam tự hào việc mình “tự học thành tài”, viết sách dày ngàn trang việc tự học, với ví dụ chính là thân vị Tôi không nghi ngờ gì thông minh vị giáo sư này, và ngoài vị giáo sư này còn là nhà quản lý khá thành công Chỉ có điều đáng tiếc rằng, vị này hoàn toàn ảo tưởng mình khoa học: vị tự coi mình là thiên tài khoa học, trăm óc vĩ đại giới, sánh ngang tầm với các nhân vật lừng danh toàn cầu, công trình vị chẳng trên giới quan tâm trích dẫn Đấy chính là ví dụ rủi ro việc tự học mà không có người hướng dẫn: dễ bị “đâm vào ngõ cụt”, dễ bị hoang tưởng mình Bản thân tôi tự học nhiều thứ, với các mức độ thành công hay thất bại khác nhau: từ học bơi, học thiên văn, tiếng Tàu, tiếng Anh, v.v chủ yếu là tự học, không có thầy Khi sang Pháp làm việc, tôi chẳng nói câu tiếng Pháp nào, và từ đó đến chẳng học tiếng Pháp nào có người dạy, toàn tự học Cả luận án tiến sĩ tôi là tự làm, không có người hướng dẫn Nói không phải để khoe, mà để làm ví dụ cho thấy khả tự học người không nhỏ Càng nhiều tuổi hay càng học lên cao chúng ta càng cần đến khả tự học, vì điều kiện để học có thầy có lớp càng ít đi, nhu cầu và niềm vui học tập tuổi nào có Nói không có nghĩa là tự học thì tốt là học có thầy hướng dẫn Tôi đến lúc dạy học cho sinh viên nói sai tiếng Pháp Nếu có điều kiện thời gian và tiền bạc để học tử tế thì là tiếng Pháp tôi đã tốt nhiều không “ngọng” Vậy, không có thầy, thì ta có thể làm nào để tự học cho hiệu quả? Sau đây là số biên pháp tốt mà tôi biết: Tìm thầy xa, trên mạng 68 (69) Tạp chí Epsilon, Số 03, 06/2015 Ngày nay, thời đại internet, ngày càng có nhiều bài giảng hay đủ các môn, người thầy giỏi nhất, có thể tìm đến mà xem Đừng ngại tiếp cận trao đổi qua thư từ với thầy xa có thể bảo cho mình Các thầy giỏi thường là bận, mình tỏ thành tâm thì có lúc dành chút thời gian cho mình Đối với số môn học, có thể tìm thầy dạy trực tiếp qua mạng với giá phải Ví dụ, có thể học tiếng Tây Ban Nha thầy (có tiếng mẹ đẻ là Tây Ban Nha) trò qua skype với giá 10$/tiếng Tìm bạn thay thầy Tiếng Việt có câu “học thầy không tầy học bạn” Bạn bè, đồng nghiệp tương trợ cho nhiều chuyện học Có bạn giải thích lại dễ hiểu thầy giải thích Có thể kết bạn trên internet cho việc cùng học cái gì đó Việc đó càng ngày càng trở nên dễ dàng Trong việc học tiếng, thì kết bạn, nói chuyện với người xứ là phương pháp hiệu Nhúng mình môi trường thuận lợi Học cái gì, mà xung quanh mình có nhiều thứ liên quan cái đó, thì vào hơn: sách vở, phim ảnh, đồ thí nghiệm, v.v tốt Không sợ sai “Ai không làm gì thì không sai” Cứ làm đi, đừng sợ sai, trừ cái sai đó là cái sai gây tai hoạ cho người khác Tất nhiên, cần phải quan sát kiểm tra để biết mình sai đâu mà còn sửa Làm từ từ, vừa làm vừa quan sát cảm nhận và suy nghĩ Khi học cái gì mới, không nên vội vàng đòi kết nhanh Nên làm từ từ (không có nghĩa là lười ít làm, mà là làm với tốc độ chậm), vừa làm vừa quan sát nghĩ ngợi để cảm nhận xem mình làm hư có đúng không, sai đâu, v.v Ví dụ tự học bơi: có thể quan sát người xung quanh bơi nào, mình bơi thì làm các động tác thật từ từ, thử nhiều kiểu khác nhau, để cảm nhận và suy nghĩ xem động tác nào 69 (70) Tạp chí Epsilon, Số 03, 06/2015 làm nào thì bơi được, các cử động phải ăn khớp với nào, v.v Sau đã làm chậm mà đúng thì làm nhanh dần lên 70 (71) THƯ CỦA KAPITSA VỀ KHOA HỌC ĐÀM THANH SƠN (Đại học Chicago, Hoa Kỳ) Lời giới thiệu GS Đàm Thanh Sơn là GS Vật lý lý thuyết trường Đại học Chicago, Hoa Kỳ Ông đoạt huy chương vàng toán quốc tế với số điểm tuyệt đối 42/42 15 tuổi Đàm Thanh Sơn quan tâm đến việc truyền bá niềm đam mê khoa học cho hệ trẻ Blog ông https://damtson.wordpress.com luôn cập nhật bài viết hay, bài toán (vật lý) thú vị dạng thường thức phổ thông hay gợi mở cho các nhà khoa học tương lai Được cho phép GS Đàm Thanh Sơn, chúng tôi chọn đăng loạt thư Pyotr Kapitsa khoa học ông chọn dịch và giới thiệu trên blog mình Nguồn các thư: Капица П.Л., Письма о науке, 1930-1980 – М.: Моск рабочий, 1989 Ban Biên tập 71 (72) Tạp chí Epsilon, Số 03, 06/2015 Tóm tắt nội dung Pyotr Kapitsa (trái) và Nikolay Semyonov, hai nhà vật lý đạt giải thưởng Nobel vật lý (tranh chân dung Boris Kustodiev, 1921) Pyotr Kapitsa (1894-1984) là nhà vật lý tiếng Nga Ông làm việc hướng dẫn Rutherford Cambridge, Anh, nhiều năm Năm 1933 ông trở thành giám đốc đầu tiên phòng thí nghiệm Mond Năm 1934 ông bị giữ lại Nga chuyến Nga Nhà nước Nga Xôviết mua lại toàn thiết bị phòng thí nghiệm ông Anh và xây cho ông Viện nghiên cứu mới, Viện các vấn đề vật lý, đó ông là giám đốc đầu tiên Năm 1937 ông phát tính siêu chảy hêli lỏng, công trình giải thưởng Nobel năm 1978 Trong năm 1937-1938 đen tối ông là người cứu Fok và Landau khỏi tù Trong chiến tranh giới thứ ông phát và đưa vào sản xuất loại máy để chế tạo ôxy lỏng, và trao danh hiệu Anh hùng lao động Liên Xô năm 1945 Do mâu thuẫn với Beria ông bị tất các chức vụ năm 1946 Ông phục hồi chức giám đốc Viện các vấn đề Vật lý năm 1955 72 (73) Tạp chí Epsilon, Số 03, 06/2015 Ông là người đầu tiên đưa ý tưởng thành lập tạp chí chuyên khoa học cho hệ trẻ, sau này chính là tạp chí Kvant Rất nhiều thư ông đã công bố Nhiều thư liên quan đến khoa học, giáo dục đáng đọc và còn giữ tính thời Tôi dịch số thư ông và đăng trên blog này Trích thư Kapitsa gửi vợ, A.A.Kapitsa 13 tháng 12 năm 1935, Moskva Hôm qua anh đánh cờ với Aleksei Nikolaevich Bakh Cụ dễ mến, anh không đồng ý với cụ điểm Anh nói với cụ là tình trạng khoa học nước ta xấu, thì cụ bảo: "Đúng thế, làm gì được, bây có nhiều thứ quan trọng là khoa học " Đây là ví dụ điển hình nhà khoa học tự nguyện đẩy mình xuống hạng ưu tiên thứ nhì, chí thứ ba Anh cho phải coi khoa học là việc quan trọng và lớn lao, và cái inferiority complex này (tiếng Anh nguyên – mặc cảm tự ti, tự cho mình là không quan trọng) giết chết khoa học nước ta Các nhà khoa học phải cố gắng chiếm vị trí hàng đầu việc phát triển văn hóa nước nhà và không lẩm bẩm "ở nước ta có thứ quan trọng hơn" Đánh giá cái gì là quan trọng nhất, và cần phải chú ý đến khoa học kỹ thuật đến mức nào là công việc các nhà lãnh đạo Còn công việc các nhà khoa học là tự tìm chỗ đứng mình đất nước và chế độ mới, và không đợi người khác cho mình phải làm gì Cái thái độ khó hiểu và xa lạ anh Khi anh nói chuyện với nhiều nhà khoa học, anh ngạc nhiên họ tuyên bố "Cậu nhiều thì cậu làm gì chả dễ dàng " Và Họ làm là bắt đầu nghiệp, hội ban đầu anh và họ không giống Họ làm là gì anh có là rơi từ trên trời xuống, không phải là anh đã bỏ bao nhiêu công sức, bao nhiêu nơron thần kinh đạt Về khía cạnh này A.N Bakh (1857-1946) là nhà hóa sinh lớn Nga, bạn thân Kapitsa 73 (74) Tạp chí Epsilon, Số 03, 06/2015 người thật hèn hạ, họ cho đời không công bằng, xung quanh có lỗi trừ họ Nhưng đấu tranh làm gì, không phải chính là để ta lợi dụng hoàn cảnh sẵn có quanh ta cho việc phát triển tài mình và tạo điều kiện làm việc cho mình? Nếu chấp nhận quan điểm Bakh và Co thì không xa Bức thư cứu Landau tháng năm 1939 Đồng chí Molotov, Trong thời gian gần đây, nghiên cứu hêli lỏng gần độ không tuyệt đối, tôi đã tìm loạt các tượng có thể làm sáng tỏ lĩnh vực bí ẩn vật lý đại Trong tháng tới tôi định công bố phần công trình đó Nhưng để làm việc này, tôi cần nhà lý thuyết giúp đỡ Ở Liên Xô, người hoàn toàn làm chủ lĩnh vực lý thuyết mà tôi cần là Landau, hiềm nỗi là đã bị bắt từ năm Tôi hy vọng là thả, vì tôi phải nói thẳng tôi không thể tin Landau lại là tên tội phạm quốc gia Tôi không thể tin điều đó vì nhà khoa học trẻ, tài và chói lọi Landau, 30 tuổi đã tiếng châu Âu, ngoài lại háo danh, và có đầy chiến tích khoa học đến mức đó thì không thể có sức lực, cảm hứng và thời gian cho công việc khác Đúng là Landau có cái miệng độc, và vì lạm dụng nó anh đã tạo nhiều kẻ thù luôn sẵn sàng gây khó dễ cho mình Nhưng dù có tính cách khá xấu mà tôi buộc phải lưu ý đến, tôi chưa thấy làm điều gì khuất tất Tất nhiên, nói tất điều đó, tôi can thiệp vào việc không phải mình, vì lĩnh vực này là thẩm quyền Landau bị bắt ngày 28/4/1938 Ngay hôm đó Kapitsa viết thư cho Stalin, không có tác dụng Sau thư trên đây, ngày 26/4/1939 Kapitsa mời đến NKVD để viết đơn bảo lãnh cho Landau tù Các tượng nhắc đến đầu thư trên là các tượng liên quan đến tính siêu chảy Landau xây dựng lý thuyết siêu chảy hêli hai bài báo năm 1941 và 1947 74 (75) Tạp chí Epsilon, Số 03, 06/2015 NKVD (Bộ Nội vụ) Nhưng dù tôi nghĩ là tôi phải nêu lên điểm bất thường sau: Landau đã ngồi tù năm, mà việc điều tra chưa kết thúc, thời gian điều tra lâu cách không bình thường Tôi, với tư cách giám đốc quan anh ta, hoàn toàn không biết bị cáo buộc tội gì Quan trọng là đã năm nay, không biết vì lý gì mà khoa học, Xô-viết lẫn giới, không có cái đầu Landau Landau ốm yếu, bị hại cách không cần thiết thì đáng xấu hổ cho người Xô-viết chúng ta Vì tôi muốn gửi tới đồng chí yêu cầu sau đây: Có thể đề nghị NKVD đặc biệt chú ý xúc tiến vụ Landau không; Nếu không được, thì liệu có thể sử dụng cái đầu Landau cho công việc khoa học lúc bị giam Butyrki [trại giam cách ly để điều tra] không Tôi nghe nói các kỹ sư đối xử P Kapitsa Về việc trả lương cho người làm khoa học Sau trở thành giám đốc Viện các vấn đề Vật lý, Kapitsa đã viết nhiều thư cho lãnh đạo Xô-viết than phiền chế tài chính quá cồng kềnh và cứng nhắc Nga Trong thư Kapitsa có viết là công việc kế toán mà thư ký ông Anh làm chưa đến tiếng ngày thì Nga phải người làm hết Trong đoạn sau ông đề nghị áp dụng chế trả lương Viện ông giống Anh Trích thư Kapitsa gửi cho V.I.Mezhlauk, phó chủ tịch Hội đồng Dân ủy Liên Xô, 26/10/1936 75 (76) Tạp chí Epsilon, Số 03, 06/2015 Nguyên tắc để đánh giá lao động Liên Xô quy định rõ ràng và chính xác Hiến pháp Stalin hưởng theo lao động và lực Nguyên tắc này áp dụng quán phong trào Stakhanov và đưa đến thành chói lọi Nó cho cá nhân khoảng không rộng lớn để phát triển; người công nhân xuất sắc, biết tổ chức lao động và vượt mức khoán nhiều lần, trả công theo lao động đúng lực người đó thể công việc Cách trả lương theo thang lương nhà nước hành mâu thuẫn với nguyên tắc này Các quan hành chính quan liêu thường hay tuyển người thích làm việc yên ổn, không thích có sáng kiến, chí trái lại cố gắng làm càng chính xác thị từ trên càng tốt Với người này, hệ thống trả lương theo thang nhà nước có lẽ là hoàn toàn bình thường Hệ thống này áp dụng ta trước đây và các nước Tây Âu và dẫn đến tệ quan liêu [Guy de] Maupassant mô tả sống động mà chưa nước nào tránh Đáng tiếc là chưa tìm cách nào để áp dụng phương pháp Stakhanov cho các nhân viên hành chính văn phòng, tức là vào suất lao động để tăng lương Áp dụng thang bậc lương nhà nước cứng nhắc các viện khoa học là hoàn toàn sai Chúng tôi không chấp nhận kiểu làm việc công chức: công việc chúng tôi mở không gian rộng lớn cho việc hoàn thiện lao động, phát triển cá nhân và phát huy lực riêng người Hệ thống trả lương cứng nhắc theo thang lương nhà nước hoàn toàn không phù hợp đây Nhưng áp dụng hệ thống trả lương kiểu Stakhanov các quan nghiên cứu khoa học không thể Trở ngại chính là chỗ công việc chúng tôi không phải là sản xuất, nên không đưa chuẩn mực cứng để đánh giá công việc theo sản phẩm Vì cần tìm phương pháp khác Đó là điều Phương pháp đáng tin cậy để đánh giá công việc viện khoa học là dựa vào ý kiến ban giám đốc viện, ý kiến mà theo tôi là có thẩm quyền để định mức lương, Hiến pháp Liên Xô năm 1936 Đúng phải là: làm theo lực, hưởng theo lao động 76 (77) Tạp chí Epsilon, Số 03, 06/2015 vì ban giám đốc là người giao nhiệm vụ, biết mức độ khó khăn nhiệm vụ, và đạo công việc cho nên có đánh giá công việc tốt Không có lối thoát nào khác Hệ thống thang lương nhà nước không thích hợp, và chứng trước mắt là tất các quan khoa học tìm cách để lách khỏi hệ thống này Các mức lương xào xáo, thổi phồng kiểu toán tổ hợp phức tạp để làm rối và trốn khỏi mắt tra quan tài chính Phương pháp này chào đón công cụ tự vệ tốt để khỏi phải tuân theo kỷ luật ngân sách nghiêm ngặt, cái kỷ luật ngược lại các nhu cầu khoa học Kiểu tự lừa dối làm tôi vô cùng ghê tởm Về việc đối xử với các nhà khoa học bất đồng quan điểm Trích thư gửi Yuri V Andropov, chủ tịch Uỷ ban An ninh Quốc gia (KGB) Ngày 11 tháng 11 năm 1980, Moskva Yuri Vladimirovich kính mến, Cũng nhiều nhà khoa học khác, tôi vô cùng lo lắng tình cảnh và số phận hai nhà vật lý lớn nước ta — A D Sakharov và Yu F Orlov Tình hình có thể mô tả đơn giản sau: Sakharov và Orlov đã đem lại cống hiến to lớn hoạt động khoa học, hoạt động họ người bất đồng quan điểm bị coi là có hại Hiện họ bị đặt vào hoàn cảnh không thể có hoạt động gì Tóm lại, họ không thể mang lại lợi ích tác hại Thử hỏi làm có lợi cho đất nước hay không? Trong thư, này tôi thử phân tích thật khách quan câu hỏi đã nêu Nếu hỏi các nhà khoa học, thì họ trả lời dứt khoát là việc nhà khoa học lớn Sakharov và Orlov bị tước khả nghiên cứu khoa học bình thường đem lại thiệt hại cho loài người Nếu hỏi các nhà hoạt động xã hội, 77 (78) Tạp chí Epsilon, Số 03, 06/2015 người thường ít biết đến hoạt động khoa học các nhà bác học này, thì họ đưa nhận định ngược lại tình trạng Trong lịch sử văn hoá loài người, từ thời Socrat đến nay, có không ít trường hợp người ta kịch liệt chống người bất đồng quan điểm Để giải khách quan vấn đề đặt tất nhiên cần cân nhắc nó bối cảnh xã hội cụ thể đất nước Trong hoàn cảnh chúng ta xây dựng chế độ xã hội mới, tôi nghĩ đúng đắn là vào quan điểm Lênin, vì đó là quan điểm toàn diện người không là nhà tư tưởng tiếng, nhà khoa học, mà còn là nhà hoạt động xã hội lớn Cách đối xử Lênin với các nhà khoa học trường hợp tương tự nhiều người biết đến Điều này thể rõ ràng và đầy đủ qua cách Lênin đối xử với I P Pavlov Sau cách mạng, biết bất đồng quan điểm Pavlov, không nước ta mà nước ngoài Ông cố tình phơi bày công khai thái độ không tán thành chủ nghĩa xã hội mình Ông đã phê phán, chí chửi lãnh đạo không e dè lời phát biểu gay gắt; ông làm dấu thánh qua nhà thờ, đeo các huy chương Nga Hoàng trao tặng, huy chương mà trước cách mạng ông không thèm để ý đến, v.v Nhưng Lênin không mảy may để ý đến biểu bất đồng quan điểm Pavlov Đối với Lênin, Pavlov là nhà khoa học lớn, và Lênin đã làm tất gì có thể để đảm bảo điều kiện làm việc tốt cho công việc khoa học Pavlov Ví dụ, người biết, các thí nghiệm quan trọng phản xạ có điều kiện Pavlov tiến hành trên chó Vào năm 1920, thực phẩm Petrograd thiếu trầm trọng, theo thị Lênin, thức ăn để nuôi các chó thí nghiệm Pavlov cung cấp bình thường Tôi còn biết hàng loạt trường hợp khác mà Lênin quan tâm đặc biệt tới các nhà khoa học Điều này biết qua thư Lênin gửi K A Timiryazev, A A Bogdanov, Carl Steinmetz v.v Cần phải đối xử với người bất đồng chính kiến cách trân trọng và thận trọng Lênin đã làm Sự bất đồng quan điểm liên quan chặt chẽ đến hoạt động sáng tạo 78 (79) Tạp chí Epsilon, Số 03, 06/2015 người, mà hoạt động sáng tạo lĩnh vực văn hoá lại đảm bảo tiến nhân loại Có thể nhận dễ dàng là nguồn gốc tất các lĩnh vực hoạt động sáng tạo người chính là bất mãn với trạng Ví dụ, nhà khoa học không thoả mãn với trình độ nhận thức lĩnh vực khoa học mà quan tâm, và tìm phương pháp nghiên cứu Nhà văn không hài lòng với mối quan hệ người với người xã hội tại, và cố gắng dùng nghệ thuật để tác động lên cấu trúc xã hội và đến hành vi người Người kỹ sư không thoả mãn với giải pháp kỹ thuật có và tìm dạng kết cấu để giải vấn đề Nhà hoạt động xã hội không lòng với các văn luật và ước lệ sử dụng để xây dựng nhà nước, và tìm hình thức để vận hành xã hội, v.v Tóm lại, để xuất ước muốn sáng tạo, thì là phải có bất mãn với trạng, nghĩa là cần trở thành người bất đồng ý kiến Điều này đúng lĩnh vực hoạt động người Tất nhiên, người bất mãn thì nhiều, để có thể thể mình cách có hiệu hoạt động sáng tạo thì còn cần có tài Cuộc sống cho thấy là có ít tài lớn, và vì cần tôn trọng và nâng niu bảo vệ họ Kể có lãnh đạo tốt thì điều này khó thực Khả sáng tạo lớn còn đòi hỏi tính cách mạnh, và điều đó dẫn đến cách thể bất mãn gay gắt, vì người tài thường là "ngang" Ví dụ, tượng này hay thấy các nhà văn lớn, vì họ thích tranh cãi và thích phản kháng Trên thực tế, hoạt động sáng tạo thường không tiếp đón nhiệt tình lắm, vì đa số người là bảo thủ và ưa sống phẳng lặng Kết là biện chứng phát triển văn hoá loài người bị kẹt mâu thuẫn bảo thủ và bất đồng quan điểm, và điều này xảy thời đại và lĩnh vực hoạt động văn hoá người Công việc sáng tạo lớn thường mang tính chất tư tưởng và không bị lung lay biện pháp hành chính hay bạo lực Lênin đã rõ phải xử lý nào trường hợp qua cách ông đối xử với Pavlov Lịch sử chứng minh Lênin đúng đã lờ biểu bất đồng 79 (80) Tạp chí Epsilon, Số 03, 06/2015 quan điểm gay gắt Pavlov các vấn đề xã hội, trân trọng cá nhân hoạt động khoa học Pavlov Điều đó dẫn đến kết là thời Xôviết, Pavlov với tư cách là nhà sinh lý học đã không gián đoạn nghiên cứu xuất sắc mình phản xạ có điều kiện, nghiên cứu còn vai trò dẫn dắt khoa học giới Còn vấn đề liên quan đến xã hội thì tất gì Pavlov nói đã bị lãng quên từ lâu Không biết bây chúng ta lại quên di huấn Lênin cách đối xử với các nhà khoa học 80 (81) NHỮNG TRIẾT LÝ SỐNG CỦA EINSTEIN BAN BIÊN TẬP EPSILON Lời giới thiệu Albert Einstein không danh với thuyết tương đối mà ông còn để lại cho nhân loại triết lý sống vô cùng thâm thúy Tất người có thể học điều gì đó từ triết lý này, cho dù họ là học sinh, sinh viên, phụ huynh, thầy cô giáo, nhà khoa học hay doanh nhân Chúng tôi trích đăng số câu nói Einstein và các bình luận các báo Tuổi trẻ và Doanh nhân Sài Gòn giới thiệu Một số câu nói khác Einstein Nguyễn Tiến Dũng trích dẫn bài viết tác giả này trên cùng số báo này 81 (82) Tạp chí Epsilon, Số 03, 06/2015 Theo đuổi trí tò mò "Tôi chẳng có tài đặc biệt nào Tôi ham thích tò mò" Tò mò giúp cung cấp trí tưởng tượng cho chúng ta Khi chúng ta đặt câu hỏi thứ khác, chúng ta có thể tìm thông tin quan trọng giúp giải vấn đề, mở cánh cửa và hình thành các mối liên kết Khi chúng ta đặt câu hỏi chính thân, chúng ta có thể đánh thức niềm tin, tiết lộ ham muốn sâu thẳm mình và tạo nên thay đổi tích cực Những câu hỏi chưa có lời giải nào xuất đầu bạn ? Sự kiên trì là vô giá "Không phải tôi quá thông minh, là tôi dành thời gian lâu để nghiền ngẫm các vấn đề." Nếu bạn có giấc mơ, bạn phải đối mặt với trở ngại; việc kiên trì với nó, Einstein nói, có thể mang ý nghĩa là khác biệt thất bại và thành công Một số cách để bắt đầu luyện tập kiên trì là cam kết giấc mơ bạn, giữ thái độ tích cực, luôn tập trung vào gì bạn muốn ngày và trở lại từ nghịch cảnh Tập trung vào "Người đàn ông nào lái xe an toàn hôn cô gái xinh đẹp thì đơn giản là người đó không trao cho nụ hôn tập trung xứng đáng" Einstein đã sử dụng ví dụ thật tuyệt vời để minh họa cho tầm quan trọng việc tập trung vào Chúng ta có thể bỏ lỡ niềm vui việc quá bận tâm tới quá khư và/hoặc tương lai Nhắc nhở thân ngày mang đến cho chúng ta bình an và niềm vui nhiều biết trân quý sống 82 (83) Tạp chí Epsilon, Số 03, 06/2015 Trí tưởng tượng là quyền "Trí tưởng tượng là tất Trí tưởng tượng là xem thử địa điểm tham quan đến sống Trí tưởng tượng còn quan trọng kiến thức" Với ý tưởng, đế chế có thể hình thành Lấy ví dụ, Walt Disney, chuyên gia đích thực việc tưởng tượng Ông lấy cảm hứng cho nhân vật chuột Mickey từ chuột thú cưng đã già nông trại ông Chú chuột trắng đen đó trở thành huyền thoại hoạt hình Trí tưởng tượng mở cánh cửa đến với vương quốc khả Hãy mắc sai lầm "Một người chưa mắc sai lầm không làm điều lạ" Sai lầm là điều chắn xảy đặc biệt bạn theo đuổi điều gì đó đáng giá Chúng có thể gây thất vọng và khó khăn cho tự tin, thường cần thiết để kiểm tra cam kết thực chúng ta với mục tiêu cuối cùng Làm gì có điều tuyệt vời nào hoàn thành mà không gặp thất bại lúc đầu theo cách nào đó Thất bại thật là không bắt đầu không hoàn thành Sống cho khoảnh khắc "Tôi chẳng nghĩ tương lai, nó đến sớm thôi" Như họ nói, khoảnh khắc này là tất gì chúng ta thực nắm giữ, khái niệm khó nắm bắt Eckhart Tolle nói sách ông, Power of Now (Quyền Hiện tại), thành công người khoảnh khắc có thể thực đo bình an mà anh/ cô cảm nhận Bằng việc trở nên ý thức thời điểm, chúng ta có thể tập trung vào vấn đề quan trọng Bạn có thể chắn gì diễn Bạn có quyền lo lắng cho tương lai và xây dựng kế 83 (84) Tạp chí Epsilon, Số 03, 06/2015 hoạch, thứ không diễn bạn dự tính Vấn đề quan trọng là sống hết mình ngày hôm Làm thứ tốt khả và không phải lo lắng gì ngày mai Tạo giá trị "Nỗ lực không để thành công, đúng là để có giá trị" Bạn định nghĩa thành công nào? Làm gì để có sống thành công? Những câu hỏi này có thể là câu hay để bạn tự chất vấn thân Đó có thể là nuôi dạy đứa khỏe mạnh và hạnh phúc, có mối quan hệ ý nghĩa và trọn vẹn, chính xác các hội thoại , viết sách, yêu công việc, cảm thấy khỏe ngày – điều gì giành cho bạn, đó là nơi để đặt tiếp tục trọng tâm Những gì chúng ta tập trung vào phát triển thực Đừng lặp lặp lại "Điên rồ : làm cùng việc lặp lặp lại nhiều lần và mong đợi kết khác nhau" Nếu bạn không hạnh phúc lĩnh vực nào sống tài chính hay các mối quan hệ, hãy chọn làm việc khác vào ngày hôm sau ý tưởng này nhằm đánh thức thói quen Nếu bạn có công việc không trọn vẹn làm nản lòng mức độ nào đó, hãy suy nghĩ bạn có thể làm gì để thay đổi tình hình Đôi cái nhìn cùng vấn đề là tất gì cần thiết để nhận gì có thể xảy Bước đầu tiên là nhận bất mãn và sau đó việc hướng tới mục tiêu đó là hài lòng Kiến thức đến từ kinh nghiệm "Thông tin không phải là kiến thức Nguồn kiến thức là kinh nghiệm" 84 (85) Tạp chí Epsilon, Số 03, 06/2015 Kinh nghiệm thực tế tạo kiến thức tôn trọng và đánh giá cao người khác Chúng ta có thể đọc sách, nghe băng, và học kinh nghiệm chúng ta có sống có thể mang lại bài học tốt cho người khác Câu chuyện sống bạn chứa đựng nhiều kiến thức và người sẵn sàng lắng nghe vì đó là cách hấp dẫn và đáng tin cậy để tạo khác biệt với đó 10 Biết nguyên tắc và làm tốt "Bạn phải biết quy luật trò chơi Và sau đó bạn phải chơi tốt ai" Để trở thành chuyên gia lĩnh vực nào đó, cần tìm hiểu tất gì có thể vấn đề đó, nghiên cứu thành công người khác và đặt mục tiêu làm tốt họ Sự cam kết và niềm đam mê với nỗ lực càng mạnh mẽ thì tâm đến với thành công càng lớn 11 Hãy đơn giản thứ "Nếu bạn không thể giải thích cho đứa trẻ tuổi hiểu được, thì chính bạn không hiểu gì cả" Cố làm cho thứ phức tạp đồng nghĩa với việc bạn không hiểu chất vấn đề Hãy nhớ đến thầy cô giáo đã dạy bạn Họ đã giải thích tất trang sách đầy chữ cho bạn lời lẽ giản dị và dễ hiểu Bạn nên ghi nhớ điều này giao tiếp huấn luyện nhân viên 12 Hãy sáng tạo "Sáng tạo có tính lây lan, hãy truyền nó đi!" Hãy khơi nguồn cảm hứng cho người làm điều mà họ yêu thích Hãy sử dụng khả sáng tạo để tạo công việc và niềm vui Đã đến lúc bạn dành tâm trí cho các ý tưởng và tạo hiệu ứng domino đến người xung quanh 85 (86) Tạp chí Epsilon, Số 03, 06/2015 13 Dám mắc sai lầm "Cách để tránh sai lầm là đừng có ý tưởng mới" Sai lầm là thứ xảy ngày với Và thứ thực làm thay đổi giới, không phải là quy trình hoàn hảo mà là ý tưởng Do đó, học cách chấp nhận sai lầm và mạo hiểm với sáng tạo là điều cần thiết để tạo thay đổi tích cực tổ chức nào 14 Hãy luôn chăm "Bạn không thất bại bạn ngừng nỗ lực" Einstein dành đời để nghiên cứu các lý thuyết vật lý và nhiều nghiên cứu số đó không đến kết nào Chúng ta không thể chắn kết việc chúng ta làm, kiên trì chính là chìa khóa Tất gì chúng ta có thể làm là chăm theo đuổi mục tiêu Thất bại tồn ta dừng lại đó mà không cố gắng bước bước để đến thành công 15 Nghĩ khác "Tôi chưa khám phá điều gì cách tư hợp lý" Những thứ tuyệt vời đến luôn đến từ việc nghĩ "ngoài hộp" và làm việc khác thường Mọi người thường khó chịu với kẻ nghĩ khác, đó là lý vì có ít người kiệt xuất Khi gặp vấn đề nan giải, hãy cố gắng nghĩ theo hướng khác, biết đâu bạn có câu trả lời 16 Phát huy trí tưởng tượng "Tưởng tượng là dạng thức tối cao nghiên cứu" "Tôi có khả để vẽ thoải mái hoạ sĩ nhờ vào trí tưởng tượng mình Trí tưởng tượng quan trọng kiến thức Kiến thức thì có giới hạn Trí tưởng tượng bao trùm giới" 86 (87) Tạp chí Epsilon, Số 03, 06/2015 Sau tưởng tượng điều gì đó khác, bạn bắt đầu chia sẻ nó với người Sau đó, người có thể nhìn thấy giới mà bạn tưởng tượng và họ liên kết và hỗ trợ bạn Hãy dành thời gian để mơ mộng, tưởng tượng và nhớ chia sẻ ý tưởng bạn với người xung quanh 17 Làm điều không thể "Chỉ nỗ lực hết mình có thể đạt điều tưởng chừng không thể " Nếu bạn sẵn sàng đối mặt với khó khăn thì bạn tiến gần đến điều không thể "Điều không thể" là khái niệm tương đối Bạn ngạc nhiên dám vượt qua điều hợp lý mà người nghĩ 18 Tôn trọng người "Từ sống thường ngày, chúng ta biết sống - trước hết là cho người xung quanh, cho người luôn nở nụ cười và khiến ta hạnh phúc" "Cuộc đời chẳng đáng sống, trừ ta sống vì người khác" Hãy quan tâm đến người trước tiên Dành ít vài phút ngày để tập trung kết nối, trò chuyện với người khác Hãy cho họ biết bạn trân trọng họ Cảm ơn và gửi tặng họ lời khen mà họ xứng đáng nhận Điều này không giúp tâm trạng họ tốt mà tâm trạng bạn 19 Luôn sẵn sàng học hỏi "Học hỏi chính là kinh nghiệm Những thứ khác là thông tin" Học là hành trình không phải là việc đắm mình tất các thông tin Hãy "lặn tìm kho báu" và luôn ghi nhớ có số điều hữu ích cho bạn, còn số khác thì không Việc học không phải là thứ có thể phù hợp với tất người, hãy tìm cách riêng bạn 87 (88) Tạp chí Epsilon, Số 03, 06/2015 20 Làm điều đúng "Luôn làm điều đúng Việc này làm hài lòng số người và làm người còn lại ngạc nhiên" Khi bạn đứng ngã ba đường, hãy chọn việc đúng đắn mà làm Các lựa chọn khác có thể dễ dàng thực mang đến cho bạn nhiều tiền Nhưng bạn nghĩ lựa chọn còn lại khó khăn cho bạn nhiều hội tương lai, đơn giản đó là điều đúng đắn cần làm thì bạn hãy chọn nó Đôi đường khó khăn gập ghềnh tốt đường mòn nhiều người đã Hãy dành thời gian để theo dõi giá trị đạt và làm điều đúng đắn để giúp bạn trội đám đông 88 (89) LỜI GIẢI VÀ BÌNH LUẬN BÀI TOÁN TRONG ĐỀ THI CHỌN ĐỘI TUYỂN V IỆT N AM 2015 TRẦN NAM DŨNG (Đại học KHTN, Tp Hồ Chí Minh) Tóm tắt nội dung Trong số trước, bạn Trần Quang Hùng đã đưa lời giải cho hai bài toán hình học đề chọn đội tuyển Việt Nam năm 2015 cùng với phân tích bình luận thú vị liên quan Ở đây, số này, chúng tôi tiếp tục giới thiệu cùng bạn đọc lời giải cho bài toán còn lại đề thi Bài toán Gọi α là nghiệm dương phương trình x2 + x = Giả sử n là số nguyên dương và các số nguyên không âm c0 , c1 , c2 , , cn thỏa mãn đẳng thức c0 + c1 α + c2 α2 + · · · + cn αn = 2015 (∗) a) Chứng minh c0 + c1 + c2 + · · · + cn ≡ (mod 3) b) Tìm giá trị nhỏ tổng c0 + c1 + c2 + · · · + cn Lời giải a) Cách Ta chứng minh quy nạp theo n Với n = 0, ta có c0 = 2015, mệnh đề đúng Với n = 1, α là số vô tỷ nên đẳng thức c0 + c1 α = 2015 có thể xảy c1 = 0, c0 = 2015, mệnh đề đúng Giả sử mệnh đề đã đúng đến n ≥ Xét các số nguyên không âm c0 , c1 , c2 , , cn , cn+1 thỏa mãn đẳng thức c0 + c1 α + c2 α2 + · · · + cn αn + cn+1 αn+1 = 2015 Sử dụng đẳng thức α2 = − α, ta có cn+1 αn+1 = cn+1 αn−1 (5 − α) Suy c0 + c1 α + c2 α2 + · · · + (cn−1 + 5cn+1 )αn−1 + (cn − cn+1 )αn = 2015 89 (90) Tạp chí Epsilon, Số 03, 06/2015 Áp dụng giả thiết quy nạp, ta có c0 + c1 + c2 + · · · + (cn−1 + 5cn+1 ) + (cn − cn+1 ) ≡ (mod 3) Nhưng đây có nghĩa là c0 + c1 + · · · + cn−1 + cn + cn+1 ≡ (mod 3) Vậy mệnh đế đúng với n + Theo nguyên lý quy nạp toán học, ta có điều phải chứng minh Cách Xét đa thức P (x) = c0 + c1 x + c2 x2 + · · · + cn xn − 2015 thì P (α) = Ta chứng minh P (x) chia hết cho Q(x) = x2 + x − 5, tức là P (x) = Q(x) · S(x) với S(x) là đa thức với hệ số nguyên Thật vậy, giả sử P (x) = Q(x) · S(x) + Ax + B, với A, B nguyên Thay x = α vào ta Aα + B = Do α là số vô tỷ nên điều này có thể xảy A = B = Vậy P (x) = Q(x) · S(x) Thay x = vào ta c0 + c1 + c2 + · · · + cn − 2015 = −3 · S(1), suy c0 + c1 + c2 + · · · + cn ≡ (mod 3) b) Với số nguyên không âm (c0 , c1 , c2 , , cn ) thỏa mãn (∗), ta gọi c0 + c1 + c2 + · · · + cn là giá số đó Do tính thứ tự tốt tập các số tự nhiên, tồn số (c0 , c1 , c2 , , cn ) thỏa mãn (∗) với giá nhỏ Ta chứng minh nhận xét quan trọng sau: Nhận xét Nếu (c0 , c1 , c2 , , cn ) là có giá nhỏ thì ci < với i = 0, 1, 2, , n Chứng minh Ta chứng minh phản chứng Giả sử tồn i cho ci ≥ Khi đó dựa vào đẳng thức = α2 + α, ta có ci αi = (ci − 5)αi + (α2 + α)αi = (ci − 5)αi + αi+1 + αi+2 Như vậy, số (c0 , c1 , c2 , , ci − 5, ci+1 + 1, ci+2 + 1, , cn ) thỏa mãn (∗) và có giá c0 +c1 +· · ·+(ci −5)+(ci+1 +1)+(ci+2 +1)+· · ·+cn = c0 +c1 +· · ·+cn −3 nhỏ giá (c0 , c1 , , cn ) Điều này mâu thuẫn với cách chọn (c0 , c1 , , cn ) trên  90 (91) Tạp chí Epsilon, Số 03, 06/2015 Bây giờ, giống phần a) ta đặt: P (x) = c0 + c1 x + c2 x2 + · · · + cn xn − 2015 và Q(x) = x2 + x − thì theo a), P (x) = Q(x) · S(x) Đặt: S(x) = b0 + b1 x + b2 x2 + · · · + bn−2 xn−2 So sánh hệ số hai vế, ta c0 − 2015 = −5b0 , c1 = b0 − 5b1 , c2 = b0 + b1 − 5b2 , c3 = b1 + b2 − 5b3 , , cn = bn−2 Từ điều kiện ≤ c0 ≤ 4, ta suy c0 = và b0 = 403 Tiếp tục sang dòng thứ hai, ta tìm c1 = và b1 = 80 Nói chung dãy (ci , bi ) xác định cách theo công thức ci = bi−2 + bi−1 mod5 và bi = bi−2 +b5i−1 −ci Sử dụng công thức này, ta tính I C 3 1 B 403 80 96 35 26 0 12 0 Từ đó tìm có giá nhỏ là (0, 3, 3, 1, 1, 1, 3, 4, 0, 0, 3, 1), và giá nhỏ là 20 Vậy giá trị nhỏ c0 + c1 + · · · + cn là 20, đạt (0, 3, 3, 1, 1, 1, 3, 4, 0, 0, 3, 1) Bình luận Ý tưởng quy nạp cách giải phần a) là khá tự nhiên Và kiện P (x) chia hết cho Q(x) mà ta dùng cách giải không phải là điều gì đặc biệt Ta có hai tính chất đơn giản quan trọng sau: Tính chất Nếu P (x) và Q(x) là các đa thức với hệ số nguyên, ngoài Q(x) đơn khởi, tức là có hệ số cao thì tồn các đa thức với hệ số nguyên S(x) và R(x) cho i) P (x) = Q(x) · S(x) + R(x); 91 (92) Tạp chí Epsilon, Số 03, 06/2015   ii) deg R(x) < deg Q(x) Tính chất Cho P (x), Q(x) là các đa thức với hệ số nguyên, đó Q(x) bất khả quy, cùng nhận số thực α làm nghiệm Khi đó, P (x) chia hết cho Q(x) Trong phần b), ta đã dùng phương pháp tìm tính chất số tối ưu, sau đó dùng tính chất này để xây dựng số tối ưu đó Lời giải phần b) liên hệ chặt chẽ đến hai cách giải phần a) Đây là bài toán khá thú vị vì nó liên hệ nhiều vấn đề cùng bài toán: đa thức, số nguyên, hệ đếm số, chia hết, thuật toán Một số bài toán liên quan (Nga 2014) Kho bạc nhà nước nước Cộng hòa toán học chọn số α > và sản xuất các đồng xu có mệnh giá rúp và αk rúp với k nguyên dương Người ta nhận thấy mệnh giá (trừ mệnh giá 1) vô tỷ Có thể xảy tình là số nguyên dương n, ta có thể chọn số đồng xu có tổng n và mệnh giá chọn không quá lần? (IMO 1976) Tổng số số nguyên dương 1976 Hỏi tích chúng lớn bao nhiêu? Bài toán Một số nguyên dương k có tính chất T (m) với số nguyên dương a, tồn số nguyên dương n cho 1k + 2k + 3k + · · · + nk ≡ a (mod m) a) Tìm tất các số nguyên dương k có tính chất T (20) b) Tìm số nguyên dương k nhỏ có tính chất T (2015 ) Lời giải a) Đặt Sk (n) = 1k + 2k + · · · + nk thì theo các công thức quen thuộc, ta có n(n + 1) S1 (n) = , n(n + 1)(2n + 1) S2 (n) = , n2 (n + 1)2 S3 (n) = và S4 (n) = n(n + 1)(2n + 1)(3n2 + 3n − 1) 6n5 + 15n4 + 10n3 − n = 30 30 92 (93) Tạp chí Epsilon, Số 03, 06/2015 Sử dụng tính chất nk+4 − nk ≡ (mod 20) với k > ta suy k > thỏa mãn tính chất T (20) và k + thỏa mãn tính chất T (20) (∗) Để có tính chất T (20), trước hết ta phải có tính chất T (5) Lập bảng mô-đun sau N S1 (n) S2 (n) S3 (n) 1 1 4 0 0 1 0 Ta thấy k = 1, 2, không thỏa mãn Vì n5 ≡ n (mod 5) nên k = không thỏa mãn Vậy tất các số không là bội không thỏa mãn tính chất T (20) Ta chứng minh k = thỏa mãn tính chất T (20), từ đó, sử dụng (∗) suy bội số thỏa mãn tính chất T (20) Ta thấy bảng đồng dư mô-đun 20 n4 là N n4 16 16 10 11 16 16 1 Suy bảng đồng dư mô-đun 20 S4 (n) là: N S4 (n) N 12 S4 (n) 10 N 23 S4 (n) 4 17 18 14 19 15 16 12 13 14 15 16 17 18 19 11 12 24 25 26 27 10 11 13 13 14 20 21 22 Như tất các số dư xuất Vậy tất các số thỏa mãn điều kiện T (20) là các bội số b) Theo kết câu a) thì k = 1, 2, không thỏa mãn điều kiện T (2015 ) Ta chứng minh k = thỏa mãn điều kiện T (20m ) với m Từ đó suy k = là giá trị nhỏ cần tìm Để chứng minh điều này, ta chứng minh với a nguyên, tồn n cho 6n5 + 15n4 + 10n3 − n ≡ 30a (mod 30 · 20m ) 93 (94) Tạp chí Epsilon, Số 03, 06/2015 Ta chứng minh với a nguyên: 1) Tồn n cho 6n5 + 15n4 + 10n3 − n ≡ 30a (mod 3), giả sử là n1 2) Tồn n cho 6n5 + 15n4 + 10n3 − n ≡ 30a (mod 22m+1 ), giả sử là n2 3) Tồn n cho 6n5 + 15n4 + 10n3 − n ≡ 30a (mod 5m+1 ), giả sử là n3 Khi đó, theo định lý Trung hoa số dư, tồn n cho n ≡ n1 (mod 3), n ≡ n2 (mod 22n+1 ), n ≡ n3 (mod 5n+1 ) và đó với n này thì 6n5 + 15n4 + 10n3 − n ≡ 30a (mod 30 · 20n ) và ta có điều phải chứng minh Mệnh đề 1) trên là hiển nhiên, ta có thể chọn n = Để chứng minh 2), ta chứng minh quy nạp theo n với a nguyên và với m nguyên dương, tồn n cho 6n5 + 15n4 + 10n3 − n ≡ 30a (mod 2m ) Với m = điều này đúng, ta có thể chọn n = Giả sử tồn n cho 6n5 + 15n4 + 10n3 − n ≡ 30a (mod 2m ) Ta chứng minh tồn n cho 6n5 + 15n4 + 10n3 − n ≡ 30a (mod 2m+1 ) Đặt: 6n5 + 15n4 + 10n3 − n = 30a + u · 2m và chọn N = n + v · 2m Khi đó, ta có 6N + 15N + 10N − N ≡ 6n5 + 15n4 + 10n3 − n − v · 2m = 30a + (u − v) · 2m (mod 2m+1 ) Như cần chọn v = u là ta có N cần tìm Mệnh đề 3) chứng minh cách hoàn toàn tương tự Ta đã hoàn tất việc chứng minh k = thỏa mãn điều kiện T (20m ) với m ∈ N∗ Vậy k = là giá trị nhỏ cần tìm 94 (95) Tạp chí Epsilon, Số 03, 06/2015 Bình luận Các hệ số đa thức 6n5 + 15n4 + 10n3 − n đóng vai trò khá quan trọng bước chuyển từ m lên m + 1, chẳng hạn từ mô-đun 2m lên mô-đun 2m+1 , các hệ số và 10 là chẵn nên có các đồng dư: 6N ≡ 6n5 (mod 2m+1 ), 10N ≡ 10n3 (mod 2m+1 ) Với đồng dư thức 15N ≡ 15n4 (mod 2m+1 ) ta dùng đến nhị thức Newton Tương tự cho trường hợp từ mô-đun 5m lên 5m+1 , các hệ số 15 và 10 chia hết cho 5, còn với hệ thức 6N ≡ 6n5 (mod 5m+1 ), ta dùng đến khai triển nhị thức Newton Việc sử dụng nâng lũy thừa và định lý Trung hoa số dư là kỹ thuật số học mô-đu-la Vì bài toán này khó các bạn chưa quen với các kỹ thuật này, ngược lại đó là bài tập khá đơn giản Đây là điểm yếu bài toán Trong thực tế chấm thi, số học sinh làm đúng phần b) ít Một số bài toán liên quan (VMO 1997) Chứng minh với số nguyên dương n, tồn số nguyên dương k cho 19k + 97 chia hết cho 2n (Saudi Arabia TST 2015) Cho n và k là các số nguyên dương Chứng minh n nguyên tố cùng với 30 thì tồn các số nguyên a và b, số nguyên tố cùng với n, cho a2 − b2 + k chia hết cho n Bài toán Có 100 sinh viên tham dự thi vấn đáp Ban giám khảo gồm 25 thành viên Mỗi sinh viên hỏi thi giám khảo Biết sinh viên thích ít 10 giám khảo số các thành viên trên a) Chứng minh có thể chọn giám khảo mà thí sinh thích ít người đó b) Chứng minh có thể xếp lịch thi cho thí sinh đúng giám khảo mình thích hỏi và giám khảo hỏi không quá 10 thí sinh 95 (96) Tạp chí Epsilon, Số 03, 06/2015 Lời giải a) Cách Gọi A1 là giám khảo nhiều sinh viên thích và a1 là số sinh viên thích A1 Khi đó, sinh viên thích ít 10 giám khảo, ta có a1 ≥ 100 · 10 = 40 25 Ta chọn giám khảo A1 và loại tất các sinh viên thích A1 , còn lại 100 − a1 sinh viên Ta lại xét A2 là giám khảo nhiều các sinh viên còn lại thích và gọi a2 là số sinh viên số sinh viên còn lại thích A2 Ta có a2 ≥ (100 − a1 ) · 10 24 Ta có lúc này a1 + a2 ≥ a1 + (100 − a1 ) · 10 14a1 + 1000 14 · 40 + 1000 = ≥ = 65 24 24 24 Ta chọn giám khảo A2 và loại tất các sinh viên thích A2, còn lại 100 − a1 − a2 sinh viên Tiếp theo, ta xét A3 là giám khảo nhiều các sinh viên còn lại thích và gọi a3 là số sinh viên số sinh viên còn lại thích A3 Ta có a3 ≥ (100 − a1 − a2 ) · 10 23 Ta có lúc này: (100 − a1 − a2 ) · 10 23 13(a1 + a2 ) + 1000 13 · 65 + 1000 = ≥ = 80, 22 23 23 a1 + a2 + a3 ≥ a1 + a2 + Suy a1 + a2 + a3 ≥ 81 Hoàn toàn tương tự ta có (100 − a1 − a2 − a3 ) · 10 22 12(a1 + a2 + a3 ) + 1000 12 · 81 + 1000 = ≥ = 89, 63 23 23 a1 + a2 + a3 + a4 ≥ a1 + a2 + a3 + Suy a1 +a2 +a3 +a4 ≥ 90 Tiếp tục vậy, ta có a1 +· · ·+a5 ≥ 95, a1 + · · · + a6 ≥ 98 và a1 + a2 + · · · + a7 ≥ 100 96 (97) Tạp chí Epsilon, Số 03, 06/2015 Cách Cũng cách 1, ta gọi A1 là giám khảo nhiều sinh viên thích và a1 là số sinh viên thích A1 Khi đó, sinh viên thích ít 10 giám khảo, ta có a1 ≥ 100 · 10 = 40 25 Ta chọn giám khảo A1 và loại tất các sinh viên thích A1 , còn lại tối đa 60 sinh viên Nếu số sinh viên không phải là 60, ta bổ sung thêm số sinh viên “ảo” cho đủ 60 Các sinh viên ảo này thích tất các giám khảo Rõ ràng ta chọn giám khảo cho các sinh viên kể các sinh viên ảo thì là giám khảo phù hợp cho 100 sinh viên ban đầu Bổ sung thế, ta có 60 sinh viên, sinh viên thích ít 10 giám khảo Ta gọi A2 là giám khảo nhiều sinh viên số 60 sinh viên này (cả thật lẫn ảo, có) thích và a2 là số sinh viên thích giám khảo A2 , ta có 60 · 10 = 25 a2 ≥ 24 Ta lại chọn giám khảo A2 và loại tất các sinh viên thích A2 Số sinh viên còn lại tối đa là 60 − 25 = 35 Ta lại bổ sung số sinh viên ảo số sinh viên nhỏ 35, và tiếp tục gọi A3 , a3 có định nghĩa tương tự trên Ta có a3 ≥ 35 · 10 = 15, 21 23 Suy a3 ≥ 16 Ta lại chọn A3 và loại tất các sinh viên thích A3 Số sinh viên còn lại không quá 35 − 16 = 19 Cứ tiếp tục ta • a4 ≥ 9, số sinh viên còn lại sau chọn A4 và loại tất các sinh viên thích A4 không quá 10 • a5 ≥ 5, số sinh viên còn lại sau chọn A5 và loại tất các sinh viên thích A5 không quá • a6 ≥ 2, số sinh viên còn lại sau chọn A6 và loại tất các sinh viên thích A6 không quá • a7 ≥ 2, số sinh viên còn lại sau chọn A7 và loại tất các sinh viên thích A7 không quá 97 (98) Tạp chí Epsilon, Số 03, 06/2015 Vậy sau chọn giám khảo A1 , A2 , , A7 thì không còn sinh viên nào, tức là sinh viên thích giám khảo đã chọn (đpcm) b) Ta xếp lịch thi cho các thí sinh cho giám khảo hỏi thi không quá 10 thí sinh, đến không thể thực thì dừng lại Giả sử còn thí sinh A chưa thi Vì A thích ít 10 giám khảo nên ta xét 10 số giám khảo mà A thích Theo giả sử thì ta không thể cho A thi đó các giám khảo đã đủ “quota”, tức là đã hỏi đủ 10 thí sinh Vì thí sinh hỏi giám khảo nên số thí sinh đã hỏi thi là 10 × 10 = 100, mâu thuẫn Vậy không còn thí sinh nào không hỏi thi và ta có điều phải chứng minh Bình luận Câu a) bài toán này có thể mô hình hóa dạng bài toán tập hợp: Có 100 tập A1 , A2 , , A100 tập hợp X = {1, 2, , 25}, |Ai | ≥ 10 Khi đó tồn Y ⊂ X, |Y | = cho Ai ∩ Y 6= ∅ với i = 1, 2, , 100 Với mô hình này, ta có thể chuyển sang mô hình ma trận thuộc Chẳng hạn, ta có thể theo hướng chứng minh phản chứng và đếm số số ma trận thuộc: Giả sử ngược lại, không tồn Y thỏa mãn yêu cầu đề bài Khi đó với cột ta luôn tìm ít dòng mà giao dòng đó với cột là các số Suy số số trên dòng ít Mặt khác, trên dòng có nhiều 15 số nên là C25 Nếu ta có số số trên dòng nhiều là 100C15 7 C25 > 100C15 thì suy mâu thuẫn Đáng tiếc (!) là đây 7 thì ta lại có C25 < 100C15 nên cách này không khả thi Thú vị là cần giảm số giám khảo từ 25 xuống 24 thì thật 7 và cách giải này trót lọt Ngược lại, thay > 100C14 C24 25 26 thì quy trình giải trên đây cho số giám khảo cần thiết là Như số 25 chọn khá có chủ ý Với các chú ý trên, có hai câu hỏi đặt là: 1) Nếu cố định các thông số 100 (số sinh viên), 25 (số giám khảo), 10 (số giám khảo tối thiểu mà sinh viên thích) thì có thể thay không? 2) Nếu cố định các thông số 100 (số sinh viên), 10 (số giám khảo tối thiểu mà sinh viên thích) và (số giảm 98 (99) Tạp chí Epsilon, Số 03, 06/2015 khảo cần chọn) thì có thể thay 25 26 không? Và có thể thay số lớn là bao nhiêu? Cách giải câu b) trình bày trên cho thấy kết câu b) khá tầm thường và bài thực là bài dễ kỳ thi Xét tổng thể bối cảnh ngày thi thứ hai thì điều này là hợp lý Tuy nhiên, kết bài làm thí sinh bài lần cho thấy tổ hợp là yếu điểm then chốt học sinh Việt Nam Một số bài toán liên quan (IMC 2002) 200 sinh viên tham dự thi toán Họ phải giải bài toán Biết bài toán giải đúng ít 120 sinh viên Chứng minh tồn hai sinh viên mà hợp lại giải đúng bài toán (Putnam) Cho tập hữu hạn X có|X| > 10 A1 , A2 , , A1066 là với i = 1, 2, , 1066 các tập X cho |Ai | > |X| Chứng minh tồn tập A gồm 10 phần tử X cho |A ∩ Ai | ≥ với i = 1, 2, , 1066 (Nga 1993) Mỗi cư dân thành phố N quen với ít 30% cư dân thành phố Một người dân bầu cử người quen người đó tranh cử Chứng minh có thể tổ chức bầu cử thị trưởng thành phố gồm hai ứng cử viên, cho có ít nửa cư dân bầu Bài toán Tìm số nguyên dương n nhỏ cho tồn n số thực a1 , a2 , , an thỏa mãn điều kiện: a1 + a2 + · · · + an > 0, a31 + a32 + · · · + a3n < 0, a51 + a52 + · · · + a5n > Lời giải Ta chứng minh n = là giá trị n nhỏ thỏa mãn điều kiện đề bài Ta nhận xét tồn n số thực thỏa mãn điều kiện đề bài thì với m > n tồn m số thực thỏa mãn điều kiện đề bài (chỉ cần bổ sung thêm m − n số 0) Vì vậy, để chứng minh n = là giá trị nhỏ thỏa mãn điều kiện đề bài, ta cần chứng minh: 1) Không tồn số thực a1 , a2 , a3 , a4 cho a1 + a2 + a3 + a4 > 0, a31 + a32 + a33 + a34 < 0, 99 a51 + a52 + a53 + a54 > (100) Tạp chí Epsilon, Số 03, 06/2015 2) Tồn số thực a1 , a2 , a3 , a4 , a5 cho a31 +a32 +· · ·+a35 < 0, a1 +a2 +· · ·+a5 > 0, a51 +a52 +· · ·+a55 > Ta chứng minh 1) và 2) Để chứng minh 1), ta giả sử ngược lại tồn số thực a1 , a2 , a3 , a4 thỏa mãn a1 + a2 + a3 + a4 > 0, a31 + a32 + a33 + a34 < 0, a51 + a52 + a53 + a54 > Khi đó các số a1 , a2 , a3 , a4 có ít số dương và ít số âm Trường hợp 1: Có số dương, số không dương Không tính tổng quát, giả sử a1 > ≥ a2 , a3 , a4 Đặt bi = −ai thì ta có a1 > b2 + b3 + b4 suy a31 > (b2 + b3 + b4 )3 ≥ b32 + b33 + b34 = −(a32 + a33 + a34 ) Mâu thuẫn Trường hợp 2: Có số không âm, số âm a1 , a2 , a3 > > a4 Đặt b4 = −a4 Ta có a31 + a32 + a33 < b34 , a1 + a2 + a3 > b4 , a51 + a52 + a53 > b54 Từ bất đẳng thức thứ hai suy a1 , a2 , a3 < b4 và a51 + a52 + a53 < a31 b24 + a32 b24 + a33 b24 = (a31 + a32 + a33 )b23 < b53 Mâu thuẫn Trường hợp 3: Có số dương, số âm Gọi hai số dương là a, b, hai số âm là −c, −d thì ta có a, b, c, d > và a + b > c + d, + b5 > c5 + d5 , a3 + b3 < c3 + d3 Ta chứng minh điều này dẫn đến mâu thuẫn Không tính tổng quát, giả sử a ≥ b và c ≥ d Thật vậy, a3 + b3 = (a + b)(a2 − ab + b2 ) < (c + d)(c2 − cd + d2 ) và a + b > c + d nên c2 − cd + d2 > a2 − ab + b2 , suy (c + d)2 + 3(c − d)2 > (a + b)2 + 3(a − b)2 , 100 (101) Tạp chí Epsilon, Số 03, 06/2015 suy c − d > a − b Nếu c ≤ a thì từ đây suy b − d > a − c ≥ 0, suy b > d Từ đó a3 + b3 > c3 + d3 , mâu thuẫn Vậy c > a Và ta có c > a ≥ b > d Đến đây ta có số hướng giải khác sau: Cách Đặt C = dc , A = ad , B = b d thì C > A, B Xét hàm số f (x) = C x + − Ax − B x thì theo điều kiện ta có f (1) < 0, f (3) > 0, f (5) < Ngoài ra, C > A, B nên f (+∞) = +∞ Theo định lý tính chất hàm liên tục, phương trình f (x) = có ít nghiệm Áp dụng định lý Rolle, suy phương trình f (x) = có ít hai nghiệm (∗) Nhưng ta lại có f (x) = C x ln C − Ax ln A − B x ln A   x  x  A B x =C 1− ln A − ln B C C Hàm ngoặc là hàm liên tục và đồng biến trên R+ đồng biến nên có nhiều nghiệm, suy f (x) có nhiều nghiệm, mâu thuẫn với (∗) Cách Ta có b ≤  c3 +d3  13 và a < (c3 + d3 − b3 ) Do đó: c + d − b3  53 + b5 > c5 + d5 (∗∗)   1  c3 +d3 3 3 Xét hàm số f (x) = (c + d − x ) + x trên d, Ta có h 2 i 2 c + d3 − x3 (−3x2 ) + 5x4 = 5x2 x2 − c3 + d3 − x3 ≤  3  13 Vì f là hàm nghịch biến và ta có x ≤ c +d f (x) = f (x) < f (d) = c3 + d3 Do đó (∗∗) không thể xảy Mâu thuẫn Cuối cùng, để hoàn tất phép chứng minh, ta chứng minh tồn số thực a1 , a2 , , a5 cho a1 + a2 + · · · + a5 > 0, a31 + a32 + · · · + a35 < và a51 + a52 + · · · + a55 > 101 (102) Tạp chí Epsilon, Số 03, 06/2015 Ta số dương a, b, c, d, e cho a + b + c > d + e, a3 + b3 + c3 < d3 + e3 , a5 + b5 + c5 > d5 + e5 Đầu tiên ta chọn a = 2x, b = c = 1, d = e = x + thì a + b + c = d + e Ta chọn x cho (2x)3 +2 < 2(x+1)3 và (2x)5 +2 > 2(x+1)5 Giải các bất phương trình này, ta có thể chọn x = 1, Sau đó điều chỉnh d chút, ta có thể chọn a = 3, b = c = 1, d = e = 2, 45 Bài toán giải hoàn toàn Bình luận Bài toán này khá thú vị cách đặt vấn đề, khá mẻ cho dù bất đẳng thức nghiên cứu cách khá rầm rộ và có nhiều công cụ mạnh trang bị cho học sinh Để giải bài này, học sinh cần áp dụng chút tư tổ hợp (trong việc phân trường hợp), phép chứng minh phản chứng và biến đổi đại số (hoặc giải tích) khá tinh tế Việc xây dựng ví dụ đòi hỏi khả dự đoán và khoanh vùng Tuy nhiên đây là bài toán nặng kỹ thuật, không thật phù hợp cho bài toán olympic, thí sinh phải làm bài điều kiện thời gian hạn chế và không sử dụng các công cụ trợ giúp Trong quá trình tìm ví dụ cho trường hợp n = 5, cách tự nhiên ta phát bài toán sau: Nếu a, b, c, d, e là các số thực dương cho a + b + c = d + e, a5 + b5 + c5 = d5 + e5 thì a3 + b3 + c3 < d3 + e3 Một cách tự nhiên, ta có thể đặt câu hỏi tổng quát: Tìm số n nhỏ cho tồn n số thực a1 , a2 , , an có tổng các luỹ thừa bậc 4k + dương (k = 0, 1, , m) và tổng các lũy thừa bậc 4k − âm (k = 1, 2, , m) Lời giải cho bài toán tổng quát có lẽ là khó Ngay trường hợp m = đã là khó Một câu hỏi khác xuất phát từ lời giải bài toán trên: Cho a1 , a2 , , P am ; b1 , b2 , P , bn là m + n số thực dương Hỏi m n x x phương trình a = i=1 i i=1 bi có nhiều bao nhiêu nghiệm thực? 102 (103) Tạp chí Epsilon, Số 03, 06/2015 Một số bài toán liên quan (Ucraina 2014, lớp 9) Tìm tất các số nguyên dương n cho tồn n số thực thỏa mãn điều kiện a1 + · · · + an > và a31 + · · · + a3n < (Bài này khá đơn giản: Với n = 1, rõ ràng không thỏa mãn, còn với n = thì chọn a1 = a2 = 32 , a3 = −1.) (IMC 2006) Cho a, b, c, d, e là các số thực dương cho a2 + b2 + c2 = d2 + e2 , a4 + b4 + c4 = d4 + e4 Hãy so sánh các số a3 + b3 + c3 và d3 + e3 (Tournament of the Towns, 2006) Các số thực dương x1 , , xk thỏa mãn điều kiện x21 + · · · + x2k < x1 + · · · + xk , x1 + · · · + xk < x31 + · · · + x3k a) Chứng minh k > 50 b) Tìm ví dụ với giá trị k nào đó c) Tìm giá trị nhỏ k để tồn ví dụ 103 (104) Tạp chí Epsilon, Số 03, 06/2015 104 (105) MARTIN GARDNER NGƯỜI LÀM VƯỜN CỦA TOÁN HỌC ĐẶNG NGUYỄN ĐỨC TIẾN (Đại học Trento, Italy) Roses are red, Violets are blue, Sugar is sweet, And so are you Có bài đồng dao thế, và có vĩ nhân đã nguệch ngoạc viết thêm vào bài báo mình sau: Roses are red Violets are blue, One point 414 Is the square root of two Giới thiệu Nền toán học kỷ hai mươi đã sản sinh nhiều cây đại thụ John Nash, Andrew Wiles, Grothendieck, Paul Erdös, Alan Turing Họ là cái tên vĩ đại mà tầm ảnh hưởng đã vượt xa khỏi thời đại mình Nhưng bài viết này chúng tôi lại muốn giới thiệu đến vĩ nhân khác: Martin Gardner, nhân vật trọng yếu lịch sử toán học thân lại không phải là nhà toán học, chí chưa kinh qua đào tạo bài toán Trong suốt nghiệp mình, ông không phải là người đóng góp công trình nghiên cứu hay phát kiến vượt bậc Nhưng ông là người truyền cảm hứng, nhà làm vườn đã gieo trồng niềm vui và đam mê toán học cho nhiều hệ người trẻ tuổi Giống Richard K Guy đã viết: “Gardner, ai, đã đem toán học đến hàng triệu người”, và theo Donald Knuth, “Rất nhiều người đã học nhiều ý tưởng toán học hay từ Gardner người nào khác lịch sử giới.” 105 (106) Tạp chí Epsilon, Số 03, 06/2015 Giải toán theo cách Martin Gardner Trước tiên, chúng ta hãy khởi động với ba bài toán theo phong cách Martin Gardner sau: Bài toán Góc hai đường thẳng màu đỏ vẽ từ hai mặt khối hộp hình là bao nhiêu? Bài toán Xét vòng kim loại hình bên đốt nóng và bị nở Điều gì xảy với lỗ bên nó? Nó co lại, nở hay giữ nguyên kích thước? Bài toán Hai bù-loong giống hệt đặt lại đối để rãnh xoắn ốc chúng dính vào hình đây Nếu bạn cùng lúc xoay bù-loong theo chiều nó (giữ tay tránh làm chệch hướng) thì hai đầu chúng tiến lại gần hay xa nhau, hay khoảng cách không đổi? 106 (107) Tạp chí Epsilon, Số 03, 06/2015 Đây là số bài toán phổ biến theo phong cách Martin Gardner Nếu bạn đọc đã đọc qua sách Martin, hẳn đã nắm rõ câu trả lời Tuy nhiên, điều thú vị ba bài toán này là chúng không phải dành riêng cho người nghiên cứu toán Chúng có thể giải có kiên nhẫn, đam mê và tất nhiên với chút kiến thức phổ thông vật lý Chúng không hứa hẹn trao cho bạn khám phá vượt bậc nào, mà đơn giản, chúng mang lại cảm giác thoả mãn ngào cho tìm lời đáp Đúng Robert P Crease đã viết: “Tìm kiếm câu trả lời Google không phải là phong cách Gardner Cách Gardner là tự mình đốt cháy niềm đam mê và trải nhiệm niềm vui vào lúc tự mình tìm câu trả lời.” Và cách đó, suốt 25 năm với chuyên mục “Các trò chơi toán học” tờ báo khoa học thường thức danh tiếng Scientific American, Gardner đã gieo mầm đam mê toán học cho hàng triệu người lứa tuổi, ông đã đem toán học đến với nhiều người lịch sử toán học đã làm “Martin đã biến hàng ngàn đứa trẻ thành nhà toán học và hàng ngàn nhà toán học thành đứa trẻ” Ronald Graham, trích Thời báo New York số ngày 19 tháng 10 năm 2009 107 (108) Tạp chí Epsilon, Số 03, 06/2015 Cuộc đời và nghiệp Martin Gardner sinh vào ngày 21 tháng 10 năm 1914 Tulsa, Oklahoma, và ngày 22 tháng năm 2010, không xa nơi ông đã sinh ra, thành phố Norman, Oklahoma Từ trái sang phải: Em trai, cha, và Martin Gardner Cha ông có tiến sĩ địa chất và mẹ ông đã dạy tiểu học Lexiton, sau đó bà nghỉ việc để chăm sóc cho ba anh em ông Thuở nhỏ, Gardner thích chơi bài, các trò ảo thuật và đọc tác phẩm phiêu lưu “Phù thủy xứ Oz” nhà văn Braum, hay “Alice lạc vào xứ thần tiên” Lewis Caroll Chính thú đam mê trò chơi trí tuệ và câu chuyện phiêu lưu lạ lùng thuở ấu thơ đã ảnh hướng lớn đến định gắn bó với Toán học giải trí đời Gardner Ông kể rằng: “Mẹ tôi đọc “Phù thủy xứ Oz” cho tôi nghe tôi bé, và tôi đã nhìn qua vai mẹ lần bà đọc nó Tôi đã học chữ đấy.” Năm 1936 Gardner tốt nghiệp khoa triết trường Đại học Chicago Sau làm việc thời gian ngắn phòng truyền 108 (109) Tạp chí Epsilon, Số 03, 06/2015 thông Đại học Chicago, năm 1937 ông trở lại Oklahoma và trở thành phóng viên cho tờ báo Tulsa Tribune Sau đó ông chuyển đến làm việc cho tạp chí Humpty Dumpty, tạp chí dành cho thiếu nhi Ông viết đặn tháng câu truyện ngắn và bài thơ đưa lời khuyên đạo đức cho thiếu niên Đến năm 1956, ông mời viết bài cho chuyên mục các trò chơi toán học (Mathematical Games) với Scientific American Cũng từ đó nghiệp huyền thoại Gardner đã bắt đầu Cùng với niềm say mê ảo thuật và đầu óc hiếu kỳ, Gardner đã trụ vững tạp chí khoa học có uy tín này và đặn đưa câu đố làm bối rối độc giả Lạ lùng, mẻ, sinh động và dễ hiểu là yếu tố đã khiến chuyên mục xuất đặn từ 1956 đến 1980 Gardner nhiều người đón đọc Chuyên mục ông đã mở cánh cổng dẫn đến giới toán học đầy rẫy điều thú vị Chúng không mang lại niềm vui cho người yêu toán hay nhà toán học mà còn góp phần nuôi dưỡng tình yêu toán cho hệ trẻ tuổi, truyền cho họ cảm hứng và niềm say mê giải các vấn đề Gardner đã phát biểu thành công mình sau: “Bí mật lớn cho thành công tôi với vai trò chủ bút chuyên mục là tôi không biết nhiều toán.” 109 (110) Tạp chí Epsilon, Số 03, 06/2015 Martin nói với tôi ông dành 25 ngày tháng cho chuyên mục mình trên tờ Scientific American Persi W Diaconis Một bò không có kỹ giải vấn đề tinh tinh, chúng đã biết cách dùng gậy để chọc vào tổ để đuổi đám mối khỏi mặt đất Sự tiến hóa đã phát triển khả não để giải vấn đề, và cùng thời điểm niềm vui việc giải vấn đề đã tạo thành não chúng ta Martin Gardner Science Good, Bad and Bogus (1981), 123 Ông tin giây phút người tận hưởng niềm vui tìm đáp án vấn đề là điều quan trọng văn hoá nhân loại Ông tin điều làm nên khác biệt xã hội công nghiệp đại với thời kỳ Hy Lạp cổ là khả giải các câu đó Một màn hình tivi thời đại, không thể là nỗ lực cá nhân ngày ngày hai mà là thành hàng trăm người đã góp trí lực vào giải câu đố nhỏ Đối với ông niềm vui đến từ câu đố nhỏ nhặt Một thứ niềm vui khiết và diệu kỳ không kém gì nhà khoa học phát kiến điều vĩ đại Đó là thứ niềm vui mà Gardner truyền vào chuyên mục mình suốt phần tư kỷ Ông đơn giản hoá phát kiến toán học lớn lao Ông nhìn vấn đề vĩ đại với mắt nhà khám phá và viết lại chúng theo cách nhà văn kể chuyện xứ Oz Ông đặt lửa dẫn đường cho hệ trẻ tự mình mày mò và say đắm giới thần tiên toán học Suốt nghiệp mình, ông đã đóng vai trò là người truyền cảm hứng, nhà ảo thuật biến điều phức tạp cao siêu trở thành món quà bí ẩn và thú vị cho tất giàu lòng say mê khám phá 110 (111) Tạp chí Epsilon, Số 03, 06/2015 Trong lễ hội Toán học (1975) ông viết: “Tôi cách tốt để đánh thức sinh viên là đưa cho họ trò chơi toán học hấp dẫn, câu đố, thủ thuật ma thuật, câu nói đùa, nghịch lý, mô hình, hay bài thơ câu (limerick) Không phải tôi đề nghị giáo viên không nên làm gì khác ngoại trừ giới thiệu các trò chơi giải trí cho sinh viên Rõ ràng có hoán đổi nghiêm túc và tính giải trí Giải trí giữ cho người đọc tỉnh táo Sự nghiêm túc làm cho trò chơi đáng giá.” Martin Gardner và vợ Bên cạnh Scientific American, Martin Gardner còn là tác giả 70 đầu sách, các trò chơi, ảo thuật, triết học, thiên văn, tôn giáo Ông có bạn bè và đồng nghiệp ông khắp các lĩnh vực sáng tạo, từ các nhà logic học, toán học Raymond Smullyan, Roger Penrose, Piet Hein đến các nhà văn Isaac Asimov và Vladimir Nabokov, và các nghệ sĩ M C Escher và Salvador Dalí Vì không có gì ngạc nhiên tác phẩm ông bao hàm nhiều lĩnh vực khác Ngoài ông là chuyên gia hàng đầu Mỹ Lewis Carroll – tác giả “Alice lạc vào xứ thần tiên” Ngày 22/5/2010, Martin Gardner qua đời bệnh viện Norman, Oklahoma 111 (112) Tạp chí Epsilon, Số 03, 06/2015 Những hạt mầm khu vườn Martin Gardner Trong phần tư kỷ (1956-1980), đề mục chung Các trò chơi toán học, Martin Gardner đã làm say mê không nhiêu bạn đọc trò chơi trí tuệ vừa dễ hiểu vừa sâu sắc và gợi mở nhiều khí cạnh mẻ Tất chúng không là trò chơi ông sáng tạo mà phần lớn là gì ông tìm thấy trên sách Sau đó thông qua trao đổi với các nhà khoa học để nắm bắt ý tưởng, ông đã trình bày lại chúng dạng trò chơi đơn giản Trong phần này bài viết, chúng tôi muốn giới thiệu đến độc giả vài bài toán khá quan trọng và tiếng đã Gardner giới thiệu qua chuyên mục “Các trò chơi toán học" trên tờ Scientific American qua số sách ông 3.1 Flexagon Flexagon là bài viết đầu tiên tính từ Gardner chính thức tham gia Scientific American (ông có bài trước đó vào năm 1952 là “Các máy logic” còn làm việc Humpty Dumpty) Bài viết này hoàn thành vào tháng 12 năm 1956, và nhờ vậy, vào tháng Giêng 1957, chuyên mục các “Các trò chơi toán học” chính thức đời Flexagon là mẫu giấy/vải phẳng, có thể gấp nếp hay uốn cong để có thể lật hai mặt trước và sau cách sáng tạo, độc đáo Ví dụ sau đây cho thấy cách tạo Flexagon với mảnh hình tam giác gấp và “uốn cong” lại thành lục giác Sau đó, vài động tác, các mặt này có thể thay đổi hoán đổi vị trí cho để tạo thành các hình dạng khác Đôi người ta viết lên các mặt Flexagon số hay lời tiên đoán vận may và dùng nó là trò chơi Độc giả có thể liên tưởng Flexagon với trò chơi “đông tây nam bắc” quen thuộc Việt Nam Flexagons có lịch sử phong phú, phát minh cách tình cờ vào năm 1939 Arthur Harold Stone, đương thời là 112 (113) Tạp chí Epsilon, Số 03, 06/2015 nghiên cứu sinh đại học Princeton Chuyện là trên đường từ Anh sang Mỹ, Stone đã tình cờ cắt mẫu báo lớn và gấp nó, là Flexagon đời Stone sau đó cùng với các bạn mình Princeton là John Tukey (nổi tiếng với phép biến đổi Fourier nhanh (FFT)), Bryant Tuckerman (nhà tô-pô học, người phát số nguyên tố Mersenne thứ 24), và Richard Feynman (nhà vật lý xuất sắc, cha đẻ sơ đồ Feynman) đã khai phá các tính chất toán học Flexagon Nhưng chiến tranh nổ sau đó đã khiến bài báo họ chìm vào quên lãng 15 năm sau, Martin Gardner đã làm sống lại nó, đặt tảng đưa ông vào giai đoạn thành công nghiệp mình 113 (114) Tạp chí Epsilon, Số 03, 06/2015 3.2 Bài toán thư ký Năm 1960, trên tờ Scientific American, lần đầu tiên Martin Gardner đã giới thiệu bài toán sau: “Yêu cầu người lấy số mảnh giấy tùy ý và viết vào mảnh số nguyên dương khác Các số này có thể nhỏ hay lớn đến kích thước googol (bắt đầu và theo sau là trăm số 0) chí lớn Các mảnh này đặt úp mặt xuống và xếp ngẫu nhiên trên mặt bàn Tại thời điểm bạn phép chọn mảnh và lật lên Mục tiêu bài toán là tìm thời điểm kết thúc việc lật số này để chọn mảnh giấy có số lớn Bạn không thể quay lại và chọn cách mảnh giấy đã lật trước đó Nếu bạn đã lật tất các mảnh giấy thì mảnh cuối cùng chính là mảnh chọn." Một bài toán tương tự đề xuất bới Arthur Cayley vào năm 1875 và có thể bài toán đã đưa từ trước đó khá lâu Johannes Kepler Về sau, bài toán sau thường biết đến với cái tên "bài toán thư ký", chi tiết độc giả có thể xem đây Bài toán trên có lời giải đẹp Đầu tiên chọn và lật hết n/e mảnh đầu tiên (trong đó e là số lôgarit tự nhiên) Sau đó chọn mảnh đầu tiên số mảnh còn lại có giá trị lớn tất các mảnh đã lật Nếu áp dụng thuật toán này thì xác suất chọn mảnh có số lớn là khoảng 1/e và đây là xác suất tối ưu Bài toán thư ký là bài toán tiếng lý thuyết dừng tối ưu Bài toán này đã nghiên cứu xác suất ứng dụng, thống kê, và lý thuyết định 3.3 Các bài toán nghịch lý Các bài toán chọn Gardner bao gồm nhiều lĩnh vực, và các nghịch lý là vấn đề yêu thích ông Một bài toán kinh điển thường nhắc đến các giáo trình xác suất là nghịch lý ngày sinh: Trong nhóm người chọn ngẫu nhiên, xác suất để có hai người có cùng ngày sinh là bao nhiêu? 114 (115) Tạp chí Epsilon, Số 03, 06/2015 Trái với dự đoán cảm tính thông thường (theo nguyên lý Dirichlet chẳng hạn), xác suất khá cao và buổi tiệc có 70 người thì xác suất có người có cùng ngày sinh lên đến 99.9% Chi tiết hơn, độc giả có thể xem đây Một nghịch lý tiếng khác Gardner giới thiệu vào tháng năm 1974 mà ngày chúng ta gọi là "Nghịch lý Newcomb", bắt nguồn từ nhà toán học William Newcomb Bài toán này sau: Giả sử bạn tham gia trò chơi và trước mặt bạn có hai hộp đậy kín Hộp chắn có chứa 1.000$, hộp thứ có thể trống, chứa 1.000.000$ Bạn phép chọn hai hộp chọn hộp thứ Luật chơi còn cho biết thêm là người dẫn trò dự đoán trước đó xem bạn chọn phương án nào: họ đoán bạn chọn hộp thì để trống hộp thứ 2, và họ đoán bạn chọn phương án chọn hộp thứ 2, họ đặt vào đó 1.000.000$ Nếu họ đoán bạn chọn ngẫu nhiên hai phương án, họ để hộp trống Và quan trọng hơn, người dẫn trò luôn đoán đúng ý định bạn! Các trường hợp có thể có cho nghịch lý Newcomb TH1 TH2 TH3 TH4 Dự đoán Lựa chọn Số tiền nhận Cả hộp Cả hộp 1.000 Cả hộp Chỉ mở hộp Chỉ mở hộp Cả hộp 1.001.000 Chỉ mở hộp Chỉ mở hộp 1.000.000 Bài toán gọi là nghịch lý vì có hai cách suy luận trái mà có vẻ hợp lý Lập luận thứ cho rằng, chiến thuật người dẫn trò sao, lấy hộp luôn có lợi Điều này xảy vì người dẫn trò đoán bạn chọn hộp (TH1 và TH2) thì tối đa bạn lấy 1.000 $ (ứng với TH1) và tình người dẫn trò đoán bạn chọn hộp (TH3 và TH4) thì chọn hộp (ứng với TH3) bạn 1.001.000 $ Lập luận thứ hai cho rằng, chọn hộp là phương án đúng đắn, vì người dẫn trò luôn đoán đúng ý định bạn, nên có TH1 và TH4 xảy ra, chọn phương án mở hộp có lợi với phần thưởng 1.000.000 $ Chi tiết và giải đáp cho nghịch lý này, độc giả có thể tìm hiểu thêm đây 115 (116) Tạp chí Epsilon, Số 03, 06/2015 3.4 Bài toán tô màu Trên Scientific American số tháng năm 1975, Gardner đã đăng đồ giấy và đặt yêu cầu dùng màu khác để tô cho các vùng cho hai vùng kề phải tô hai màu khác Trong trò chơi này ông đã cố tình chơi trò "Cá tháng tư" và nói đồ với 110 vùng bên là phản ví dụ cho định lý màu Nguồn: Wolfram Bài toán tô màu nêu khoảng năm 1840 Möbius, cha đẻ dải băng Möbius tiếng Giả thuyết đầu tiên định lý 4-màu phát biểu chính thức lần đầu tiên Guthrie vào năm 1852, còn gọi là định lý Guthrie, phát biểu với đồ phẳng tô màu cho vùng có cùng đường biên không tô giống màu Hai số người tiên phong chứng minh định lý này là Alfred Kempe, người nghiên cứu luật, vào năm 1879, và Peter Tait, nhà vật lý, vào năm 1880 Nhưng 10 năm sau, Percy Heawood đã sai lầm cách chứng minh Kempe và đồng thời đưa định lý 5-màu Đến năm 1891, Julius Petersen sai lầm cách chứng minh Tait Hơn kỷ kể từ đời, đến năm 1977, cuối cùng thì định lý chứng minh Kenneth Appel và Wolfgang 116 (117) Tạp chí Epsilon, Số 03, 06/2015 Haken trường Đại học Illinois với trợ giúp máy tính Mặc dù nhiều nhà toán học không chấp nhận kết này nghi ngờ tính đúng đắn xử lý máy tính và các vấn đề liên quan Năm 1996, chứng minh độc lập, và ngắn gọn đưa Robertson và cộng Chi tiết cho bài toán tiếng này, độc giả có thể xem đây Dưới đây là cách tô màu đồ đặt câu hỏi Gardner, tìm thấy Stan Wagon, nhà toán học quen thuộc với Epsilon hai số đầu tiên Mặc dù đồ phẳng nào chứng minh là tô màu, thực tế việc tìm cách tô với màu cho đồ cụ thể thường là thách thức Lời giải cho bài toán tô màu đồ Martin Gardner với màu Nguồn: Wolfram 3.5 Phân dạng Phân dạng, hay quen thuộc là Fractal hay hình (học) Fractal, là các hình có cấu trúc tự đồng dạng, giới thiệu Gardner vào tháng 12 năm 1976, với tên gọi ban đầu là "đường cong quái vật" (Monster curve) Các nhà toán học bắt đầu nghiên cứu các hình tự đồng dạng từ kỷ 17, Gottfried Leibniz xem xét các đường gấp khúc và 117 (118) Tạp chí Epsilon, Số 03, 06/2015 định nghĩa đường thằng là đường phân dạng chuẩn: "các đường thẳng là đường cong, phần nào nó tương tự với toàn bộ" Benoı̂t Mandelbrot là người đầu tiên đưa khái niệm fractal, bắt nguồn từ tiếng La-tinh fractus nghĩa là "đứt gãy" Mandelbrot sống không xa Gardner bài này viết, và thời gian này, nhà riêng mình, Gardner giới thiệu Mandelbrot với Conway, nhà toán học người Anh chuyên nghiên cứu lý thuyết số, mã hóa, tổ hợp và đặc biệt là người phát minh "trò chơi đời" Như Gardner thuật lại hồi ký mình, "Conway đã có phát cho bài toán ghép hình Penrose (Penrose tiling), và Mandelbrot đã quan tâm đến các phát minh này vì mẫu ghép Penrose có dạng Fractals Bạn có thể mở rộng thu hẹp chúng, chúng luôn luôn có hình dạng tương tự." 3.6 Mã hóa RSA Tháng năm 1977, Martin Gardner cho đăng bài “Một cách mã hóa hàng triệu năm để giải mã" Bài viết này đã giới thiệu phương pháp mã hóa RSA, phương pháp mã hóa công khai mà trước đây không tin có thể thực Khi nhận bài viết từ tác giả Ron Rivest, Adi Shamir và Leonard Adleman, Gardner đã thích thú và có ấn tượng sâu sắc Ông đã phá vỡ nguyên tắc thông thường mình, chuẩn bị bài đăng từ trước đó nhiều tháng mà xuất bài viết phương pháp RSA 118 (119) Tạp chí Epsilon, Số 03, 06/2015 Ý tưởng RSA độc giả có thể tìm hiểu đây Để chứng tỏ khả RSA, các tác giả đã gửi cho Gardner thông điệp E đã mã hóa gồm 128 ký tự, cùng với khóa công khai s = 9007 và trao giải thưởng 100 $ cho giải mã thông điệp gốc E Thông điệp đã mã hóa thách thức Gardner gửi đến độc giả Mật mã RSA đã trở thành tiêu chuẩn công nghiệp và các biến thể nó sử dụng ngày hôm Trong thời gian gần đây các câu hỏi độ an toàn nó đã xem xét lại Và dù cho bài viết này Gardner mang tính chất đột phá, tiêu đề nó đã không hoàn toàn chính xác Thông điệp thách thức đã giải mã thành công vào khoảng tháng năm 1994, 17 năm sau bài toán đăng thay vì hàng triệu năm Ngoài vấn đề nêu trên, nhiều bài toán tiếng và quan trọng khác đã Gardner phổ biến đến đông đảo độc định lý lớn Fermat, trò chơi đời, giả thuyết 3n + và hẳn độc giả hãy còn nhớ, "bài toán đội nón" mà chúng tôi đã giới thiệu hai số Epsilon đầu tiên ươm mầm từ người bạn tốt mà toán học có Những điều ít biết Gardner Chú giải tác phẩm Alice lạc vào xứ thần tiên (The Annotated Alice) (1960) là tác phẩm bán chạy ông Nó phản ánh 119 (120) Tạp chí Epsilon, Số 03, 06/2015 tình yêu Gardner các tác phẩm nhà văn tiếng người Anh, Lewis Carroll (cũng chính là nhà toán học Charles Dodgson) Những chú giải Gardner đã giải đáp các ẩn ý lô-gic và toán học “Alice lạc vào xứ thần tiên” và "Through the Looking-Glass" Gardner và Alice Central Park Bên cạnh niềm say mê toán học, Gardner là cây bút nhiệt tình việc lên tiếng chống lại tượng mạo danh khoa học Trong số 100 sách, “Những trò lố bịch nhân danh khoa học” đã nói nhiều lỗi khoa học cùng trò thủ đoạn Năm 1976, Gardner tham gia vào nhóm Carl Sagan, Isaac Asimov và nhiều người khác thành lập Ủy ban điều tra tượng huyền bí, siêu nhiên Ông đã xuất bài báo khoa học (có bình duyệt) đầu tiên tuổi 74! Bài báo ông viết chung với cặp vợ chồng Fan Chung và Ronald Graham đăng tạp chí Mathematics Magazine Hơn hai thập kỷ kế tiếp, nhiều bài báo khác đăng trên MAA, bao gồm bài báo đã đạt giải thưởng Math Horizons “The Square Root of Two = 1.41421 35623 73095 ” (Căn bậc là 1.41421 35623 73095 ) khảo sát tính vô tỷ bậc mà chúng tôi đã sử dụng bốn câu "đồng dao" đầu tiên bài ông làm lời "đề từ" cho bài viết này mình Đây là điểm kết cho bài viết chúng tôi giới thiệu Martin Gardner, người làm vườn vĩ đại toán học 120 (121) Tạp chí Epsilon, Số 03, 06/2015 Chú thích Vì bài viết trích dẫn và đề cập đến nhiều nhà khoa học, nhà văn, nghệ sĩ khác nên chúng tôi liên kết trực tiếp tiểu sử người thông qua Wikipedia mà không sử dụng chú thích Bài viết này tổng hợp và biên dịch từ các nguồn sau: BBC - Martin Gardner, puzzle master extraordinaire Tham khảo chủ yếu cho phần Huffingtonpost - Martin Gardner – The Best Friend Mathematics Ever Had - Colm Mulcahy Tham khảo chủ yếu cho phần AMS - Magical Mathematics - A Tribute to Martin Gardner - Joseph Malkevitch Tham khảo chủ yếu cho phần Tia sáng - Martin Gardner Người gợi cảm hứng cho niềm say mê toán học - Ngọc Tú Tham khảo chủ yếu cho phần New York Times - Martin Gardner, Puzzler and Polymath, Dies at 95 - Douglas Martin Tham khảo chủ yếu cho phần Scientific American (blog) - The Top 10 Martin Gardner Scientific American Articles - Colm Mulcahy Tham khảo chủ yếu cho các phần và Trò chơi đời - Hà Dương Tường Tham khảo chủ yếu cho phần Creational Mathematics Magazine No 2014 - Martin Gardner’s Mathemagical Life - Tereza Bártlová Tham khảo cho toàn bài Tất các tài liệu từ Internet truy cập vào tháng năm 2015 121 (122) Tạp chí Epsilon, Số 03, 06/2015 122 (123) CỰC TRỊ TẬP HỢP TRẦN MINH HIỀN (Trường THPT Chuyên Quang Trung, Bình Phước) Một số định lý quan trọng Định nghĩa 1.1 Cho F là họ các tập hợp tập n phần T tử X Khi đó F gọi là họ giao với A, B ∈ F ta có A B 6= ∅ Định lý 1.2 Cho F là họ giao tập n phần tử X Khi đó |F| ≤ 2n−1 Chứng minh Lấy tập A ⊆ X Khi đó với cặp tập (A, X\A) X, thì nhiều là hai tập A X\A thuộc vào F (vì hai tập A, X\A thuộc vào F thì A ∩ (X\A) = ∅ mâu thuẫn với F là họ giao các tập X) Vì X có 2n tập con, và cặp tập (A, X\A) có nhiều tập thuộc F Do đó |F| ≤ · 2n = 2n−1 Định lý 1.3 (Định lý Erdos-Ko-Rado) Một họ F các k_tập (một tập hợp có k phần tử gọi là k_tập) tập n phần tử X n (k ≤ ) Khi đó   n−1 |F| ≤ k−1 Chứng minh Với n, k là các số nguyên dương với n ≥ 2k Một k_cung là tập {i, i + 1, , i + k}, với các số nguyên lấy theo modulo n Một cách hình dung cho k_cung là k đoạn cung tròn liên tiếp, nối hai điểm i và i + k(modn) trên đường tròn Ta nói hai cung A và A0 giao chúng có chung đoạn cung tròn (k_cung và hai cung giao minh họa hình đây) 123 (124) Tạp chí Epsilon, Số 03, 06/2015 Một họ {A1 , A2 , , At } các k_cung giao đôi [n] thì t ≤ k Thật vậy, điểm i (điểm gắn cho cung tròn) là điểm kết thúc hai cung: cung nhận i là điểm đầu tiên và cung nhận i là điểm cuối cùng Do hai cung này không giao nhau, dẫn đến nhiều hai cung này thuộc vào họ F Xét cung A1 cho trước Vì các cung còn lại phải giao với A1 đoạn cung chung Do đó các cung còn lại phải nhận 124 (125) Tạp chí Epsilon, Số 03, 06/2015 các điểm cung A1 , là các điểm i1 , i2 , , ik−1 , là điểm kết thúc Vì điểm kết thúc có không quá k_cung thuộc F, nên có k − điểm kết thúc có không quá k_cung thuộc F, cộng với cung A1 thì có không quá k_cung thuộc tập F Bây xét hoán vị [n] có dạng (a1 , a2 , , an ) Ta đánh dấu các đoạn cung tròn các số hình vẽ ban đầu Mỗi tập F xem là k_cung hoán vị này Theo kết trên, thì với hoán vị này ta có ≤ k các k_cung • Lấy tổng hết tất các hoán vị [n] trên đường tròn, có (n − 1)! hoán vị, thì ta thấy có nhiều là k(n − 1)! các tập này • Tuy nhiên, xếp trên đường tròn và cách đếm trên, tập A1 đếm làm nhiều lần (vì xếp trên đường tròn, thì tập A1 hay tập nào lấy để làm thứ tự là giống nhau) Vì nhiều lần hoán vị (n−1)!, tạo hoán vị khác nhau, phần tử A1 Do đó có k! cách xếp các phần tử A1 và (n − k)! cách xếp các phần tử bù tập A1 Do đó |F|k!(n − k)! ≤ k(n − 1)! 125 (126) Tạp chí Epsilon, Số 03, 06/2015 k(n − 1)! ⇒ |F| ≤ = k!(n − k)!   n−1 k−1 Định lý 1.4 (Bollobas) Cho A1 , A2 , , Am và B1 , B2 , , Bm là các tập S = [n] Đặt = |Ai | và bi = |Bi | với i = 1, 2, , m Giả sử Ai ∩ Bj = ∅ và i = j Khi đó −1 m  X + b i i=1 ≤ Chứng minh Với hoán vị số n! hoán vị các phần tử S, ta nói hoán vị π là chứa B sau A tất các phần tử A đứng trước các phần tử B π Với hoán vị π có tính chất chứa Bi sau Ai , mà có thêm tính chất chứa Bj sau Aj , đó Ai ∩ Bj = ∅ (nếu Ai đứng trước Bj , minh họa hình dưới) Aj ∩ Bi = ∅ (khi Ai kết thúc sau Bj ) Nhưng điều này mâu thuẫn với giả thiết Ai ∩ Bj = ∅ và i = j Do đó, với hoán vị π, tồn nhiều số i để π chứa Bi sau Ai Ngược lại, với i ∈ {1, 2, , m}, ta đếm số hoán vị mà Ai đứng trước Bi  n • Chọn + bi ví trí để xếp cho Ai và Bi , ta có +b i cách • Đặt phần tử Ai vào vị trí đầu tiên + bi ví trí vừa chọn có ! cách, sau đó xếp bi các phần tử Bi vào các vị trí còn lại có bi ! cách Vậy có !.bi ! cách xếp Ai trước Bi 126 (127) Tạp chí Epsilon, Số 03, 06/2015 • Sắp xếp n − − bi phần tử còn lại S vào n − − bi vị trí còn lại có (n − − bi )! cách Vậy ta có  n + b i  · ! · bi ! · (n − − bi )! = n! +bi  Lấy tổng số tất các các số i = 1, 2, , m ta có −1 m m  X X n! + b i  ≤ n! ⇒ ≤ +bi a i a i=1 i=1 i Định lý chứng minh Định nghĩa 1.5 Cho S là tập hợp hữu hạn Một họ các tập A1 , A2 , , An S gọi là xích Ai ⊂ Ai+1 và |Ai+1 | = |Ai | + ∀i = 1, 2, , n Do đó xích poset là tập hợp {x1 , x2 , , xk } thỏa mãn x1 ≤ x2 ≤ ≤ xk Định nghĩa 1.6 Cho P là tập hợp hữu hạn Một họ các tập A1 , A2 , , An S gọi là phản xích Ai 6⊂ Aj , ∀i 6= j Khi đó phản xích poset chính là tập các phần tử y1 , y2 , , yk mà yi , yj không thể so sánh theo quan hệ poset với i 6= j Định lý 1.7 (Bất đẳng thức LYM, Lubell, Yamamoto, Meshalkin) Cho F là phản xích [n] (ta dùng ký hiệu [n] thay cho {1, 2, , n}) Khi đó ta có bất đẳng thức X  n −1 ≤ |A| a∈F Chứng minh Gọi C là tập hợp tất các xích C, xích C gồm n tập hợp, tập hợp là tập [n] và đây là xích chứa nhiều phần tử nhất, C1 ⊂ C2 ⊂ ⊂ Cn , |Ci | = i, ∀i = 1, 2, , n Hỏi C có bao nhiêu phần tử? (mỗi phần tử thuộc C là xích độ dài n) Xét xích C ∈ C thì 127 (128) Tạp chí Epsilon, Số 03, 06/2015 • Có n cách chọn cho tập C1 Thật vậy, vì |C1 | = nên C1 ∈ {{1}, {2}, , {n}} • Với cách chọn tập C1 , có n − cách chọn tập C2 Thật vậy, minh họa cho C1 = {3}, đó C2 ∈ {{3, 1}, {3, 2}, {3, 4}, , {3, n}} • Cứ tiếp tục vậy, ta có n − cách chọn tập C3 , n − cách chọn tập C4 , và cuối cùng là cách chọn tập Cn vì Cn = [n] Vậy C có n! phần tử Một tập A ⊂ [n] thì A có thể vừa thuộc vào xích C, vừa thuộc vào đối xích F Do đó ta đếm số cặp N gồm các (A, C) với C ∈ C và A là tập hợp vừa thuộc xích C vừa thuộc đối xích F • Với C ∈ C, có nhiều tập A mà A ∈ C và A ∈ F Thật vậy, giả sử có hai tập A1 , A2 vừa thuộc vào C, vừa thuộc vào F Vì A1 , A2 thuộc xích C nên A1 ⊂ A2 A2 ⊂ A1 , đó thì A1 , A2 không thể thuộc vào F được, vì các tập F không có tập nào là tập tập khác Vậy |N | ≤ n! • Với tập A ∈ F mà |A| = k Khi đó có k! cách chọn các tập A1 , A2 , , Ak−1 (cách đếm giống hệ ý 1) mà A1 ⊂ A2 ⊂ ⊂ Ak−1 ⊂ A, |Ai | = i, ∀i = 1, 2, , k − và có (n − k)! cách chọn các tập Ak+1 , An mà A ⊂ Ak+1 ⊂ ⊂ An , |Aj | = j, ∀j = k + 1, k + 2, , n Suy A ∈ F ta có k!(n − k)! = |A|!.(n − |A|)! cách chọn xích C chứa A Do đó X |N | = |A|!(n − |A|)! A∈F 128 (129) Tạp chí Epsilon, Số 03, 06/2015 Từ hai cách đếm trên, ta có X A∈F |A|!(n − |A|)! ≤ n! ⇒ X |A|!(n − |A|)! ≤1 n! A∈F đây chính là điều phải chứng minh Nhận xét Bất đẳng thức LYM có thể dễ dàng suy từ định lý Bollobas, định lý 1.4 cách, đặt F = {A1 , A2 , , Am } và đặt Bi = [n]\Ai Khi đó điều kiện Ai ∩ Bi = ∅ thỏa mãn và điều kiện Ai ∩Bj 6= ∅ trở thành Ai 6⊆ Aj , tức là điều kiện phản xích Chú ý là bi = n − Đến đây thì m  −1 X n i=1 = −1 n  X + b i i=1 ≤ Định lý 1.8 Một họ F các tập tập n phần tử X gọi là không so sánh A, B là hai phần tử F thì A 6⊆ B Định lý 1.9 (Định lý Sperner) Cho F là họ không so sánh các tập tập n phần tử X Khi đó [n] |F| ≤ Cn Chứng minh Mặt khác, theo tính chất nhị thức Newton thì hệ số khai triển (1 + x)n thỏa tính chất           n n n n n   < n < < < < n−1 2 Áp dụng vào ta có     n n < n kj Thay vào (*) ta có đánh giá m n [ ] n ≤ m X j=1 n kj  ≤ Từ đây ta có điều phải chứng h n iminh Dấu xảy B chứa tất các tập gồm có phần tử tập B 129 (130) Tạp chí Epsilon, Số 03, 06/2015 Hệ 1.10 (Bất đẳng thức Lubell) Cho F là họ không so sánh các tập tập n phần tử X Gọi ak là số các k_tập thuộc F Khi đó n X ak ≤ k C n k=1 Một số phương pháp giải toán cực trị tập hợp 2.1 Khoảng cách Hamming - chặn Plotkin Cho n là số nguyên dương, ký hiệu [0, 1]n = {x1 x2 xn |xi ∈ {0, 1}, i = 1, 2, , n} là tập tất các xâu nhị phân độ dài n Định nghĩa 2.1 Cho hai xâu nhị phân x = x1 xn và y = y1 yn Khi đó khoảng cách hai xâu x và y d(x, y) = số vị trí i mà xi 6= yi Khoảng cách này gọi là khoảng cách Hamming thỏa tất các điều kiện • d(x, x) = 0, ∀x ∈ [0, 1]n ; • d(x, y) > 0, ∀x 6= y ∈ [0, 1]n ; • d(x, y) = d(y, x), ∀x, y ∈ [0, 1]n ; • d(x, z) ≤ d(x, y) + d(y, z), ∀x, y, z ∈ [0, 1]n Định nghĩa 2.2 Cho d là số nguyên dương Gọi C là tập hợp tất các các xâu nhị phân [0, 1]n cho d(x, y) ≥ d, ∀x 6= y ∈ C Định lý 2.3 (PLOTKIN) Cho n, d là các số nguyên dương Đặt M = |C| Khi đó 130 (131) Tạp chí Epsilon, Số 03, 06/2015 Nếu d chẵn, 2d > n thì  d M ≤2 2d − n  Nếu d lẻ, 2d + > n thì   d+1 M ≤2 2d + − n Để chứng minh định lý, ta sử dụng số nhận xét sau: Bổ đề 2.4 Giả sử N, M là các số nguyên dương và ≤ N ≤ M thì ( M M chẵn N (M − N ) ≤ M42 −1 M lẻ Bổ đề 2.5 Nếu x ∈ R thì [2x] ≤ 2[x] + Bổ đề 2.6 Cho v là xâu nhị phân ∈ [0, 1]n Khi đó d(v + w) với số số xuất v + w Chứng minh Bằng việc kiểm tra tất các khả vi và wi , ta thấy ( vi = wi (v + w)i = vi 6= wi Do đó d(v, w) = |{i|vi 6= wi }| = |{i|(v + w)i = 1}|  Chứng minh định lý với d chẵn Ta viết ma trận A = M2 × n, với dòng là phần tử u + v, với u, v ∈ C Ta đếm số lần xuất số ma trận trên Ta đếm số lần xuất số cột Xét cột j Lấy dòng tùy ý, giả sử dòng đó là xâu nhị phân u + w, đó số xuất cột j và v và w có số vị trí j, xâu còn lại có số vị trí j Gọi N là số xâu ký tự C có số vị trí j, đó số cách chọn 131 (132) Tạp chí Epsilon, Số 03, 06/2015 cặp u, w cho v + w có số vị trí j là N (M − N ) Khi đó số số xuất cột thứ j là ( M M chẵn N (M − N ) ≤ M42 −1 M lẻ Điều này đúng cho j = 1, 2, , n Do đó số số xuất ma trận A là ( M chẵn n M4 M −1 n M lẻ Ta đếm số lần xuất số dòng Với dòng chứau + w Ta đã biết số số dòng đó là d(v, w) Mà theo giả thiết thì d(v, w) ≥ d Do đó có ít d số dòng Do đó số các số A ít là   M · d Từ đây ta thấy Nếu M chẵn Khi đó kết hợp hai kết trên thì   M M2 ≤n· dM dM M2 ⇒ − ≤n· 2 ⇒ (2d − n)M ≤ 2dM Theo giả thiết thì 2d − n và M là các số nguyên dương, nên 2d M≤ 2d − n Lại vì M là số nguyên dương nên     2d d M≤ ≤2 + 1; 2d − n 2d − n  d  mà M chẵn, và 2d−n + lẻ nên   d M ≤2 2d − n 132 (133) Tạp chí Epsilon, Số 03, 06/2015 Nếu M lẻ, thì   M M +1 M M2 − ⇒d· ≤n d· ≤n 4 Do đó (2d − n)M ≤ n nên M≤ n 2d = − 2d − n 2d − n Do M là số nguyên, ta   2d M≤ −1 2d − n   2d = −1 2d − n   d ≤2 +1−1 2d − n   d , =2 2d − n tức chúng ta đã có chặn Plotkin cho d chẵn Ví dụ 2.7 (Vĩnh Phúc 2012) Có em học sinh tham gia vào m nhóm hoạt động ngoại khóa, học sinh có thể tham gia nhiều nhóm hoạt động Biết với hai nhóm tùy ý thì có ít học sinh tham gia vào hai nhóm đó Tìm giá trị lớn có thể có m Chứng minh Gọi bảy học sinh là a1 , a2 , , a7 và đặt X là tập hợp các học sinh Ta nhóm là xâu nhị phân x x x7 , đó xi = nhóm đó chứa và xi = nhóm đó không chứa Xét hai nhóm A, B tùy ý số m nhóm, đó có ít học sinh, giả sử là a1 , a2 , a3 , a4 Gọi hai xâu biểu diễn cho A, B là v, w • Nếu {a1 , a2 , a3 , a4 } ∈ A thì 6∈ B, ∀i = 1, 2, 3, Khi đó hai xâu có dạng 133 (134) Tạp chí Epsilon, Số 03, 06/2015 Khi đó d(v, w) ≥ • Nếu a1 ∈ A, {a2 , a3 , a4 } ∈ B Khi đó hai xâu có dạng Khi đó d(u, v) ≥ Xét tất các trường hợp tương tự, ta luôn có d(v, w) ≥ với v, w Vậy d = 4, n = Khi đó theo định lý 3.3 thì   m≤2 = 2.4 − Khi đó m ≤ Với m = ta có cách xây dựng các nhóm là, với a, b, c, d, e, f, h là học sinh A1 : a A2 : a A3 : a b c d f A4 : b A5 : b d c A7 : c a b c d d g f e A6 : A8 : e g f e f e f g Ví dụ 2.8 (China TST 1994) Cho A1 , A2 , , Ak là các tập X = {1, 2, , 10} cho ( |Ai | = 5, ∀i = 1, 2, , k |Ai ∩ Aj | ≤ 2, ∀1 ≤ i < j ≤ k Tìm giá trị lớn có thể có k 134 (135) Tạp chí Epsilon, Số 03, 06/2015 Chứng minh Ta coi tập At là xâu nhị phân dạng x1 x10 , đó xi = i ∈ At và xi = i 6∈ At Do |Ai ∩ Aj | ≤ 2, ∀1 ≤ i < j ≤ k nên suy d(x, y) ≥ (Giả sử xâu v biểu diễn cho Ai , xâu w biểu diễn cho Aj Nếu |Ai ∩ Aj | = thì có hai xâu v, w có đúng hai số trùng vị trí Như hình bên minh họa cho hai số ngoài cùng bên trái Đối với xâu v còn số 1, tương ứng vị trí đó xâu w phải là số và ngược lại xâu w còn số 1, tương ứng ví trí đó xâu v phải là số Trong trường hợp này là d(v, w) = Nếu |Ai ∩ Aj | = hay Ai ∩ Aj = ∅ thì rõ ràng d(x, y) > 6.) Theo định lý 3.3 thì   = k≤2 2.6 − 10 Vậy k lớn 6, và đây là cách xây dựng các tập A1 : A2 : 3 A3 : A4 : A6 : 4 A5 : 8 6 9 10 10 10 Cách xây dựng tập hợp trên không phải là Dưới đây là tập hợp thỏa mãn 135 (136) Tạp chí Epsilon, Số 03, 06/2015 A1 : A2 : A3 : A4 : A5 : A6 : 5 10 10 8 10 Ví dụ 2.9 (PTNK 2012) Cho số nguyên dương n và tập hợp X = {1, 2, 3, , 4n} Hai tập A, B X gọi là không giống |A∆B| ≥ 2n + 1, với A∆B = (A\B) ∪ (B\A) Xét m tập hợp A1 , A2 , , Am là các tập đôi không giống X Chứng minh m≤ 4(n + 1) Chứng minh Ta đồng tập At là xâu nhị phân độ dài 4n x1 x2 x4n với xi = i ∈ At và xi = i 6∈ At Xét hai tập Ai , Aj tùy ý m tập, gọi v, w là hai xâu nhị phân biểu diễn chúng Do |A∆B| ≥ 2n + nên tương tự ví dụ 3.1 thì d(v, w) ≥ 2n + Từ đó suy d = 2n + Do 2.d = 4n + > 4n, nên theo định lý 3.3 thì     2n + + 2n + 4(n + 1) m≤2 =2 ≤ 4n + + − 4n 3 Ví dụ 2.10 (Inspired IMO 1998) Trong thi có n thí sinh và p giám khảo, đó n, p là các số nguyên dương, p > Mỗi giám khảo đánh giá thí sinh và cho kết luận thí sinh đó đỗ hay trượt Giả sử k là số thỏa mãn điều kiện: với hai 136 (137) Tạp chí Epsilon, Số 03, 06/2015 giám khảo tùy ý, số thí sinh mà họ cho kết giống nhiều là k Chứng minh k p−2 ≥ n 2(p − 1) Chứng minh Giả sử n thí sinh là S1 , S2 , , Sn Mỗi giám khảo cho tương ứng với xâu nhịn phân độ dài n: x1 x2 xn với xi = thí sinh Si đỗ và xi = thí sinh Si rớt Theo điều kiện bài toán thì d = n − k Xét hai trường hợp Nếu 2(n − k) ≤ n thì k p−2 ≥ > n k 2(p − 1) Nếu 2(n − k) > n, xét hai khả xảy • Nếu n − k chẵn, theo định lý 3.3 thì   2(n − k) n−k n−k = ≤2 p≤2 2(n − k) − n 2(n − k) − n n − 2k Do đó p(n − 2k) ≤ 2n − 2k ⇒ k p−2 ≥ n 2(p − 1) • Nếu n − k lẻ, theo định lý 3.3 thì   n−k+1 n−k+1 2(n − k + 1) p≤2 ≤2 = 2(n − k) + − n 2(n − k) + − n n − 2k + Do đó p(n−2k+1) ≤ 2n−2k+2 ⇒ p−2 n+1 p−2 k ≥ > n 2(p − 1) n 2(p − 1) Bài toán chứng minh hoàn toàn 2.2 Sử dụng nguyên lý tổ hợp Ví dụ 2.11 Gọi A là tập tất các số tự nhiên lẻ không chia hết cho và nhỏ 30 Tìm số k nhỏ cho tập A gồm k phần tử tồn hai số chia hết cho 137 (138) Tạp chí Epsilon, Số 03, 06/2015 Giải Ta có A = {1, 3, 7, 9, 11, 13, 17, 19, 21, 23, 27, 29}, |A| = 12 Xét tập B = {9, 11, 13, 17, 19, 21, 23, 29} Khi đó hai phần tử bất kì thuộc B thì không chia hết cho Từ đó ta suy k ≥ Ta chứng minh k = thỏa đề bài Xét S là tập bất kì A và |S| = Xét ba cặp {21, 7}, {27, 9}, {1, 11} ta thấy cặp là bội Nếu cặp trên có ít cặp thuộc S thì bài toán giải Giả sử ba cặp trên không có cặp nào cùng thuộc S, |S| = nên S phải chứa số cặp và chứa số còn lại Từ đó suy S phải có cặp {3, 9} {3, 27} và cặp này là bội Hay nói cách khác S luôn tồn hai số chia hết cho Vậy k = Nhận xét Mấu chốt bài toán trên là chúng ta phát tập A0 để từ đó ta khẳng định k ≥ và dự đoán k = Để tìm tập A0 , ta liệt kê hết các số A mà không có hai số nào là bội Với bài toán này, việc tìm tập A0 khá đơn giản Ví dụ 2.12 (KMO 1990) Cho n tập hợp A1 , A2 , , An thỏa mãn   |Ai | = 30, ∀i = 1, 2, , n |Ai ∩ Aj | = 1, ∀i 6= j   A1 ∩ A2 ∩ · · · ∩ An = ∅ Chứng minh n < 872 Chứng minh Giả sử n ≥ 872 Xét tập hợp A1 , |A1 | = 30 Do |Ai ∩ A1 | = 1, ∀i = 2, 3, , n nên theo nguyên lý Dirichlet, tồn phần tử a ∈ A1 thuộc vào ít là   hni 872 +1≥ + = 30 30 30 tập hợp Không tính tổng quát, gọi các tập hợp đó là A2 , A3 , , A31 138 (139) Tạp chí Epsilon, Số 03, 06/2015 Vì A1 ∩ A2 ∩ ∩ An = ∅, nên tồn tập B số các tập A32 , , An không chứa phần tử a Xét 31 tập hợp A2 , A3 , , A31 và B với a ∈ Aj , ∀j = 2, 3, , 30 và a 6∈ B • Vì |Aj ∩ B| = 1, ∀j = 2, 3, , 31, các tập Aj chứa a, còn B không chứa a, nên {xj } = Aj ∩ B thì xj ∈ B\{a} • Có 29 phần tử x2 , , x29 tập B\{a}, tử chúng thuộc vào ít tập hợp 30 tập Aj , j ∈ {2, , 31}, nên tồn phần với t ∈ {2, 3, , 29} thuộc vào hai tập hợp Ar , As {2, 3, , 31}) phần tử xt , (r, s ∈ • Khi đó {a, xt } ⊂ Ar ∩ As , mâu thuẫn với giả thiết |Ar ∩ As | = Vậy điều giả sử là sai Bài toán chứng minh Ví dụ 2.13 (China TST1994, Ví dụ 3.1.2) Cho A1 , A2 , , Ak là các tập X = {1, 2, , 10} cho ( |Ai | ≥ 5, ∀i = 1, 2, , k |Ai ∩ Aj | ≤ 2, ∀1 ≤ i < j ≤ k Tìm giá trị lớn có thể có k Chứng minh Trong ví dụ 3.1.2 ta đã giải bài toán này chặn Plotkin, và xây dựng với k = Do đó k ≥ Nếu k ≥ 7, ta trình bày lời giải kết hợp đếm số tập hợp chứa phần tử và nguyên lý bao hàm loại trừ (inclusion - exclusion principle) Với i ∈ X, đặt ni = |{j ∈ {1, 2, , k|i ∈ Aj }}| tức đếm số tập hợp A1 , A2 , , Ak chứa phần tử i Khi đó 10 X i=1 ni = k X j=1 |Ak ≥ 5k = 35 Do P đó phải tồn số i0 cho ni0 ≥ 4, vì tất ni ≤ thì 10 i=1 ni ≤ × 10 = 30, mâu thuẫn với đánh giá trên Tức là tồn phần tử i0 X thuộc vào ít tập hợp k 139 (140) Tạp chí Epsilon, Số 03, 06/2015 tập A1 , A2 , , Ak Không tính tổng quát, giả sử i0 thuộc vào tập hợp A1 , A2 , A3 , A4 Khi đó với ≤ i < j < t ≤ thì |Ai ∩ Aj ∩ At | ≥ |A1 ∩ A2 ∩ A3 ∩ A4 | ≥ Theo nguyên lý IE thì 10 = |X| ≥ |A1 ∪ A2 ∪ A3 ∪ A4 | = X i=1 |Ai | − X 1≤i<j≤4 |Ai ∩ Aj | + X 1≤i<j<t≤4 |Ai ∩ Aj ∩ At |− |A1 ∩ A2 ∩ A3 ∩ A4 |      4 ≥4×5− ×2+ − × = 11, mâu thuẫn Do đó điều giả sử là sai Suy k ≤ Vậy k = Ví dụ 2.14 (India Postal Coaching 2014 Set Problem 4, Ví dụ 3.3.3) Tập M viết dạng M = A1 ∪ A2 ∪ ∪ An và Ai ∩ Aj = ∅ với ≤ i < j ≤ n, thì các tập A1 , A2 , , An gọi là n_phân hoạch M Giả sử A1 , A2 , , An và B1 , B2 , , Bn là hai n_phân hoạch M thỏa mãn |Ai ∪ Bj | ≥ n, ∀1 ≤ i, j ≤ n n2 Dưới đây ta trình bày lời giải khác cách cùng công thức IE Chứng minh |M | ≥ Chứng minh Đặt k = min{|Ai |i = 1, 2, , n} Không tính tổng quát, giả sử |A1 | = k Khi đó |M | = |A1 ∪ A2 ∪ ∪ An | = |A1 | + · · · + |An | ≥ n|A1 | ⇒ |A1 | ≤ M n Tương tự đặt j = min{|Bi |i = 1, 2, , n} Không tính tổng quát, giả sử |B1 | = j và tương tự đánh giá trên thì M n Theo giả thiết bài toán thì |A1 ∪ B1 | ≥ n nên theo công thức IE thì n2 2|M | n ≤ |A1 ∪ B1 | = |A1 | + |B1 | − |A1 ∩ B1 | ≤ |A1 | + |B1 | ≤ ⇒M ≥ n |B1 | ≤ 140 (141) Tạp chí Epsilon, Số 03, 06/2015 Ví dụ 2.15 Cho 2005 tập hợp, tập hợp có có 44 phần tử Biết hai tập hợp có đúng phần tử chung Chứng minh tồn phần tử thuộc tất 2005 tập hợp đã cho Ta cần khẳng định tồn phần tử thuộc tất 2005 tập hợp đã cho Do đó ta làm việc với phần tử x thuộc nhiều tập hợp (vì có phần tử này có cơi hội nhiều thỏa đề bài) Để làm điều này, ta phải biết x thuộc bao nhiêu tập hợp Nếu x thuộc ít 2005 tập hợp, lập luận dẫn đến vô lý Giải Ta có 2005 tập hợp, tập có 44 phần tử nên có tối đa là 2005 × 44 phần tử Với x là phần tử chúng, đặt Ax = {số tập hợp 2005 tập hợp mà chứa phần tử x} ∈ N Vì {Ax }x ⊂ N là hữu hạn nên tồn phần tử lớn chúng, mà ta giả sử luôn là x Tức x là phần tử thuộc nhiều tập hợp nhất, gọi các tập hợp này là A1 , A2 , , Ak Nếu k = 2005 thì bài toán chứng minh Giả sử k < 2005, tức tồn tập hợp B, số 2005 tập hợp, không chứa x Theo giả thiết bài toán x1 = B ∩ A1 6= x, x2 = B ∩ A2 6= x, dễ thấy x1 6= x2 vì không thì x1 = x2 ∈ A1 ∩ A2 , đó theo điều kiện bài toán thì A1 ∩ A2 = x, Tương tự ta có B chứa các phần tử x1 , x2 , , xk Tuy nhiên B chứa đúng 44 phần tử nên k ≤ 44 Vì phần tử B không thuộc quá 44 tập hợp (vì phần tử x thuộc nhiều tập hợp là k tập hợp mà k ≤ 44) nên B có giao khác rỗng với nhiều là 442 = 1936 < 2004 tập hợp, mâu thuẫn với giả thiết B có giao khác rỗng với 2004 tập hợp còn lại Vậy điều giả sử là sai, tức k = 2005 Bài toán giải xong 141 (142) Tạp chí Epsilon, Số 03, 06/2015 Ví dụ 2.16 Cho 2014 thùng trái cây, giỏ trái cây có ba loại trái cây Táo, Xoài, Cam Chứng minh có thể chọn 1008 thùng trái cây, cho tổng số Xoài, tổng số Táo, tổng số Cam 1008 thùng trái cây này lớn nửa tổng số Xoài, tổng số Táo, tổng số Cam 2014 thùng trái cây ban đầu Chứng minh Đánh số A1 , A2 , · · · , A2014 là 2014 thùng trái cây đó Gọi (ai , bi , ci )lần lượt là số táo, xoài, cam thùng Ai Chọn thùng Ai , Aj có số táo và số xoài lớn Nếu i = j thì ta chọn thùng Ai và thùng Gọi A và B là số táo và số xoài lớn 2012 thùng còn lại Ta chứng minh 2012 thùng còn lại, có thể chia vào nhóm I, II (mỗi nhóm có 1006 thùng) cho  P P   − ≤ A  i∈I i∈II i∈I i∈II P P   bi − bi ≤ B  Giả sử a1 ≥ a2 ≥ ≥ a2012 , ta xét các cặp Xi = (a2i−1 , a2i ), (i = 1, 2, , 1006) Với cặp ( a2i−1 ∈ Xi a2i ∈ Xi ta cho cặp đó vào nhóm Khi đó với cách chia thì X X − ≤ a1 + a3 + · · · + a2011 − a2 − a4 − · · · − a2012 i∈I i∈II X X i∈I − i∈II = a1 + (a3 − a2 ) + · · · + (a2011 − a2010 ) − a2012 ≤ a1 ; ≥ a2 + a4 + · · · + a2012 − a1 − a3 − · · · − a2011 = −a1 + (a2 − a3 ) + · · · + (a2010 − a2011 ) + a2012 ≥ −a1 Nên với cách chia thì X i∈I − X i∈II ≤ A Gọi T là cách chia Nếu X i∈I bi − X i∈II 142 bi ≤ B (143) Tạp chí Epsilon, Số 03, 06/2015 thì chọn cách này Nếu X i∈I bi − X bi > B i∈II không tính tổng quát, giả sử X X X X bi > bi ⇒ bi − bi > B i∈I i∈II i∈I i∈II Khi đó tồn j ∈ I, k ∈ II cho bj > bk Khi đó ta việc đổi chỗ (aj , bj ) từ nhóm I sang nhóm II và (ak , bk ) từ nhóm II sang nhóm I P (cách đổi chỗ này không ảnh P hưởng đến điều kiện (1)) Khi đó bi giảm ít 1, còn bi tăng lên ít i∈I i∈I Do đó X i∈I bi − X bi i∈II giảm ít là Nếu sau đổi chỗ mà thỏa điều kiện (2) thì dừng, chưa thỏa thì tiếp tục làm trên đến lúc thỏa vì hiệu trên giảm ngặt trên tập số nguyên dương Vậy ta luôn chia thành nhóm cho X i∈I X i∈I − bi − X i∈II X i∈II ≤ A bi ≤ B Trong hai nhóm I và II, thì tổng số cam nhóm đã xác định, nên ta chọn nhóm nào số số cam nhiều Giả sử nhóm I Khi đó thêm thùng đã lấy cho vào nhóm I, thì số cam lấy lớn nửa và X X X X −A ≤ − ≤ A ⇒ ≤ + A, i∈I i∈II i∈II i∈I tức số Táo thỏa mãn điều kiện Tương tự kiểm tra cho số Xoài Ví dụ 2.17 Cho X là tập hợp hữu hạn, |X| = n và A1 , A2 , , Am là các tập X thỏa mãn |Ai | ≥ 2, ∀i = 1, 2, , n và |Ai ∩ Aj | = 1, ∀1 ≤ i < j ≤ n 143 (144) Tạp chí Epsilon, Số 03, 06/2015 Chứng minh có thể tô các phần tử X hai màu, phần tử tô màu, cho tập Ai chứa các phần tử tô hai màu Chứng minh Giả sử X = {x1 , x2 , , xn } Giả sử kết luận bài toán sai Trong tất các cách tô màu phần tử tập X, chọn cáchtô tập cực đại X, giả sử Y = {x1 , x2 , , xk }(k < n) thỏa mãn bài toán theo nghĩa: thêm phần tử xk+1 vào Y thì không thể tô màu cho xk+1 để thỏa mãn bài toán Điều đó dẫn đến có hai tập, giả sử là Ar , As chứa xk+1 và Ar , As ⊆ {x1 , x2 , , xk , xk+1 } cho • Nếu xk+1 tô màu xanh thì Ar ∩ Y gồm toàn các phần tử màu xanh; • Còn xk+1 tô màu đỏ thì As ∩ Y gồm toàn các phần tử màu đỏ Nhưng đó thì Ar ∩ As = {xk+1 }, mâu thuẫn với giải thiết bài toán Vậy điều giả sử là sai Bài toán chứng minh Ví dụ 2.18 (2000 Hungarian-Israeli) Đặt S = {1, 2, , 2000} Nếu A và B là hai tập S, ta ký hiệu |A| và |B| là số phần tử tập A và tập B tương ứng Giả sử |A|.|B| ≥ 3999 Chứng minh đó hai tập hợp A − A và B − B chứa ít phần tử chung Ký hiệu X − X = {s − t, s ∈ X và t ∈ X, s 6= t} Giải Đặt T = {(a, b)|a ∈ A, b ∈ B} thì |T | = |A|.|B| ≥ 3999 Tập W = {a+b|a ∈ A, b ∈ B} là tập tập {2, 3, , 4000} Nếu W = {2, 3, , 4000} thì vì hai số và 4000 nằm W nên suy hai tập A và B chứa hai số và 2000 Suy hai tập A − A và B − B có phần tử chung là 1999 144 (145) Tạp chí Epsilon, Số 03, 06/2015 Nếu W 6= {2, 3, , 4000} thì W có ít 3999 phần tử Theo nguyên lý Dirichlet, phải tồn hai cặp (a, b) 6= (a0 , b0 ) T để a + b = a0 + b0 =⇒ a − a0 = b − b0 Khi đó hai tập hợp A − A và B − B chứa chung phần tử là a − a0 = b − b0 Ta có điều phải chứng minh 2.3 Thiết lập ánh xạ các tập hợp Ví dụ 2.19 (Italy 2000) Cho X là tập hợp hữu hạn với |X| = n, và đặt AT1 , A2 , , Am là các tập chứa phần tử X thỏa mãn |Ai Aj | ≤√1 với i 6= j Chứng tỏ tồn tập A X chứa ít [ 2n] phần tử mà không nhận tập Ai (i = 1, 2, , m) nào là tập nó Giải Để tập A không chứa tập Ai nào làm tập con, thì lẽ tự nhiên tập A chứa càng ít phần tử càng tốt √ Tuy nhiên đề bài lại yêu cầu |A| ≥ 2n, tức chặn cho |A| Nghĩa là đề bài yêu cầu tồn tập A có số lượng phần tử "tương đối" nhiều Để làm việc với tập tương đối nhiều phần tử, thông thường ta làm trên tập có nhiều phần tử Giả sử A là tập X mà không chứa tập Ai nào, với phần tử lớn Đặt k = |A| • Ý nghĩa sau: tập nào X có số phần tử > k, chứa tập Ai nào đó • Do đó ta khảo cứu tập vi phạm sinh từ A, tập đó tập A thêm phần tử ngoài A, tức thêm vào A phần tử tập X\A Cách Vì |A| chứa k phần tử nên X\A có n − k phần tử Cách Xét x là phần tử X không thuộc A, S x ∈ X\A Theo tính tối đại tập A, thì A {x} không thỏa mãn điều kiện bài toán Nghĩa là tồn số i(x) ∈ {1, 2, , m} cho Ai(x) ⊆ A 145 [ {x} (146) Tạp chí Epsilon, Số 03, 06/2015 S Vì Ai(x) 6⊂ A và Ai(x) ⊆ A {x} nên x ∈ Ai(x) Từ đó suy Ai(x) \{x} ⊂ A Vì tập Ai có ba phần tử nên Ai(x) \{x} có hai phần tử, mà nó lại là tập A, dẫn đến tập \ Lx = A Ai(x) T có đúng hai phần tử Lại theo giả thiết bài toán |Ai Aj | ≤ với i 6= j nên các tập Lx là các tập phân biệt (theo nghĩa, hai tập hợp Lx và Ly , đây x 6= y thuộc X\A thì Lx 6= Ly Vì Lx = Ly , giả sử T Lx = Ly = {a, b} Khi đó {a, b} ⊂ Ax , {a, b} ⊂ Ay , dẫn đến Ax Ay = {a, b}, mâu thuẫn) Từ đó chúng ta đã xác lập đơn ánh từ tập X\A đến tập hợp chứa các tập hai phần tử A Do đó theo tính chất đơn ánh thì   k k2 − k n−k ≤ = 2 √ √ 8n √ −1 + + 8n =⇒ k + k ≥ 2n =⇒ k ≥ ≥ = 2n 2 Ví dụ 2.20 (AMM, E3459) Cho X là tập hợp, |X| = n ≥ 12 và F = {A1 , A2 , , Am } là họ các 4_tập X cho |Ai ∩ Aj | ≥ 2, ∀i 6= j ∈ {1, 2, , m} Chứng minh tồn tập S S, |S| ≥ cho Ai 6⊂ S, ∀i ∈ {1, 2, , m} Chứng minh lập nều √ 6n − Ta gọi tập T X là tập độc Ai 6⊂ T, ∀i ∈ {1, 2, , m} Xét S là tập độc lập có kích thước lớn X, đặt |S| = k Khi đó k ≥ Vì S lớn nhất, nên với phần tử x ∈ X\S, tồn 3_tập f (x) ⊆ S cho f (x) ∪ {x} ∈ F (vì tập B chứa phần tử S mà B ∪ {x} ∈ F thì ta bổ sung x và S và tập S ∪ {x} là tập độc lập, mâu thuẫn với tính lớn S) 146 (147) Tạp chí Epsilon, Số 03, 06/2015 Lấy x1 6= x2 (x1 , x2 thuộc X\S) Theo kết trên, tồn hai tập chứa phần tử f (x1 ), f (x2 ) S cho f (x1 ) ∪ {x1 } ∈ F, f (x2 ) ∪ {x2 } ∈ F Rõ ràng f (x1 ) 6= f (x2 ), vì f (x1 ) ≡ f (x2 ) Khi đó hai tập A1 = f (x1 ) ∪ {x1 }, A2 = f (x2 ) ∪ {x2 } có |A1 ∩ A2 | = |f (x1 )| = mâu thuẫn giả thiết Từ trên ta thấy f là đơn ánh từ X\S vào các tập chứa phần tử S Do đó   k |X\S| ≤ |S| ⇔ n − k ≤ Từ đây suy   √ k 6n ≤ + 6k = k − 3k + 8k ≤ k − ⇒ k ≥ 6n + 3 Bài toán chứng minh hoàn toàn Ví dụ 2.21 (India Postal Coaching 2014 Set Problem 4) Tập M viết dạng M = A1 ∪ A2 ∪ ∪ An và Ai ∩ Aj = ∅ với ≤ i < j ≤ n, thì các tập A1 , A2 , , An gọi là n_phân hoạch M Giả sử A1 , A2 , , An và B1 , B2 , , Bn là hai n_phân hoạch M thỏa mãn |Ai ∪ Bj | ≥ n, ∀1 ≤ i, j ≤ n Chứng minh |M | ≥ n2 147 (148) Tạp chí Epsilon, Số 03, 06/2015 Chứng minh Đặt k = min{|Ai |, |Bj |, ≤ i, j ≤ n} Không tính tổng quát, giả sử |A1 | = k Do B1 , B2 , , Bn là các tập phân biệt Và |A1 | = k, nên có nhiều k tập Bj {B1 , B2 , , Bn } mà A1 ∩ Bj 6= ∅ Khi đó tồn ít n − k tập Bj {B1 , B2 , , Bn } mà A1 ∩ Bj = ∅ Giả sử các tập B1 , B2 , , Bn có chính xác m tập Bj mà A1 ∩Bj 6= ∅, đó m ≤ k Không tính tổng quát, gọi m tập đó là B1 , B2 , , Bm Khi đó n−m tập Bm+1 , Bm+2 , , Bn còn lại có A1 ∩ Bj = ∅, ∀j = m + 1, m + 2, , n Vì Bj ∩ A1 = ∅, ∀j = m + 1, m + 2, , n, lại theo giả thiết |Bj ∪ A1 | ≥ n, ∀j = m + 1, m + 2, , n Chứng tỏ |Bj | ≥ n − |A1 | = n − k, ∀j = m + 1, , n Theo định nghĩa k thì |Bj | ≥ |A1 | = k, ∀j = 1, 2, , m Từ đó ta có |M | = |B1 ∪ B2 ∪ ∪ Bn | = |B1 | + · · · + |Bm | + |Bm+1 | + · · · + |Bn | {z } | {z } | m tập n−m tập ≥ m × k + (n − m) × (n − k) = n(n − k) − m(n − 2k) • Nếu n ≥ 2k, thì n − 2k ≥ Do m ≤ k nên n  n2 n2 |M | ≥ n(n − k) − k(n − 2k) = +2 −k ≥ 2 • Nếu n < 2k, đó |M | = |B1 ∪ B2 ∪ ∪ Bn | = |B1 | + · · · + |Bn | ≥ n × k > Bài toán giải hoàn toàn 148 n2 (149) Tạp chí Epsilon, Số 03, 06/2015 Ví dụ 2.22 Cho Ai là các tập hợp hữu hạn, với i = 1, 2, , n Chứng minh X |Ai ∩ Aj | < 1, |Ai |.|Aj | 1≤i<j≤n thì tồn ∈ Ai (i = 1, 2, , n) để 6= aj , ∀1 ≤ i < j ≤ n Chứng minh Đặt S = {1, 2, , n} và T = A1 ∪ A2 ∪ ∪ An Xét ánh xạ f : S õT cho: với i ∈ S thì f (i) ∈ Ai (i = 1, 2, , n) Gọi M là tập hợp tất các ánh xạ Khi đó |M | = |A1 |.|A2 | |An | Ta chứng minh có ít đơn ánh M Nếu f ∈ M mà không là đơn ánh, đó tồn i, j ∈ S, i 6= j cho f (i) = f (j) Với ánh xạ f vậy, thì f (i) = f (j) nhận nhiều là |Ai ∩ Aj | giá trị khác nhau, và f (k) (k 6= i, j) nhận nhiều là |Ak | giá trị khác Do đó với i, j ∈ S(i 6= j), số ánh xạ f mà f (i) = f (j) nhiều là |Ai ∩ Aj | · n Y k=1,k6=i,j |Ak | = |Ai ∩ Aj | |M | |Ai |.|Aj | Từ đó suy số ánh xạ M mà không phải là đơn ánh nhiều là X |Ai ∩ Aj | M < |M | |Ai |.|Aj | 1≤i<j≤n Từ đó suy tồn ít đơn ánh f0 ∈ M Đặt f0 (i) = (i = 1, 2, , n) Khi đó ∈ Ai (i = 1, 2, , n) và f0 đơn ánh nên i, j ∈ S(i 6= j) thì = f0 (i) 6= f0 (j) = aj Ví dụ 2.23 (Iran 1999) Cho n là số nguyên dương và tập hợp X = {1, 2, , n} Các tập A1 , A2 , , Ak là các tập X cho với ≤ i1 , i2 , i3 , i4 ≤ n ta có |Ai1 ∪ Ai2 ∪ Ai3 ∪ Ai4 | ≤ n − Chứng minh k ≤ 2n−2 149 (150) Tạp chí Epsilon, Số 03, 06/2015 Chứng minh Một tập T ⊂ X gọi là 2−phủ T ⊆ Ai ∪ Aj với i, j thuộc {1, 2, , k} (i, j không thiết phân biệt) Trong số tất các tập X không bị 2−phủ, ta chọn tập có số lượng phần tử nhỏ Gọi đó là tập A Xét họ tập hợp S1 = {A ∩ A1 , A ∩ A2 , , A ∩ Ak } (ở đây A ∩ Ai có thể trùng A ∩ Aj , xảy điều này ta bỏ khỏi tập S1 tập trùng nhau) Vì A không phải 2−phủ nên X ∈ S1 thì A\X 6∈ S1 (thật vậy, giả sử X và A\X ∈ S1 , đó X = A ∩ Ar , A\X = A ∩ As với r, s thuộc {1, 2, , k} Nhưng đó thì A = X∪(A\X) = (A∩Ar )∪(A∩As ) = A∩(Ar ∪As ) ⇒ A ⊆ Ar ∪As , mâu thuẫn với A không là 2−phủ.) Như có không quá nửa số tập A nằm S1 Do đó |S1 | ≤ 2|A|−1 Bây lấy B = X\A (dĩ nhiên B là 2−phủ, vì |B| > |A|), lại xét tiếp tập hợp S2 = {B ∩ A1 , B ∩ A2 , , B ∩ Ak } Khi đó X ∈ S2 thì B\X 6∈ S2 Thật vậy, giả sử hai tập X và B\X nằm S2 Khi đó X = B ∩ Ap B\X = B ∩ Aq thì B ⊆ Ap ∪ Aq Theo tính nhỏ |A|, suy tập nào có < |A| phần tử là 2−phủ Do đó với m ∈ A thì A\{m} = Ac ∪ Ad với c, d ∈ {1, 2, , k} Khi đó |Ac ∪ Ad ∪ Ap ∪ Aq | ≥ |B ∪ (A\{m})| = |X\{m}| = n − 1, mâu thuẫn với giả thiết Từ đây, tương tự lập luận trên, suy |S2 | ≤ 2|B|−1 = 2n−|A|−1 150 (151) Tạp chí Epsilon, Số 03, 06/2015 Mặt khác, tập Ai xác định (B∩Ai )∪(A∩Ai ) Do đó k ≤ |S1 |.|S2 | ≤ 2|A|−1 2n−|A|−1 = 2n−2 2.4 ĐẾM SỐ TẬP HỢP CHỨA PHẦN TỬ - ĐẾM HAI CÁCH Ví dụ 2.24 (PUTNAM 1980) Cho A1 , A2 , , A1066 là các tập tập hữu hạn X cho |Ai | > |X|, với ≤ i ≤ 1066 Chứng minh tồn 10 phần tử x1 , x2 , , x10 X cho tập Aj (j ∈ 1, 1066) chứa ít phần tử mười phần tử này Chứng minh Bài toán xét |X| ≥ 10 Đặt X = {x1 , x2 , , xm }, với m = |X| Với t (t ∈ {1, 2, , m}), đặt nt = |{j ∈ {1, 2, , 1066}|xt ∈ Aj }| tức nt đếm số tập hợp Aj chứa phần tử xt Khi đó • n1 đếm số tập hợp Aj chứa phần tử x1 ; • n2 đếm số tập hợp Aj chứa phần tử x2 ; • ; • nm đếm số tập hợp Aj chứa phần tử xm Do đó n1 + n2 + · · · + nm = |A1 | + |A2 | + · · · + |A1066 | > 1066 · m = 533m Từ đây suy có số ni , không tính tổng quát giả sử là n1 , mà n1 > 533 Tức là ta có nửa tập hợp số các tập A1 , A2 , , A1066 chứa phần tử x1 Gọi B1 , B2 , , Bs là các tập hợp số các tập Ai mà không chứa phần tử x1 Ta có s = 1066 − n1 ≤ 532 151 (152) Tạp chí Epsilon, Số 03, 06/2015 Đặt Y = X\{x1 } = {x2 , , xm } Ngoài 1 |Bi | = |Ai | > m > (m − 1) = |Y | 2 Với t (t ∈ {2, , 1066}), đặt kt = |{j ∈ {1, 2, , s}|xt ∈ Bj }| tức kt đếm số tập hợp Bj chứa phần tử xt Khi đó k2 + k3 + · · · + km = |B1 | + |B2 | + · · · + |Bs | > s · m−1 s = (m − 1) 2 Vế trái là tổng m−1 số, đó phải tồn ki , không s tính tổng quát giả sử k2 cho k2 > Nghĩa là có nửa tập hợp số B1 , B2 , , Bs chứa phần tử x2 Đến đây lại đặt C1 , C2 , , Cr là các tập hợp số các tập Bj mà không chứa phần tử x2 Ta có r = s − k2 < s − s s 532 = ≤ = 265 2 r (hơn nửa số các tập C1 , , Cr ) chứa phần tử x3 Ta lại r dãy tập hợp D1 , D2 , , Du với u < = 132 không chứa phần tử x4 Tiếp tục quá trình trên, ta lại > Cứ tiếp tục quá trình này lần thứ đến lần thứ 10, dãy không nhiều 65, 32, 15, 7, 3, và Do đó ta nhận các phần tử x1 , x2 , , x10 thỏa mãn yêu cầu bài toán Ví dụ 2.25 (China 1996, Romania 1994) Cho 11 tập hợp Mi , i = 1, 2, , 11 thỏa mãn ( |Mi | = 5, ∀i = 1, 2, , 11 (I) Mi ∩ Mj 6= ∅, ∀1 ≤ i < j ≤ 11 152 (153) Tạp chí Epsilon, Số 03, 06/2015 Gọi m là số nguyên dương lớn cho tồn các tập Mi1 , , Mim số 11 tập trên để m \ k=1 Mik 6= ∅ Tìm giá trị nhỏ m trên tất cách chọn 11 tập hợp Mi thỏa mãn (I) S Chứng minh Đặt X = 11 i=1 Mi Với x ∈ X, đặt nx = |{j ∈ {1, 2, , 11}|x ∈ Mj }| và m = max{nx |x ∈ X} Khi đó X x∈X nx = |M1 | + |M2 | + · · · + |M11 | = 55 Theo giả thiết thì Mi ∩ Mj 6= ∅ với ≤ i < j ≤ 11  • Có 11 cách chọn giao khác rỗng, cách chọn giao khác rỗng là cách chọn cặp tập hợp Mi , Mj số 11 tập M1 , M2 , , M11 ;  • Mặt khác, phần tử x xuất n2x giao khác rỗng các tập Mi , Mj Do đó X nx  x∈X Do đó   11 ≥ = 55 X nx (nx − 1) x∈X ≥ 55 Vì nx ≤ m, ∀x ∈ X, đó 55 ≤ X nx (nx − 1) x∈X ≤ m−1 X m−1 m−1 nx = 55 ⇒ ≥ ⇒ m ≥ x∈X 2 Nếu m = 3, đó dấu phải xảy tất P các bất đẳng thức trên, nên nx = m = 3, ∀x ∈ X Nhưng x∈X nx = 55 không chia hết cho 3, nên nx không thể với giá trị 153 (154) Tạp chí Epsilon, Số 03, 06/2015 x ∈ X Từ đây suy m ≥ Tiếp theo ta chứng minh m = thỏa mãn Xét a e b f b g d h Khi đó 11 tập Mi lấy sau: • tập dòng M1 = {a, b, c, d, D}, M2 = {e, f, g, D}, M3 = {1, 2, 3, 4, D}, M4 = {5, 6, 7, 8, D} • tập cột M5 = {a, e, 1, 5, C}, M6 = {b, f, 2, 6, C}, M7 = {c, g, 3, 7, C}, M8 = {d, h, 4, 8, C} • tập đường chéo M9 = {a, f, 3, 8, D}, M10 = {b, g, 4, 5, D}, M11 = {c, h, 1, 6, D} (Mỗi tập trên lấy trên dòng, cột đúng phần tử) Ví dụ 2.26 (USAMO 2011) Cho A1 , A2 , , A11 là các hợp cho ( |Ai | = 45, ∀1 ≤ i ≤ 11 |Ai ∩ Aj | = 9, ∀1 ≤ i < j ≤ 11 Chứng minh |A1 ∪ A2 ∪ ∪ A11 | ≥ 165 và cho ví dụ với trường hợp dấu xảy Chứng minh Đặt X = A1 ∪ A2 ∪ ∪ A11 , với x ∈ X, đặt nx = |{j ∈ {1, 2, , 11}|x ∈ Aj }| Khi đó X x∈X nx = |A1 | + |A2 | + · · · + |A11 | = 11 × 45 = 495 154 (155) Tạp chí Epsilon, Số 03, 06/2015  • Có 11 cách chọn hai tập giao khác rỗng Ai , Aj số 11 tập A1 , A2 , , A11  • Mặt khác, phần tử x xuất n2x giao khác rỗng các tập Ai , Aj Theo giả thiết thì |Ai ∩ Aj | = với ≤ i < j ≤ 11 nên tập giao hai tập Ai , Aj thì phần tử đếm lặp lần Do đó   X nx  11 =9× = 495 2 x∈X Từ đây suy X nx = x∈X X nx  x∈X ⇒ nx = 3, ∀x ∈ X Mặt khác X nx = x∈X X nx  x∈X = 495 ⇒ X x∈X n2x = × 495 Đặt n = |X|, theo bất đẳng thức Cauchy - Schwarz thì X x∈X 1.nx !2 ≤ X x∈X ! X n2x x∈X ! ⇒ 4952 ≤ n × × 495 ⇒ n ≥ 165 Để kết thúc bài toán, ta thuật xây dựng Coi 11 tập hợp là 11 mặt phẳng R3 , đó không có hai mặt phẳng nào song song với Khi đó ba mặt phẳng tùy ý cắt điểm Tổng số giao điểm đạt là   11 = 165 là các điểm tập X Ví dụ 2.27 (USA TST 2005) Cho n là số nguyên dương lớn Với số nguyên dương m, đặt Xm = {1, 2, , mn} Giả sử tồn họ F = {A1 , A2 , , A2n } gồm 2n tập Xm cho |Ai | = m, ∀i = 1, 2, , 2n; 155 (156) Tạp chí Epsilon, Số 03, 06/2015 |Ai ∩ Aj | ≤ 1, ∀1 ≤ i < j ≤ 2n; Mỗi phần tử Xm nằm đúng hai tập hợp thuộc F Tìm giá trị lớn m theo n Chứng minh Với i ∈ Xmn , đặt ni = |{j ∈ {1, 2, , 2n}|i ∈ Aj }| Khi đó, theo giả thiết thứ thì ni = 2, ∀i = 1, 2, , mn Mặt khác, sử dụng giả thiết thứ thì mn   X ni i=1 Từ đây suy = X 1≤i<j≤mn |Ai ∩ Aj | ≤ X 1= 1≤i<j≤mn   2n n   2n mn ≤ ⇒ m ≤ 2n − Mặt khác, ta lấy 2n đường thẳng R2 , đó không có hai đường thẳng nào song song Tổng số giao điểm các cặp đường thẳng này là   2n = n(2n − 1) = n.m là các phần tử Xmn Khi đó tập Ai chứa 2n − giao điểm trên đường thẳng i Nhận thấy các tập Ai thỏa mãn điều kiện bài toán Vậy giá trị lớn n là 2m − Ví dụ 2.28 Tìm số nguyên dương n lớn cho tồn n tập hợp A1 , A2 , , An thỏa mãn   |Ai | = 4, ∀i = 1, 2, , n |Ai ∩ Aj | = 1, ∀1 ≤ i < j ≤ n   |A1 ∪ A2 ∪ ∪ An | = n Chứng minh Đặt A1 ∪ A2 ∪ ∪ An = {1, 2, , n} Với i ∈ {1, 2, , n}, đặt ni = |j ∈ {1, 2, , n}|i ∈ Aj | 156 (157) Tạp chí Epsilon, Số 03, 06/2015 Khi đó ta có n X i=1 ni = |A1 | + · · · + |An | = 4n (∗) Dẫn đến với i, thì trung bình nó xuất tập hợp Aj Giả sử tồn phần tử, không tính tổng quát, giả sử là nằm nhiều tập hợp Không tính tổng quát, giả sử là A1 , A2 , , A5 • Vì |Ai ∩ Aj | = 1, ∀1 ≤ i < j ≤ và ta luôn có Ai ∩ Aj = {1}, ∀1 ≤ i < j ≤ 5, nên × = 15 phần tử còn lại các tập A1 , A2 , A3 , A4 , A5 phải khác • Ngoài không thể nằm tất các tập hợp A1 , , An , vì không áp dụng lập luận trên thì × n = 3n phần tử còn lại các tập A1 , , An phải phân biệt Nhưng đó thì |A1 ∪ A2 ∪ ∪ An | = 3n + > n, mâu thuẫn • Do đó tồn tập hợp A6 , , An không chứa phần tử Giả sử A6 ∩ Ai (i = 1, 2, , 5) là các tập phân biệt Nếu ngược lại, giả sử A6 ∩ A1 = A6 ∩ A2 = {b} thì b 6= 6∈ A6 Nhưng đó thì b ∈ A1 , b ∈ A2 , mâu thuẫn với ý đầu tiên Từ đây dẫn đến tập A6 phải có ít phần tử, mâu thuẫn với giả thiết Vậy phần tử nằm không quá tập hợp Ngoài có phần tử nào đó nằm ít phần tử, theo (*) phải có phần tử nằm ≤ tập hợp, dẫn đến mâu thuẫn Vậy phần tử xuất đúng tập hợp Aj 157 (158) Tạp chí Epsilon, Số 03, 06/2015 Giả sử nằm A1 , A2 , A3 , A4 Khi đó, giả sử A1 = {1, 2, 3, 4}, A2 = {1, 5, 6, 7}, A3 = {1, 8, 9, 10}, A4 = {1, 11, 12, 13} Khi đó n ≥ 13 Nếu n ≥ 14 thì phần tử 14 nằm tập hợp, giả sử A5 Khi đó phần tử còn lại A5 nằm các tập A1 , A2 , A3 Dẫn đến A5 và A4 không có phần tử chung, mâu thuẫn Vậy n lớn là 13, và đây là 13 tập hợp thỏa mãn A1 = {1, 2, 3, 4}, A2 = {1, 5, 6, 7}, A3 A4 = {1, 11, 12, 13}, A5 = {2, 5, 8, 11}, A7 = {2, 7, 10, 13}, A8 = {3, 5, 10, 12}, A10 = {3, 7, 9, 11}, A11 = {4, 5, 9, 13}, A13 = {4, 7, 8, 12} = {1, 8, 9, 10}, A6 = {2, 6, 9, 12}, A9 = {3, 6, 8, 13}, A12 = {4, 6, 10, 11}, Ví dụ 2.29 (China 1999) Cho n nguyên dương và X là tập hợp với |X| = n Gọi A1 , A2 , , An là các tập X cho |Ai | ≥ 2, ∀i = 1, 2, , n Giả sử với tập A0 ⊂ X, |A0 | = thì tồn số i cho A0 ⊆ Ai Chứng minh Ai ∩ Aj 6= ∅, ∀1 ≤ i < j ≤ n Chứng minh Vì tập con, giả sử {a, b} X thì tồn số i cho {a, b} ⊆ Ai và tập chứa hai phần tử Ai thì không thể là tập tập Aj nào khác Do đó  n  X |Ai | i=1   n = (1) 158 (159) Tạp chí Epsilon, Số 03, 06/2015 Đặt X = {x1 , x2 , , xn } Với i ∈ {1, 2, , n}X, ký hiệu ni = |{j ∈ {1, 2, , n}|xi ∈ Aj }| Khi đó n X ni = i=1 Mặt khác n X i=1 |Ai | (2) • Mỗi phần tử xi ∈ X nằm ni tập hợp Do đó phần tử xi xuất n2i tập giao • Xét tập giao, giả sử {xi , xt } = Ar ∩ As Khi đó xi và xt tính tập giao này Do đó n   X ni i=1 X = 1≤i<j≤n |Ai ∩ Aj | (3) Theo giả thiết bài toán, thì |Ai ∩ Aj | ≤ Và kết luận bài toán cần chứng minh |Ai ∩ Aj | = Khi đó (3) trở thành n   X ni i=1   n = Từ đây theo (1), (2) và đẳng thức trên ta  n  X |Ai | i=1 n   X ni = , i=1 n X ni = i=1 n X i=1 |Ai | dẫn đến kết luận bài toán đưa chứng minh n X i=1 n2i = n X i=1 |Ai |2 (∗) Với phần tử xi ∈ X, xét tập Aj mà xi 6∈ Aj Giả sử Aj = {y1 , y2 , , ys } Khi đó tập hợp chứa phần tử sau đây {xi , y1 }, {xi , y2 }, , {xi , ys } là tập các tập, chẳng hạn A1 , A2 , , As (lưu ý không thể xảy {xi , y1 }, {xi , y2 } cùng thuộc vào tập, 159 (160) Tạp chí Epsilon, Số 03, 06/2015 chẳng hạn A1 Vì đó {y1 , y2 } là tập A1 , mâu thuẫn với giả thiết Điều này đúng cho tất các tập còn lại) Từ đây suy xi thuộc vào s = |Aj | tập hợp A1 , A2 , , As (dĩ nhiên x còn thuộc vào số tập khác nữa) Điều này chứng tỏ xi 6∈ Aj thì ni ≥ |Aj | ⇒ |Aj | ni ≥ n − ni n − |Aj | Số tập hợp Aj không chứa xi là n − ni Do đó X ni ni = (n − ni ) · = ni n − ni n − ni j|xi 6∈Aj Do đó n X ni = i=1 ≥ = = n X X i=1 j|xi 6∈Aj ni n − ni i=1 j|xi 6∈Aj |Aj | n − |Aj | j=1 i|xi 6∈Aj |Aj | n − |Aj | n X X n X X n X j=1 |Aj | Nhưng theo (2) đẳng thức phải xảy ước lượng trên Do đó ni = |Aj | Từ đây suy n n X X X di (n − ni )ni = i=1 i=1 j|xi 6∈Aj = n X X i=1 j|xi 6∈Aj = n X X j=1 i|xi 6∈Aj = n X j=1 |Aj | |Aj | (n − |Aj )|Aj | từ đó (*) chứng minh Bài toán kết thúc 160 (161) Tạp chí Epsilon, Số 03, 06/2015 2.5 Xây dựng - Quy nạp - Truy hồi Ví dụ 2.30 Cho n nguyên dương và tập hợp M = {1, 2, , 20} Gọi A1 , A2 , , An là các tập phân biệt khác rỗng M cho |Ai ∩ Aj | ≤ 2, ∀1 ≤ i < j ≤ n Tìm giá trị lớn n Chứng minh Giả sử A1 , A2 , , An là các tập M thỏa mãn điều kiện bài toán Nếu các tập A1 , A2 , , An có ít phần tử, không tính tổng quát, giả sử |A1 | ≥ Gọi a là phần tử A1 • Khi đó A1 \{a} là tập có ít ba phần tử Do |A1 ∩ Ai | ≤ 2, ∀i = 2, 3, , n nên tập A1 \{a} phải phân biệt với tất các tập A2 , , An (vì A1 \{a} ≡ Ai nào đó, thì A1 \{a} ∩ Ai = A1 \{a} có ít ba phần tử Khi đó A1 ∩ Ai có ít ba phần tử, vô lý.) • Ngoài | (A1 \{a}) ∩ Aj | ≤ |A1 ∩ Aj | ≤ 2, ∀j = 2, 3, , n Từ các kết trên, thay A1 A1 \{a} thì hệ tập hợp A1 \{a}, A2 , A3 , , An thỏa mãn bài toán Hệ này có cùng số tập hợp với hệ ban đầu, số phần tử tập hợp ≤ so với số phần tử tập hợp hệ ban đầu Cứ tiếp tục quá trình này, ta thu dãy tập hợp A∗1 , , A∗n thỏa mãn bài toán và |A∗i | ≤ 3, ∀i = 1, 2, , n Tức là các tập hợp này, tập hợp có tối đa ba phần tử Do đó       20 20 20 n≤ + + = 1350 161 (162) Tạp chí Epsilon, Số 03, 06/2015 Tiếp theo, tổng số các tập chứa phần tử, chứa phần tử, chứa phần tử M là       20 20 20 + + = 1350 và tất các tập này thỏa mãn điều kiện bài toán (rõ ràng hai tập phân biệt các tập này có giao không vượt quá phần tử) Do đó n = 1350 đạt Từ đó suy giá trị lớn n là n = 1350 Ví dụ 2.31 (Indian 2014) Cho số tự nhiên n và X = {1, 2, , n} Với hai tập A và B X, ký hiệu A∆B = {i ∈ X|i ∈ (A\B) ∪ (B\A)} Gọi F là họ các tập X cho với A, B ∈ F thì |A∆B| ≥ Chứng minh |F| ≤ 2n−1 và tìm tất các họ F có 2n−1 phần tử Chứng minh Với tập A ⊂ {1, 2, , n − 1}, thì cặp tập hợp (A, A ∪ {n}), tối đa tập hợp thuộc vào F Thật vì A\(A ∪ {n}) = ∅, (A ∪ {n})\A = {n} ⇒ A∆(A ∪ {n}) = {n} Mặt khác, ta có tối đa 2n−1 cặp tập hợp (A, A ∪ {n}) Do đó tập F có tối đa 2n−1 phần tử Tiếp theo ta chứng minh quy nạp theo n là |F| = 2n−1 thì F chứa tất các tập chứa số lẻ phần tử F chứa tất các tập hợp chứa số lẻ phần tử • Kết hiển nhiên đúng với n = • Giả sử kết đúng đến n = m−1, với m nguyên dương lớn Xét trường hợp với n = m Đặt F1 = {A ∈ F|m ∈ A}, F2 = {A ∈ F|m 6∈ A} Theo giả thiết quy nạp, tập F∈ có nhiều 2m−2 phần tử 162 (163) Tạp chí Epsilon, Số 03, 06/2015 • Với tập hợp A ∈ F1 , ta xét tập hợp A\{m} Khi đó F3 = {A\{m}|A ∈ F1 } Khi đó tập F3 thỏa mãn yêu cầu bài toán và theo giả thiết quy nạp thì lại có |F3 | ≤ 2m−2 Nhưng |F2 | = |F3 | ≤ 2m−2 ⇒ |F| = |F1 |+|F2 | ≤ 2m−2 +2m−2 = 2m−1 Do đó |F| = 2m−1 thì |F1 | = |F2 | = 2m−2 Nhưng đó lại theo giả thiết quy nạp, họ F∈ chứa tất các tập \1, 2, , m−1 với, giả sử, số chẵn phần tử Khi đó F∞ chứa tất các tập {1, 2, , m}, chứa m, dạng A ∪ {m} Tuy nhiên A không thể thuộc vào F2 , vì không thì A, A ∪ {m} thuộc vào F vô lý Khi đó A phải có số lẻ phần tử, A ⊂ {1, 2, , m − 1}, và |F1 | = 2m−2 nên A phải chạy trên tất các tập có số lẻ phần tử Do đó F1 chứa các tập có số chẵn phần tử Vậy F chứa tất các tập hợp có số chẵn phần tử Tương tự F2 chứa tất các tập có số lẻ phần tử Bài toán chứng minh Ví dụ 2.32 (Bankal 2012) Cho n là số nguyên dương Đặt tập hợp Pn = {2n , 2n−1 3, 2n−2 32 , , 3n } Với tập X Pn , đặt SX là tổng tất các phần tử X, đây S∅ = Cho y là số thực thỏa mãn điều kiện ≤ y ≤ 3n+1 − 2n+1 Chứng minh tồn tập Y Pn thỏa mãn điều kiện ≤ y − SY < 2n Giải Ta có SPn = 3n + 3n−1 + · · · + 32 2n−2 + 3.2n−1 + 2n = (3 − 2)(3n + 3n−1 + · · · + 32 2n−2 + 3.2n−1 + 2n ) = 3n+1 − 2n+1 Bằng cách chia cách phần tử Pn cho 2n ta đưa bài toán dạng tương đương sau: Cho n là số nguyên dương, a = , và Qn = {1, a, a2 , , an } Chứng tỏ với giá trị x thỏa mãn ≤ x ≤ + a + a2 + · · · + an , luôn tồn tập X Qn cho ≤ x − SX < Ta chứng minh quy nạp theo n Khi n = thì S∅ = 0, S{1} = 163 (164) Tạp chí Epsilon, Số 03, 06/2015 , S{1,a} = Từ đây kiểm chứng thấy là x là 2 số thực thỏa ≤ x ≤ thì luôn có tập X Q1 thỏa yêu cầu Giả sử kết bài toán đúng đến số nguyên dương n Xét x là số thực với ≤ x ≤ + a + a2 + · · · + an + an+1 + Nếu ≤ x ≤ + a + a2 + · · · + an thì theo giả thiết quy nạp tồn tập X ⊂ Qn ⊂ Qm+1 thỏa ≤ x − SX < an+1 − n , đó vì + Xét với x > + a + a + · · · + a = a−1 1, S{a} = an+1 − 1a − =  an+1 − = an+1 − −1 = an+1 + (an+1 − 2) > an+1 + a2 − = an+1 + > an+1 nên < (x − an+1 ) ≤ + a + a2 + · · · + an Theo giả thiết quy nạp, tồn tập X ⊂ Qn thỏa mãn ≤ (x − an+1 ) − SX < =⇒ ≤ x − SX < với X0 = X [ {an+1 } ⊂ Qn+1 Bài toán chứng minh Ví dụ 2.33 (Romania, grade 9, final round) Cho p, q là hai số nguyên dương, p ≥ 2, q ≥ Một tập hữu hạn X gọi là có tính chất (S) nếu: với cách chọn p tập Bi ⊂ X, i = 1, 2, , p, không thiết phân biệt, tập có q phần tử, thì luôn tồn tập Y ⊂ X có p phần tử, cho |Y ∩ Bi | ≤ 2, ∀i = 1, 2, , p Chứng minh Mọi tập X có pq − q phần tử không có tính chất (S) Mọi tập hợp X có pq − q + phần tử có tính chất (S) 164 (165) Tạp chí Epsilon, Số 03, 06/2015 Chứng minh Nếu X có pq − q = (p − 1)q phần tử, đó ta chọn p − tập, tập có q phần tử (là phân hoạch X) sau B1 = {1, 2, , q}, B2 = {q + 1, q + 2, , 2q}, Bp−1 = {(p − 2)q + 1, , (p − 1)q} ., và tập Bp là tập tùy ý có q phần tử Khi đó, theo nguyên lý Dirichlet, thì với tập Y có p phần tử X, tồn ít tập Bi (i ∈ {1, 2, , p − 1}) cho giao nó với Y có ≥ phần tử Vậy tập X không có tính chất (S) Bây với i cho trước, nhận xét tập [ Bi j6=i có ≤ (p − 1)q phần tử Trong đó tập X có pq − q + = (p − 1)q + phần tử Do đó ta tìm ít phần tử tập X mà nó không nằm tập Bj , với j 6= i • Với i = 1, sử dụng lập luận trên, ta tìm phần tử x1 mà x1 6∈ Bj , ∀j = 2, , p Nếu x1 ∈ B1 , ta tiếp tục qua bước 2, x1 6∈ B1 , ta xây dựng tập B1 cách bỏ khỏi B1 phần tử y1 , và thêm vào tập B1 phần tử x1 Dĩ nhiên phần tử y1 là phần tử X • Tiếp tục với i = 2, lưu ý tập B1 sử dụng bây là tập B1 "mới" Lại sử dụng lập luận trên, ta tìm phần tử x2 mà x2 không nằm tập B1 , B3 , B4 , , Bn Khi đó x2 6= x1 (vì x1 ∈ B1 , x2 6∈ B1 ) Nếu x2 ∈ B2 ta tiếp tục quy trình, x2 6∈ B2 , ta xây dựng tập B2 cách bỏ khỏi B2 phần tử y2 , và thêm vào tập B2 phần tử x2 Cứ tiếp tục quy trình này đến bước p, ta tìm tập Y = {x1 , x2 , , xp } Tập Y giao với tập Bi "mới" chính xác phần tử, đó là xi Bây thay ngược trở lại xi yi ta quay trở lại tập Bi "cũ" Nếu yi 6∈ Y thì Y ∩ Bi = ∅, còn yi ∈ Y thì Y ∩ Bi = {yi } 165 (166) Tạp chí Epsilon, Số 03, 06/2015 Ví dụ 2.34 (China TST 2015) Cho X là tập khác rỗng và A1 , A2 , , An là n tập X cho |Ai | ≤ 3, ∀i = 1, 2, , n; Bất kỳ phần tử nào X nằm ít tập số A1 , A2 , , An   3n tập hợp số các tập Chứng minh có thể chọn A1 , A2 , , An mà hợp chúng X Chứng minh Kết luận bài toán yêu cầu chọn số tập hợp, hợp lại X, đó ta Chọn tập đầu tiên có phần tử, giả sử A1 , |A1 | = Sau đó, ta chọn tiếp tập A2 mà |A2 | = và A2 ∩ A1 = ∅ Sau chọn tập A2 , ta chọn tiếp tập A3 mà |A3 | = và A3 ∩ A1 = ∅, A3 ∩ A2 = ∅, tiếp tục đến không thể chọn thêm tập nào vào hệ Trong tất cách cách lựa chọn các tập hợp A1 , A2 , , An , ta chọn hệ tập hợp S3 cực đại, giả sử S3 = {A1 , A2 , , Ai } (i ≤ n) (tức là họ S3 chứa nhiều tập hợp có thể có) mà |At | = 3, ∀t = 1, 2, , i và Ar ∩ As = ∅, ∀1 ≤ r < s ≤ i (điều này có nghĩa, lần bổ sung tập hợp vào S3 thì tập X3 có số lượng phần tử tăng lên 3) S Đặt X3 = Ar ∈S3 Ar Do tính tối đại tập S3 , nên với tập hợp Aj (j > i) thì |Aj ∩ (X\X3 )| ≤ 166 (167) Tạp chí Epsilon, Số 03, 06/2015 vì không thì ta tiếp tục bổ sung Aj vào tập S3 , mâu thuẫn với tính tối đại S3 Và đó |X3 | = 3i Bây ta tiếp tục chọn họ S2 cực đại chứa các tập Aj còn lại số Ai+1 , , An , cho lần thêm tập hợp vào họ S2 , thì số lượng phần tử hợp chúng tăng lên Không tính tổng quát, giả sử S2 = {Ai+1 , Ai+2 , , Aj } Đặt X2 = [ Ar ∈S2 Ar ∩ (X\X3 ) thì theo cách xác định S2 ta có |X2 | = 2j và theo tính tối đại tập S2 thì |At ∩ (X\(X2 ∪ X3 ))| ≤ 1, ∀t = j + 1, , n Bây ta tiếp tục chọn họ S1 chứa các tập As còn lại số Aj+1 , , An , cho lần thêm tập hợp vào họ S1 , thì số lượng phần tử hợp chúng tăng lên và dĩ nhiên các tập hợp họ S1 chứa hết tất các phần tử X\(X3 ∪ X2 ) = X1 Không tính tổng quát, giả sử S1 = {Aj+1 , Ai+2 , , Ak } 167 (168) Tạp chí Epsilon, Số 03, 06/2015 Khi đó |X| = |X1 | + |X2 | + |X3 | = 3i + 2j + k, X = X1 ∪ X2 ∪ X3 và |S3 | + |S2 | + |S1 | = i + j + k = m Ta cần chứng minh m ≤ 3n • Vì phần tử X1 nằm ít tập hợp, |Ar ∩ X1 | ≤ 1, ∀r = j + 1, , n nên n ≥ i + j + 4k (1) Mỗi phần tử X1 ∪ X2 xuất ít tập hợp, |Ar | ∩ (X1 ∪ X2 ) ≤ 2, ∀r = i + 1, , n nên n≥i+ 4(2j + k) = i + 4j + 2k (2) Mỗi phần tử X xuất ít tập hợp, và |Ar ∩ X| ≤ 3, ∀r = 1, 2, , n, đó n≥ 4(3i + 2j + k) (3) • Lấy 20 × (1) + 12 × (2) + 27 × (3) ta có 59n ≥ 140(i + j + k) = 140m ⇒ m ≤ 59n 3n < 140 Bài toán chứng minh hoàn toàn Ví dụ 2.35 (Romania TST 2006) Cho n là số nguyên dương Một tập S ⊂ {0, 1, 2, , 4n − 1} gọi là tập rời rạc với số k bất kỳ, hai điều kiện sau thỏa mãn T Tập S {4k − 2, 4k − 1, 4k, 4k + 1, 4k + 2} có tối đa phần tử T Tập S {4k + 1, 4k + 2, 4k + 3} có tối đa phần tử Hỏi tập {0, 1, , 4n − 1} có bao nhiêu tập rời rạc? Chứng minh • Ta diễn giải tập S, tập số: gồm số chia hết cho 4, chia dư 1, chia dư 2, chia dư và thêm số chia dư thì tập S chứa tối đa phần tử chúng Trong tập số: chia dư 1, dư 2, dư thì tập S chứa tối đa số Rõ hai số chia cho dư và dư không thể cùng nằm tập S (vì vi phạm điều kiện (ii)) 168 (169) Tạp chí Epsilon, Số 03, 06/2015 • Gọi Sn là tập hợp chứa các tập rời rạc tập {0, 1, 2, , 4n − 1} Rõ ràng là các số có dạng 4k + 1, 4k + bị ràng buộc hai điều kiện Đây chính là sở cho việc xây dựng công thức truy hồi • Gọi Xn là tập hợp chứa các tập Sn mà tập nó chứa phần tử 4n − (tức là các tập Sn mà các tập đó chứa phần tử chia dư 3) • Gọi Yn là tập hợp chứa các tập Sn mà tập nó không chứa hai phần tử 4n − và 4n − (tức là chứa các tập Sn mà tập nó không chứa hai phần tử chia dư và chia dư 3, nghĩa là nó chứa các phần tử chia hết cho chia dư 1) • Gọi Zn là tập hợp chứa các tập Sn mà tập nó chứa phần tử 4n − (tức là chứa các tập Sn mà các tập đó chứa phần tử chia dư 2) Khi đó |Sn | = |Xn | + |Yn | + |Zn | Xét tập hợp Xn+1 xây dựng từ các tập hợp còn lại Xét tập hợp {1, 2, , 4n − 2, 4n − 1, 4n, 4n + 1, 4n + 2, 4n + 3} • Từ tập Xn , đó ta có thể thêm vào tập Xn phần tử 4n không thêm vào, vì thân tập Xn+1 đã chứa phần tử 4n + Vậy |Xn+1 | = |Xn | • Từ tập Yn , ta có thể thêm vào tập Yn phần tử 4n không thêm vào Vì |Xn+1 | = |Yn | • Từ tập Zn , ta có thể thêm vào tập Yn phần tử 4n không thêm vào Vậy |Xn+1 | = |Zn | Từ đó ta có quan hệ truy hồi |Xn+1 | = |Xn | + |Yn | + |Zn | = |Sn | (1) Xét tập hợp Yn+1 xây dựng từ các tập hợp còn lại • Từ tập Yn , đó ta có thể thêm vào tập Yn phần tử 4n 4n + 1, thêm hai không thêm phần tử nào Khi đó |Yn+1 | = |Yn | 169 (170) Tạp chí Epsilon, Số 03, 06/2015 • Từ tập Xn , đó ta có thể thêm vào tập Xn phần tử 4n 4n + không thể thêm đồng thời hai phần tử này, không thêm vào Khi đó |Yn+1 | = |Xn | • Từ tập Zn , đó ta có thể thêm vào tập Zn phần tử 4n 4n + không thể thêm đồng thời hai phần tử này, không thêm vào Khi đó |Yn+1 | = |Zn | Từ đó ta có quan hệ hồi quy |Yn+1 | = |Xn | + |Yn | + |Zn | (2) Tương tự ta có |Zn+1 | = |Xn | + |Yn | + |Zn | = |Sn | (3) Từ ba quan hệ (1), (2), (3) ta có |Sn+1 | = |Sn | Từ đó ta tính |Sn+1 | = 7n |S1 | = 8.7n Bài tập vận dụng Bài tập 3.1 (Olympic KHTN lần 1) Xét tập M = {1, 2, 3, , 10} và A1 , A2 , , An là dãy các tập khác rỗng, phân biệt M cho |Ai ∩ Aj | ≤ 3, ∀1 ≤ i < j ≤ n Tìm giá trị lớn n Đáp số: n = 385 Bài tập 3.2 (Taiwan 1998) Cho n ≥ k ≥ và X = {1, 2, , n} Gọi Fk là họ các k_tập X cho với A, B ∈ Fk thì |A ∩ B| ≤ k − Chứng minh tồn tập Mk X, |Mk | ≥ [log2 n]+1 cho Mk không nhận phần tử nào Fk là tập nó 170 (171) Tạp chí Epsilon, Số 03, 06/2015 Bài tập 3.3 (THTT 444 - Trần Nam Dũng) Cho 100 điểm A1 , A2 , , A100 nằm hình vuông ABCD có cạnh Chứng minh luôn tồn tập X E = {1, 2, , 100} gồm 50 phần tử cho X −−→ X −−→ √ AAi − AAi ≤ i∈X i∈E\X Bài tập 3.4 (China 2006) Cho trước số nguyên dương n ≥ và tập X hữu hạn Gọi B1 , B2 , , Bn là n tập tùy ý X, tập chứa ít hai phần tử Tìm giá trị nhỏ |X| cho tồn tập Y X thỏa mãn hai điều kiện |Y | = n; |Y ∩ Bi | ≤ 1, ∀i = 1, 2, , n Đáp số: |X| = 2n − Bài tập 3.5 (Tuyển tập Olympic 30-4 năm 2007) Cho A1 , A2 , , A10 là các tập hợp thỏa mãn điều kiện |Ai | = 8, ∀i = 1, 2, , 10; |Ai ∩ Aj | = 1, ∀1 ≤ i < j ≤ 10 Chứng minh |A1 ∪ A2 ∪ ∪ A10 | ≥ 39 Bài tập 3.6 (Dan Schwarz) Cho X là tập hợp Gọi n và m ≥ là các số nguyên không âm cho |X| ≥ m(n − 1) + Giả sử B1 , B2 , , Bn là n tập X cho |Bi | ≤ m, ∀i ∈ {1, 2, , n} Chứng minh tồn tập Y X cho |Y | = n và |Y ∩ Bi | ≤ 1, ∀i = 1, 2, , n Bài tập 3.7 (China 2010) Cho số nguyên dương n ≥ và A1 , A2 , , A2n là các tập phân biệt tập {1, 2, , n} Xác định giá trị lớn 2n X |Ai ∩ Aj | i=1 |Ai |.|Aj | 171 (172) Tạp chí Epsilon, Số 03, 06/2015 Đáp số: Bài tập 3.8 (Chọn đội tuyển KHTN 2010) Cho số nguyên dương n > 10 Tìm số nguyên dương m lớn thỏa mãn điều kiện: Tồn m tập Aj tập {1, 2, , 2n}, tập gồm n phần tử cho |Ai ∩ Aj ∩ Ak | ≤ 1, ∀1 ≤ i < j < k ≤ m Đáp số: m = Bài tập 3.9 (China 2014) Với các tập hợp khác rỗng S, T , ta định nghĩa hai tập S + T = {s + t|s ∈ S, t ∈ T }, 2S = {2s|s ∈ S} Cho n là số nguyên dương và A, B là các tập khác rỗng {1, 2, , n} Chứng minh tồn tập D A + B cho D + D ⊆ 2(A + B); |D| ≥ |A|.|B| 2n Bài tập 3.10 Cho n và k là các số nguyên dương cho n > k − k + Cho n tập hợp, tập hợp có k phần tử cho hai tập hợp tùy ý n tập hợp đó có đúng phần tử chung Chứng minh n tập hợp đó có phần tử chung Bài tập 3.11 Cho ke ≥ là số tự nhiên Tìm số tự nhiên nhỏ n cho với tập gồm n số nguyên luôn có số mà tổng hiệu chúng chia hết cho 2k + Bài tập 3.12 Xác định số n lớn cho tồn cho tồn các tập phân biệt S1 , S2 , , Sn thỏa mãn S |Si Sj | ≤ 2006 với ≤ i, j ≤ n S S Si Sj Sk = {1, 2, , 2010} với ≤ i < j < k ≤ n Bài tập 3.13 (VMO 2004) Cho tập A gồm 16 số nguyên dương đầu tiên Hãy tìm số nguyên dương k nhỏ có tính chất: tập có k phần tử A tồn hai số phân biệt a, b cho a2 + b2 là số nguyên tố 172 (173) Tạp chí Epsilon, Số 03, 06/2015 Bài tập 3.14 (Stars of Mathematics 2014) Cho các số nguyên m(m + 1) và A là dương M, m, n thỏa mãn ≤ m ≤ n, ≤ M ≤ tập {1, 2, , n} cho |A| = m Chứng minh tồn tập B ⊆ A cho X 0≤ b − M ≤ n − m b∈B Định lý 3.1 (Erdos) Cho F là họ các tập tập n phần tử X thỏa mãn |F| ≥ và với A ∈ F thì |A| ≥ 2 Với hai phần tử x, y ∈ X tồn A ∈ F để {x, y} ∈ A Khi đó |F| ≥ n Tài liệu tham khảo Một số định lý lý thuyết tập hợp cực trị, Vũ Thế Khôi, Bài giảng Viện Toán Học dành cho trường hè 2012 Combinatorial Mathematics, Stefan H M van Zwam, Princeton University 2013 Khoảng cách Hamming, bài giảng trên mạng Combinatorics of sets, Po-Shen Loh, June 2013 Combinatorial problems in Mathematical Competitions, Yao Zhang, World Scientific 2011 Đề thi học sinh giỏi các nước trên trang mathlinks.ro 173 (174) Tạp chí Epsilon, Số 03, 06/2015 174 (175) MỘT BÀI TOÁN SỐ HỌC HAY VỚI NHIỀU CÁCH GIẢI NGUYỄN DUY LIÊN (THPT Chuyên Vĩnh Phúc) Lời giới thiệu Giải bài toán Số học hay và khó, ta đã cảm thấy thích thú Nhưng bài toán Số học hay và khó mà giải nhiều cách mà từ đó ta có thể giải được, hay tạo số bài toán cùng lớp bài toán đó thì niềm vui còn nhân lên nhiều lần Bài viết này, tôi xin giới thiệu với các bạn cách giải cho bài toán số Số học khá hay và khó kỳ thi Olympic Toán học Quốc tế (IMO) lần thứ 42 Hoa Kỳ Chúng ta cùng bắt đầu với bài toán đó nhé Bài toán Cho các số nguyên dương a, b, c, d với a > b > c > d > Giả sử ac + bd = (b + d + a − c) (b + d − a + c) Chứng minh ab + cd không phải là số nguyên tố Lời giải Giả sử ab + cd là số nguyên tố Ta có ab + cd = (a + d) c + (b − c) a = m · gcd (a + d, b + c) (∗) ( với m là số nguyên dương và gcd (a + d, b − c) là ước số chung lớn a + d và b − c) Từ (∗) suy m = gcd (a + d, b − c) = Trường hợp : m = thì gcd (a + d, b − c) = ab + cd > ab + cd − (a − b + c + d) = (a + d) (c − 1) + (b − c) (a + 1) ≥ gcd (a + d, b − c) điều này dẫn tới vô lý Trường hợp : gcd (a + d, b − c) = Ta có ac + bd = (a + c) b − (b − c) a 175 (176) Tạp chí Epsilon, Số 03, 06/2015 kết hợp với đề bài ac + bd = (b + d + a − c) (b + d − a + c) ta : (a + c) b − (b − c) a = (b + d + a − c) (b + d − a + c) Suy (a + d) (a − c − d) = (b − c) (b + c + d) (∗∗) Từ đẳng thức (∗∗) tồn số nguyên dương k cho: a − c − d = k (b − c) và b + c + d = k (a + d) từ đó suy ra: a + b = k (a + b − c + d) ⇔ k (c − d) = (k − 1) (a + b) kết hợp với a > b > c > d > ta có: • Nếu k = ⇒ c = d vô lý • Nếu k ≥ thì ≥ k a+b = > vô lý k−1 c−d Từ vô lý các trường hợp và 2, nên ab + cd không phải là số nguyên tố Lời giải Theo đề bài ac + bd = (b + d + a − c) (b + d − a + c) biến đổi ta a2 − ac + c2 = b2 + bd + d2 (1) Xét tứ giác ABCD với \ = 600 ; BCD \ = 1200 AB = a, BC = d, CD = b, DA = c ; BAD Rõ ràng tứ giác ABCD tồn ( qua việc dựng hình) [ = α ⇒ ADC \ = 1800 − α Gọi ABC Áp dung định lý hàm số côsin hai tam giác BAD và BCD , ta có \ = b2 + d2 − 2bd cos BCD \ BD2 = a2 + c2 − 2ac cos BAD 176 (177) Tạp chí Epsilon, Số 03, 06/2015 Suy đẳng thức (1) Áp dung định lý hàm số côsin hai tam giác ABC và ACD, ta có AC = a2 + d2 − 2ad cos α = b2 + c2 + 2bc cos α Suy cos α = a2 + d2 − b2 − c2 ad + bc và a2 + d − b − c (ab + cd) (ac + bd) AC = a + d − ad = ad + bc ad + bc Tứ giác ABCD nội tiếp đường tròn, theo định lý Ptôlêmê ta có 2 (AC · BD)2 = (ab + cd)2 suy  (ac + bd) a2 − ac + c2 = (ab + cd) (ad + bc) (2) Từ a > b > c > d > 0, ta suy ab + cd > ac + bd > ad + bc (3) Giả sử ab + cd là số nguyên tố Từ (3) ta thấy hai số ab + cd và ac + bd nguyên tố cùng Cho nên từ đẳng thức (2) ta có ac + bd chia hết ad + bc theo (3) vô lý Nên ab + cd không phải là số nguyên tố Lời giải Từ a > b > c > d > 0, ta suy ab + cd > ac + bd > ad + bc (3) Theo đầu bài ac + bd = (b + d + a − c) (b + d − a + c) nên ta có a2 − ac + c2 = b2 + bd + d2 Do : (4)   (ab + cd) (ad + bc) = ac b2 + bd + d2 + bd a2 − ac + c2 Từ (4) và (5), suy (ab + cd) (ad + bc) = (ac + bd) a2 − ac + c2  (5) (6) Giả sử ab + cd là số nguyên tố Từ (3) ta thấy hai số ab + cd và ac + bd nguyên tố cùng Cho nên từ đẳng thức (6) ta có ac + bd chia hết ad + bc theo (3) vô lý Nên ab + cd không phải là số nguyên tố 177 (178) Tạp chí Epsilon, Số 03, 06/2015 Lời giải Theo đề bài ac + bd = (b + d + a − c) (b + d − a + c) biến đổi ta a2 − ac + c2 = b2 + bd + d2 (7) Giả sử ab + cd là số nguyên tố, đặt ab + cd = p ⇒ ab ≡ −cd (modp) ¯ kết hợp với (7) ta có   = b2 a2 − ac + c2 + b2 b2 + bd + d2 ≡ c2 d2 + bc2 d + b2 c2 + b4 + b3 d + b2 d2   ≡ b2 + c2 b2 + bd + d2 (modp) (8) Từ (8) suy b2 + c2 ≡ (modp) b2 + bd + d2 ≡ (modp) Trường hợp b2 + c2 ≡ (modp) < b2 + c2 < (ab + cd) = 2p ⇒ b2 + c2 = p nên ta suy b2 + c2 = ab + cd ⇔ b (a − b) = c (c − d) dẫn tới ⇒ c (c − d) ≡ (modp) (9) Theo giả thiết ab + cd là số nguyên tố cho nên hai số (b, c) = 1, đó từ (9) suy c − d ≡ (modp) vô lý Trường hợp b2 + bd + d2 ≡ (modp) Điều này tương đương với a2 − ac + c2 ≡ (modp) mà < a2 − ac + c2 < (ab + cd) = 2p nên ta suy a2 − ac + c2 = p = ab + cd đó, ta có ( a2 − ac + c2 = ab + cd b2 + bd + d2 = ab + cd ( c (c − d) = ab + ac − a2 ⇔ d (c − d) = b2 + bd − ab 178 (179) Tạp chí Epsilon, Số 03, 06/2015 ( a| c (c − d) ⇒ b| d (c − d) (10) Mà ab + cd là số nguyên tố cho nên (a , c) = và (b , d) = nên từ (10) suy ( a| c − d b| c − d điều này vô lý Từ vô lý các trường hợp và 2, nên ab + cd không phải là số nguyên tố Lời giải Theo đề bài ac + bd = (b + d + a − c) (b + d − a + c) suy a + b − c + d | ac + bd ⇒ a + b − c + d | ac + bd + a (a + b − c + d) hay a + b − c + d | a2 + bd + ab + ad = (a + b) (a + d) Giả sử (a + b − c + d, a + d) = ⇒ a + b − c + d | a + b Đặt a + b = k (a + b − c + d) (11) với k là số nguyên dương Nếu k = 1, từ (11) ⇒ a + b = a + b − c + d ⇒ c = d vô lý Nếu k ≥ 2, từ (11) ta suy a + b = k (a + b − c + d) ≥ (a + b − c + d) > a + b điều này vô lý a > b > c > d > Vậy (a + b − c + d, a + d) 6= Giả sử có số nguyên tố p cho p | (a + b − c + d , a + d) Ta có  p | a+d p | b−c  a ≡ −d (modp) ⇔ b ≡ c (modp) dẫn tới ab ≡ −cd (modp) ⇔ ab + cd ≡ o (modp) Mà ab + cd > p cho nên ab + cd không phải là số nguyên tố, nó là hợp số Từ cách giải trên các bạn vận dụng vào giải các bài toán tương tự sau đây nhé 179 (180) Tạp chí Epsilon, Số 03, 06/2015 Bài Chứng minh : a2 + ac − c2 = b2 + bd − d2 với các số nguyên dương a > b > c > d > thì ab + cd không phải là số nguyên tố Bài Cho các số nguyên dương a, b, c, d với a > b > c > d > thoả mãn điều kiện a + b − c + d | ac + bd Chứng minh rằng: an bm + cm dn không phải là số nguyên tố ( đó m, n là số nguyên dương và n là số lẻ ) Bài Cho các số nguyên dương a, b, c, d và số nguyên tố p thoả ap + b p Chứng minh rằng: a + b + c + d chia = mãn hệ thức p p c +d p−1 hết cho p Và các bài toán trên liệu có bao nhiêu cách giải các bạn hãy tìm tòi và suy nghĩ cùng tôi 180 (181) ĐỊNH LÝ CARNOT VỀ SỰ ĐỒNG QUY CỦA CÁC ĐƯỜNG VUÔNG GÓC VỚI CÁC CẠNH CỦA TAM GIÁC VÀ ỨNG DỤNG VŨ THANH TÙNG, NGUYỄN CHƯƠNG CHÍ Lời giới thiệu Vừa qua trên Forum "Bài toán hay–Lời giải đẹp–Đam mê toán học" đã diễn thảo luận sôi các thành viên đề tài mà chúng tôi đề cập bài báo này Đầu tiên là bài toán hay giả thiết đưa Đào Thanh Oai Khi đó, giả thiết này chưa có lời giải Bài toán (Đề bài Đào Thanh Oai [1]) Cho tam giác ABC có Ma , Mb , Mc là trung điểm các cạnh BC, CA, AB và Ha , Hb , Hc là chân các đường cao tương ứng với các đỉnh A, B, C Gọi A1 , B1 , C1 là tâm ba đường tròn (AMb Hc ), (BMc Ha ) và (CMa Hb ) Chứng minh ba đường thẳng qua A1 , B1 , C1 và vuông góc với ba cạnh BC, CA, AB đồng quy Bài toán là thử thách không nhỏ có nhiều điểm, nhiều đường Trong đó, phương pháp chứng minh đồng quy lại đa dạng là sử dụng tứ giác nội tiếp, Ceva, Desargues, v.v Tuy vậy, bài toán trên không quá phức tạp Sử dụng định lý Carnot, chúng tôi đã chứng minh bài toán trên cách khá gọn gàng Ngay sau bài toán chứng minh, đã có nhiều nghiên cứu sâu và hướng mở rộng khác Bài toán (Nguyễn Ngọc Giang [1]) Cho tam giác ABC có Ma , Mb , Mc là ba trung điểm các cạnh BC, CA, AB Gọi P là điểm trên mặt phẳng chứa tam giác ABC và có các hình chiếu vuông góc xuống BC, CA, AB là Pa , Pb , Pc Gọi A1 , B1 , C1 là tâm ba đường tròn (AMb Pc ), (BMc Pa ), (CMa Pb ) Chứng minh ba đường thẳng qua A1 , B1 , C1 và vuông góc với ba cạnh BC, CA, AB đồng quy 181 (182) Tạp chí Epsilon, Số 03, 06/2015 Hình 11.1: Giả thiết Đào Thanh Oai Bài toán (Nguyễn Văn Lợi [1]) Cho tam giác ABC và hai điểm P, N trên mặt phẳng Gọi hình chiếu vuông góc P xuống BC, CA, AB là Pa , Pb và Pc và N là Na , Nb và Nc Gọi A1 , B1 , C1 là tâm ba đường tròn (ANb Pc ), (BNc Pa ) và (CNa Pb ) Chứng minh ba đường thẳng qua A1 , B1 , C1 và vuông góc với cạnh BC, CA, AB đồng quy Như ban đầu Đào Thanh Oai dùng trực tâm và tâm ngoại tiếp, đó Nguyễn Ngọc Giang dùng điểm P và tâm ngoại tiếp, còn Nguyễn Văn Lợi dùng hai điểm P và N làm liệu cho giả thuyết mình Điều đáng lưu ý là đây ba giả thuyết đúng và chứng minh cách gọn gàng dùng định lý Carnot là công cụ chính Đến đây câu hỏi đặt ra: Nếu chúng ta bỏ qua điều kiện sáu điểm Na , Nb , Nc , Pa , Pb , Pc là hình chiếu vuông góc hai điểm N, P và để lại điều kiện là các điểm đó nằm trên ba cạnh tam giác thì điều kiện cần và đủ để kết luận bài toán đúng là gì? Chúng tôi đã nghiên cứu lời giải các bài 182 (183) Tạp chí Epsilon, Số 03, 06/2015 Hình 11.2: Hai tam giác trực giao toán đưa trên và đã tìm điều kiện cần và đủ cho sáu điểm Na , Nb , Nc , Pa , Pb , Pc cho kết luận bài toán đúng Bài toán (Bài toán tổng quát [1]) Cho tam giác ABC và sáu điểm Na , Pa ∈ BC, Nb , Pb ∈ CA, Nc , Pc ∈ CA Gọi A1 , B1 , C1 là tâm đường tròn ngoại tiếp các tam giác ANb Pc , BNc Pa và CNa Pb Chứng minh các đường thẳng qua A1 , B1 , C1 và vuông góc với BC, CA, AB đồng quy và các đường trung trực Na Pa , Nb Pb , Nc Pc đồng quy Rõ ràng các bài toán trên là trường hợp đặc biệt bài toán tổng quát vừa giới thiệu Cùng với việc nhắc lại nội dung định lý Carnot chúng tôi đưa khái niệm - đại lượng Carnot, kèm theo đó là số tính chất nhằm trợ giúp cho việc áp dụng định lý này Lời giải bài toán tổng quát đưa đầy đủ Cuối cùng là số bài toán dùng để minh họa cho định lý Carnot và để các bạn tự luyện tập 183 (184) Tạp chí Epsilon, Số 03, 06/2015 Hình 11.3: Bài toán tổng quát Định lý Carnot Xét hai tam giác ABC và A0 B C trên mặt phẳng Các đường thẳng d0A , d0B , d0C qua A0 , B , C và vuông góc với BC, CA, AB Ta định nghĩa đại lượng Carnot tam giác A0 B C tam giác ABC sau: cABC (A0 B C ) = (A0 B − A0 C ) + (B C − B A2 ) + (C A2 − C B ) Đại lượng này đưa vì nó tiện lợi cho việc phát biểu định lý Carnot theo cách Chú ý đại lượng Carnot phụ thuộc vào thứ tự các đỉnh ta xét hai tam giác Nói cách khác cABC (A0 B C ) là khác với cABC (B A0 C ) hay cABC (A0 C B ) Định lý (Định lý Carnot - 1803) : [2] Ba đường thẳng d0A , d0B , d0C qua A0 , B , C và vuông góc với BC, CA, AB đồng quy vả (A0 B − A0 C ) + (B C − B A2 ) + (C A2 − C B ) = 184 (185) Tạp chí Epsilon, Số 03, 06/2015 hay cABC (A0 B C ) = Từ định nghĩa đại lượng Carnot ta thấy: cABC (A0 B C ) + cA0 B C (ABC) = Nhờ định lý Carnot, ta thấy d0A , d0B , d0C đồng quy điểm X thì ba đường thẳng dA , dB , dC qua A, B, C và vuông góc với B C , C A0 , A0 B đồng quy điểm X nào đó Trong hình học, hai tam giác ABC và A0 B C thỏa mãn điều kiện định lý Carnot gọi là trực giao với (orthologic), hai điểm đồng quy gọi là hai tâm trực giao (orthologic centers) X gọi là tâm trực giao tam giác ABC với tam giác A0 B C và X gọi là tâm trực giao tam giác A0 B C với tam giác ABC [3] Sau đây là số tính chất đại lượng Carnot Bổ đề Gọi Ta , Tb , Tc là trung điểm BC, CA, AB và số thực k ∈ R Ta có: −−→ −−→ −→ −−→ −→ −−→ cABC (A0 B C ) = −2.(BC.A0 Ta + CA.B Tb + AB.C Tc ) Nếu A01 A02 ⊥BC, B10 B20 ⊥CA, C10 C20 ⊥AB thì: cABC (A01 B10 C10 ) = cABC (A02 B20 C20 ) Những mệnh đề sau là tương đương: (a) d0A , d0B , d0C đồng quy (b) (A0 B − A0 C ) + (B C − B A2 ) + (C A2 − C B ) = (c) cABC (A0 B C ) = (d) cA0 B C (ABC) = (e) dA , dB , dC đồng quy −−−→ −−−→ −−−→ −−−→ −−−→ −−−→ Nếu A01 A00 = k A01 A02 , B10 B00 = k B10 B20 , C10 C00 = k C10 C20 thì: (a) cABC (A00 B00 C00 ) = k.cABC (A01 B10 C10 ) + (1 − k).cABC (A02 B20 C20 ) (b) Nếu A01 B10 C10 , A02 B20 C20 là trực giao với ABC thì A00 B00 C00 −−−→ −−−→ trực giao với ABC Hơn nữa, X10 X00 = k.X10 X20 với X00 , X10 , X20 là các tâm trực giao các tam giác A00 B00 C00 , A01 B10 C10 , A02 B20 C20 tam giác ABC 185 (186) Tạp chí Epsilon, Số 03, 06/2015 Hình 11.4: Bổ đề 1.4 2.0.0.1 Chứng minh Ta có: −−→ −−→ −−→ −−→ −−→ −−→ A0 B − A0 C = A0 B.(A0 C − BC) − A0 C.(A0 B + BC) −−→ −−→ −−→ = −BC.(A0 B + A0 C) −−→ −−→ = −2.BC.A0 Ta Tương tự vậy, ta có: −→ −−→ B C − B A2 = −2.CA.B Tb ; −→ −−→ C A2 − C B = −2.AB.C Tc Cộng ba đẳng thức trên, ta được: −−→ −−→ −→ −−→ −→ −−→ cABC (A0 B C ) = −2.(BC.A0 Ta + CA.B Tb + AB.C Tc ) Từ phần ta có: −−→ −−−→ −→ −−−→ −→ −−−→ cABC (A01 B10 C10 ) − cABC (A02 B20 C20 ) = −2.(BC.A01 A02 + CA.B10 B20 + AB.C10 C20 ) 186 (187) Tạp chí Epsilon, Số 03, 06/2015 = Trước hết ta thấy rằng: b ⇔ c ⇔ d Ta chứng minh a ⇔ c Gọi H là giao điểm d0B và d0C Khi đó, từ phần ta có: −−→ −−→ −−→ −−→ cABC (A0 B C ) = cABC (A0 HH) = cABC (HHH)−2.BC.A0 H = −2.BC.A0 H −−→ −−→ Suy cABC (A0 B C ) = ⇔ BC.A0 H = ⇔ BC⊥A0 H Như a ⇔ c Tương tự ta có: e ⇔ d a Từ giả thiết ta có: −−−→ −−−→ −−−→0 Ta A0 = k.Ta A01 + (1 − k).Ta A02 ; −−→ −−→ −−→0 Tb B0 = k.Tb B10 + (1 − k).Tb B20 ; −−→ −−→ −−→0 Tc C0 = k.Tc C10 + (1 − k).Tc C20 Từ ba đẳng thức trên và từ phần ta suy hệ thức cần chứng minh −−−→ −−−→ b Gọi X00 là điểm cho: X10 X00 = k.X10 X20 Từ định lý Thales ta có: A01 X10 kA00 X00 kA02 X20 Mặt khác vì A01 X10 ⊥BC nên A00 X00 ⊥BC Tương tự ta có: B00 X00 ⊥CA, C00 X00 ⊥AB Do đó X00 là tâm trực giao tam giác A00 B00 C00 tam giác ABC  Lời giải bài toán tổng quát và bình luận 3.1 Lời giải bài toán tổng quát Gọi Xa , Xb , Xc là trung điểm Na Pa , Nb Pb , Nc Pc Ta chứng minh rằng: cABC (Xa Xb Xc ) = cABC (A1 B1 C1 ) Thật vậy, gọi Ta , Tb , Tc là các điểm cho ATa , BTb , CTc là đường kính các đường tròn (A1 ), (B1 ) và (C1 ) Khi đó A1 , B1 , C1 là trung điểm ATa , BTb và CTc Nhờ bổ đề ta có: cABC (Ta Tb Tc ) + cABC (ABC) = 2.cABC (A1 B1 C1 ) Tuy nhiên ta thấy cABC (ABC) = 0, đó: cABC (Ta Tb Tc ) = 2.cABC (A1 B1 C1 ) Mặt khác theo bổ đề ta lại có: cABC (Ta Tb Tc ) = (Ta B − Ta C ) + (Tb C − Tb A2 ) + (Tc A2 − Tc B ) 187 (188) Tạp chí Epsilon, Số 03, 06/2015 Hình 11.5: Lời giải bài toán tổng quát = (Ta B − Ta A2 ) − (Ta C − Ta A2 ) + (Tb C − Tb B ) −(Tb A2 − Tb B ) + (Tc A2 − Tc C ) − (Tc B − Tc C ) = (BPc2 − APc2 ) − (CNb2 − APc2 ) + (CPa2 − BPa2 ) −(ANc2 − BNc2 ) + (APb2 − CPb2 ) − (CNa2 − BNa2 ) = cABC (Na Nb Nc ) + cABC (Pa Pb Pc ) = 2.cABC (Xa Xb Xc ) Từ đó, ta suy cABC (Xa Xb Xc ) = cABC (A1 B1 C1 ) Áp đụng định lý Carnot ta có: các đường thẳng qua A1 , B1 , C1 vuông góc với BC, CA và AB đồng quy ⇔ cABC (A1 B1 C1 ) = ⇔ cABC (Xa Xb Xc ) = ⇔ các đường thẳng qua Xa , Xb , Xc vuông góc với BC, CA và AB đồng quy ⇔ các đường trung trực Na Pa , Nb Pb , Nc Pc đồng quy 188 (189) Tạp chí Epsilon, Số 03, 06/2015 3.2 Một số trường hợp đặc biệt Trong bài toán trên, tập sáu điểm Na , Pa , Nb , Pb , Nc , Pc thỏa mãn điều kiện bài toán tổng quát có số trường hợp đặc biệt sau • Na , Pa , Nb , Pb , Nc , Pc là giao điểm đường tròn (O) với các cạnh tam giác ABC Khi đó, các đường trung trực Na Pa , Nb Pb , Nc Pc đồng quy O • (Giả thuyết Nguyễn Văn Lợi đã nêu trên [1]) Gọi Na , Nb , Nc là hình chiếu điểm N xuống BC, CA, AB; Pa , Pb , Pc là hình chiếu điểm P xuống BC, CA, AB Khi đó ta dễ dàng nhận thấy các đường trung trực Na Pa , Nb Pb , Nc Pc đồng quy trung điểm N P Một trường hợp đặc biệt (Na , Pa , Nb , Pb , Nc , Pc ) xây dựng bài tập đây Ngoài chúng ta thấy vai trò hai điểm Na và Pa trên BC là ngang nên chúng ta có thể hoán đổi hai điểm này Tương tự, chúng ta có thể hoán đổi hai điểm Nb và Pb , hai điểm Nc và Pc Như vậy, chúng ta có tám ba đường thẳng đồng quy Các bạn hãy thử vẽ hình và tìm tính chất tám điểm đồng quy này nhé! Một số bài toán luyện tập Tiếp theo là số bài toán minh họa cho định lý Carnot và lời giải chúng Bài toán (ví dụ sáu điểm thỏa mãn điều kiện bài toán tổng quát) Cho tam giác ABC, đường tròn qua B, C cắt CA, AB Ab , Ac ; đường tròn qua C, A cắt BC, BA Bc , Ba ; đường tròn qua A, B cắt CA, CB Ca , Cb Chứng minh các đường trung trực các đoạn Ca Ba , Ab Cb , Bc Ac đồng quy Lời giải Gọi Aa , Bb , Cc là trung điểm Ca Ba , Ab Cb , Bc Ac Theo định lý Carnot, chúng ta cần phải chứng minh rằng: (ACc2 − BCc2 ) + (BA2a − CA2a ) + (CBb2 − ABb2 ) = 189 (190) Tạp chí Epsilon, Số 03, 06/2015 Hình 11.6: điểm thỏa mãn điều kiện bài toán tổng quát Ta xét phần biểu thức trên: ACc2 − BCc2 = (ACc + BCc ).(ACc − BCc ) = AB.(ACc − BCc ) Vì ACc = 1/2.(AAc + ABc ) = 1/2.(AAc + AB − BBc ) BCc = 1/2(BBc + AB − AAc ) Do ACc2 − BCc2 = AB(AAc − BBc ) Tương tự trên: BA2a − CA2a = BC(BBa − CCa ); CBb2 − ABb2 = CA(CCb − AAb ) Do đó ta cần phải chứng minh: AB.AAc + BC.BBa + CA.CCb = BA.BBc + CB.CCa + AC.AAb 190 (191) Tạp chí Epsilon, Số 03, 06/2015 Vì bốn điểm B, C, Ac , Ab đồng viên nên AB.AAc = AC.AAb Tương tự, BC.BBa = BA.BBc và CA.CCb = CB.CCa Như biểu thức trên đúng Từ định lý Carnot, ta suy điều phải chứng minh Bài toán (Định nghĩa orthopole (Soons-1886) [4]) Cho tam giác ABC và đường thẳng d trên mặt phẳng Gọi A0 , B , C là hình chiếu A, B, C trên d Chứng minh các đường thẳng qua A0 , B , C và vuông góc với BC, CA, AB đồng quy P (điểm đồng quy P gọi là orthopole đường thẳng d tam giác ABC) Hình 11.7: Orthopole Lời giải Ta có: cABC (A0 B C) = (A0 B − A0 C ) + (B C − B A2 ) + (C A2 − C B ) = (B B + A0 B 02 − C C − A0 C 02 ) + (C C + B C 02 − A0 A2 − A0 B 02 ) +(A0 A2 + A0 C 02 − B B − B C 02 ) = 191 (192) Tạp chí Epsilon, Số 03, 06/2015 Từ định lý Carnot, ta có điều phải chứng minh Bài toán (Hai tam giác trực giao hoàn toàn (định lý Pantazi) [3]) Giả sử tam giác ABC trực giao với tam giác A0 B C và B C A0 Chứng minh tam giác ABC trực giao với tam giác C A0 B Lời giải Từ định nghĩa của đại lượng Carnot ta dễ dàng chứng minh được: cABC (A0 B C ) + cABC (B C A0 ) + cABC (C A0 B ) = Từ đó theo định lý Carnot: cABC (A0 B C ) = 0; cABC (B C A0 ) = Do đó ta suy cABC (C A0 B ) = Theo định lý Carnot, ta có điều phải chứng minh Kết luận Chúng tôi đã giới thiệu định lý Carnot, công cụ hiệu cần chứng minh các đường thẳng vuông góc với ba cạnh tam giác đồng quy Đại lượng Carnot là khái niệm đã đưa vào tiện lợi việc trình bày lời giải các bài toán cách có hệ thống Sử dụng định lý Carnot bài toán tổng quát gần đây xuất đã giải gọn gàng Tài liệu tham khảo [1] Nhóm "Bài toán hay - Lời giải đẹp - Đam mê toán học": www.facebook.com/groups/Loicenter/ [2] Định lý Carnot www.cut-the-knot.org/pythagoras/Carnot shtml [3] Hai tam giác trực giao OrthologicTriangles.html www.mathworld.wolfram.com/ [4] Cực trực giao www.mathworld.wolfram.com/Orthopole.html 192 (193) VỀ MỘT BÀI TOÁN HÌNH HỌC TỪ DIỄN ĐÀN AO PS TRẦN QUANG HÙNG, NGUYỄN BẢO NGỌC (Trường THPT Chuyên KHTN, ĐHKHTN, ĐHQG Hà Nội) Tóm tắt Bài viết này là bài toán trên diễn đàn AoPS và trình bày các ứng dụng nó với các công cụ hình học túy Mở đầu Trên diễn đàn AoPS có bài toán thú vị sau (ký hiệu bài toán đã sửa lại để phù hợp với bài viết) Bài toán Cho tam giác nhọn ABC nội tiếp đường tròn (O) Gọi H là giao điểm hai đường cao BE, CF ; AH cắt (O) D khác A; DE cắt (O) G khác D Chứng minh BG chia đôi EF Lời giải sau là ý tưởng Jeck Lim, nick name là oneplusone trên diễn đàn AoPS G A E M O F H B C K D 193 (194) Tạp chí Epsilon, Số 03, 06/2015 Lời giải Gọi K là giao điểm AH và BC Ta dễ dàng chứng minh K là trung điểm HD và hai tam giác EF B và EHK đồng dạng Gọi M là trung điểm EF , ta có EF BF EF BF = hay = HE HK 2HE 2HK hay FM BF = HE HD Kết hợp với điều kiện ∠BF M = ∠DHE, ta suy hai tam giác BF M và DHE đồng dạng Do đó ∠F M B = ∠HDE = ∠F BG Vậy B, M, G thẳng hàng Ta có điều phải chứng minh Nhận xét Việc dùng kỹ thuật đồng dạng "gấp đôi và chia đôi cạnh" lời giải bài toán trên hay, nó giúp chúng ta có nhiều cách để phát triển và mở rộng bài toán này Các mở rộng Trong bài toán trên E, F chính là các giao điểm đường tròn đường kính BC với các cạnh CA, AB Vậy chúng ta thử thay đường tròn đường kính BC thành đường tròn bất kỳ, ta có bài toán sau: Bài toán Cho tam giác ABC nội tiếp đường tròn (O) Một đường tròn (K) qua B, C cắt AC, AB E, F ; BE giao với CF H; AH cắt (O) P khác A; P E cắt (O) R khác P Chứng minh BR chia đôi EF Lời giải Gọi D là hình chiếu K lên AH Lấy điểm N đối xứng với F qua DK Suy N thuộc (K) Vì ∠BCP = ∠BAP = ∠BF N = ∠BCN nên C, P, N thẳng hàng Gọi AH cắt BC L và Q đối xứng với P qua D Ta có ∠F QA = ∠QF N = ∠F N C = ∠AP C = ∠ABC nên tứ giác BF QL nội tiếp 194 (195) Tạp chí Epsilon, Số 03, 06/2015 R A E I Q H F D B O K C L N P Tương tự trên tứ giác CEQL nội tiếp nên tứ giác AEQF nội tiếp Mặt khác ta dễ dàng chứng minh hai tam giác EF B và EQD đồng dạng Gọi I là trung điểm EF Ta có hai tam giác IF B và EQP đồng dạng Suy ∠F BI = ∠EP Q = ∠ABR Vậy BR qua I chia đôi EF Nhận xét Lời giải trên sử dụng kỹ thuật đồng dạng "gấp đôi và chia đôi cạnh" Tuy nhiên rõ ràng là bài toán mở rộng nó đã dùng khéo léo để vận dụng hết các kiện mở rộng bài toán Trong bài toán gốc, ta có thể coi H nằm trên đường tròn đối xứng với đường tròn ngoại tiếp tam giác ABC qua BC Vậy ta thử thay điểm H thành điểm trên đường tròn này, ta có kết thú vị sau: Bài toán Cho tam giác nhọn ABC nội tiếp đường tròn (O) P thuộc cung nhỏ BC cho Q đối xứng P qua BC thì Q nằm tam giác ABC QB, QC cắt CA, AB E, F ; P E cắt (O) R khác E Chứng minh BR chia đôi EF 195 (196) Tạp chí Epsilon, Số 03, 06/2015 R A I F E O G B Q C D P Lời giải Gọi D là giao điểm đường tròn ngoại tiếp tam giác ABE và BC Ta dễ có tứ giác AEQF nội tiếp nên ∠EQC = ∠BAC = ∠EDC suy tứ giác EQDC nội tiếp Từ đó ∠BAD = ∠BED = ∠QCD = ∠DCP = ∠BAP ta thu A, D, P thẳng hàng Gọi G đối xứng P qua D, từ GQ k BC suy ∠DGQ = ∠GDB = ∠AEB vì tứ giác AEQG nội tiếp Từ đó dễ có ∠EGD = ∠EF B và ∠F BE = ∠GDE Vậy hai tam giác EF B và EDG đồng dạng Gọi I là trung điểm EF suy tam giác IF B và EGP đồng dạng Vậy ta thu ∠F BI = ∠GP E = ∠ABR nên BR qua I chia đôi EF Nhận xét Một lần ta thấy kỹ thuật đồng dạng "gấp đôi và chia đôi cạnh" lại sử dụng cách khéo léo để khai thác các giả thiết mở rộng Hai bài toán trên cho chúng ta hai mở rộng khác cùng bài toán Các cách làm này còn gặp lại nhiều các vấn đề khác 196 (197) Tạp chí Epsilon, Số 03, 06/2015 Một số ứng dụng bài toán gốc và các mở rộng Bài toán gốc và bài toán mở rộng có khá nhiều ứng dụng thú vị, chúng ta hay tìm hiểu qua các bài toán sau Bài toán sau đề xuất [3] Bài toán Cho tam giác ABC có các đường cao AK, BE, CF Gọi D là giao điểm AK và (O); L thuộc KE cho BL ⊥ OA; DE cắt (O) điểm G khác D Chứng minh AL và BG cắt trung điểm EF G A E K F B H L O C K D Lời giải Gọi H là trực tâm tam giác ABC và M là trung điểm EF Theo bài toán thì BG qua M Mặt khác ta có EF ⊥ OA nên BL k EF Vì BG qua trung điểm M EF nên B(EF, GL) = −1 Từ hàng điều hòa ta có E(BA, F K) = −1 nên B(EF, GL) = E(BA, F K) Suy A, M, L thẳng hàng Vậy BL và AG cắt trung điểm EF Ta có điều phải chứng minh Với ý tưởng sử dụng hàng điều hòa ta thu bài toán tổng quát sau 197 (198) Tạp chí Epsilon, Số 03, 06/2015 Bài toán Cho tam giác nhọn, không cân ABC nội tiếp đường tròn (O) Một đường tròn (K) qua B, C cắt đoạn thẳng CA, AB E, F khác C, B; BE giao CF H; AH cắt BC D; AD cắt (O) G khác A; GE cắt (O) N khác G; P thuộc DE cho BP k EF Chứng minh BN và AP cắt trung điểm EF N A E I F O H P K B C D G Lời giải Theo bài toán 4, ta chứng minh BN qua trung điểm I EF Do BP k EF nên chùm B(EF, N P ) = −1 Kết hợp với chùm E(BA, F D) = −1 suy B(EF, N P ) = −1 = E(BA, F D) Suy giao điểm tương ứng là A, I, P thẳng hàng Ta có điều phải chứng minh Bài toán Cho tam giác nhọn ABC có các đường cao AD, BE, CF đồng quy trực tâm H Gọi M, N là trung điểm DE và DF ; BM cắt CN P Chứng minh HP chia đôi EF 198 (199) Tạp chí Epsilon, Số 03, 06/2015 A K S Q E L F H T M O N P B C D Lời giải Gọi (O) là đường tròn ngoại tiếp tam giác ABC Gọi K, L là giao điểm BE và CF với (O) Theo bài trước ta dễ thấy LE và BM cắt S thuộc (O) Tương tự KF và CN cắt T thuộc (O).Gọi Q là giao điểm SL và KT BT L Áp dụng định lý Pascal cho ta suy P, H, Q thẳng CSK hàng Ta dễ dàng chứng minh tứ giác EF LK là hình thang nên theo bổ đề hình thang HQ chia đôi EF Suy P H chia đôi EF Ta có điều phải chứng minh Bài toán Cho tam giác nhọn ABC có đường cao AD, BE, CF đồng quy trực tâm H Trung trực HD cắt DF, DE Q, R; BQ cắt CR P Chứng minh HP chia đôi EF A I E F H M N P R O Q B S D C T 199 (200) Tạp chí Epsilon, Số 03, 06/2015 Lời giải Theo bài trước ta chứng minh BQ qua trung điểm M EF và CR qua trung điểm N DF thì bài toán chứng minh Thật vậy, gọi HQ, HR cắt BC S, T Vì tam giác HSD vuông D và tam giác QHD cân H suy tam giác QSD cân Q Do đó ∠QSD = ∠QDS = ∠EDC nên HQ k DE Mặt khác, dễ thấy Q là trung điểm HS nên BQ qua trung điểm M DE Tương tự CR qua trung điểm N DF Theo bài trước ta có điều phải chứng minh Nhận xét Sử dụng phương pháp chiếu song song ta dễ dàng suy bài toán đúng với điểm P mặt phẳng thay cho H Bài toán tổng quát này có thể giải đơn giản biến đổi diện tích Các bạn hãy quan tâm tới vấn đề này Bài toán Cho tam giác ABC nội tiếp đường tròn (O) và đường cao AD, CF Gọi K là giao điểm AD và (O); KF cắt (O) L khác K; đường thẳng qua A vuông góc OC cắt CL N Chứng minh F N ⊥ F O A L N Q M P F H B O C D K Lời giải Ta chứng minh 4F N A ∼ 4F OC để suy 4F N O ∼ 4F AC Từ đó suy ∠N F O = 90◦ 200 (201) Tạp chí Epsilon, Số 03, 06/2015 Thật vậy, ta dễ dàng chứng minh ∠N AC = ∠ABC = ∠ALC nên ∠N AF = ∠N AC − ∠BAC = ∠ABC − ∠BAC = ∠F CO Gọi AD cắt CF H và P là hình chiếu F lên AD, M đối xứng H qua P Dễ thấy ∠F M H = ∠F HM = ∠ABC = ∠N AC và ∠F KM = ∠ACN Do đó 4KF M ∼ 4CN A Ta có tam giác 4F AL ∼ 4F KB và 4F DP ∼ 4ACF Từ đó ta có biến đổi tỷ số N A LA N C KB HB N C 2OQ AC NA = = = = FA LA F A AC F K AC F K AC M K OQ OC CF BF OC AC OC 2OQ = = = 2DP OC CF DP BC CF F D CF Ta có điều phải chứng minh = Bài toán Cho tam giác nhọn ABC nội tiếp đường tròn (O) với đường cao BE, CF cắt H Gọi D là giao điểm AH và (O); DE cắt (O) M khác D; I là trung điểm EF ; P thuộc CM cho P I ⊥ IO; lấy Q đối xứng P qua I Chứng minh DF và BQ cắt trên (O) M A P N E I F Q O B C D 201 (202) Tạp chí Epsilon, Số 03, 06/2015 Lời giải Theo bài toán thì BM qua I Gọi DF cắt (O) điểm N khác D Vẫn theo bài toán thì CN qua I Trong tứ giác BCM N có BM cắt CN I Mặt khác đường thẳng qua I vuông góc OI cắt CM P Theo bài toán bướm thì đối xứng P qua I là Q thuộc BN Từ đó BQ và DF cắt N thuộc (O) Ta có điều phải chứng minh Bài toán 10 Cho tam giác nhọn ABC nội tiếp đường tròn (O) với đường cao AD Đường tròn (A, AD) cắt (O) M, N với M, B khác phía AC; M N cắt CA, AB E, F ; BE cắt CF H; AH cắt (O) K; KF cắt (O) R khác K; CR cắt M N P Chứng minh 2P Q = |M E − N F | A R P F N Q O E M H B D C K Lời giải Ta có tam giác AM N cân nên ∠N BA = ∠N M A = ∠AN M Từ đó AN = AF.AB Tương tự AM = AE.AC Do AM = AN nên AF.AB = AE.AC Từ đó tứ giác BCEF nội tiếp Theo bài toán thì CR chia đôi EF suy Q là trung điểm EF Mà Q là trung điểm M N nên |M E − N F | = |P M − P N | = |QN − QM + 2P Q| = 2P Q Ta có điều phải chứng minh 202 (203) Tạp chí Epsilon, Số 03, 06/2015 Bài toán 11 Cho tam giác nhọn ABC nội tiếp đường tròn (O) Đường cao AD, BE, CF đồng quy H; CH cắt (O) G khác C; GD cắt (O) K khác G; N là trung điểm DF ; AN cắt (O) L khác A Chứng minh đường tròn ngoại tiếp tam giác KLN chia đôi DE A P O E Q G F N H B M C D L K Lời giải Gọi M là trung điểm DE Ta chứng minh đường tròn ngoại tiếp tam giác KLN qua M Thật vậy, theo bài toán thì AK qua M Gọi BH cắt (O) P khác B Cũng theo bài toán thì LD qua P là đối xứng H qua CA Từ đó suy AG = AH = AP ⇒ GP ⊥ OA ⊥ EF ⇒ GP k EF k M N Gọi AL cắt GP Q Ta có ∠M N A = ∠AQP = ∠AGQ + ∠QAG = ∠AP G + ∠QAG = ∠AKG + ∠GKL = ∠AKL Suy tứ giác M KN L nội tiếp Ta có điều phải chứng minh Bài toán 12 Cho tam giác nhon ABC nội tiếp đường tròn (O) Đường cao AD, BE, CF đồng quy H M, N là trung điểm DE, DF AM, AN cắt (O) P, Q khác A DP, DQ cắt AM, AN S, T Chứng minh ST k M N 203 (204) Tạp chí Epsilon, Số 03, 06/2015 A K E L F H M T N S B C D Q P Lời giải Theo bài toán trên ta đã chứng minh tứ giác P QN M nội tiếp Gọi K, L đối xứng với H qua CA, AB thì K, L thuộc (O) Theo bài toán thì DQ qua K và DP qua L Vì AK = AH = AL nên tam giác AKL cân Từ đó ∠QSP = ∠SAP + ∠SP A = ∠SAP + ∠AKL = ∠SAP + ∠ALK = ∠SAP + ∠AQT = ∠QT P Suy tứ giác P QST nội tiếp Vậy từ hai tứ giác P QN M và P QST nội tiếp, ta dễ dàng suy ST k M N Với kĩ thuật gấp đôi và chia đôi cạnh, ta có thể tìm nhiều bài toán hình học hay Với cách nghĩ đơn giản và cổ điền, hầu hết học sinh có thể nghiên cứu và sử dụng nó Phải chăng, bài toán càng tinh túy phương pháp giải càng sơ cấp? Tài liệu tham khảo [1] Topic Divide in two eqal segments www.artofproblemsolving com/Forum/viewtopic.php?f=46&t=386417 [2] Lim Jeck www.limjeck.com [3] Topic Midpoint www.artofproblemsolving.com/Forum/ viewtopic.php?f=47&t=389720 204 (205) ĐỊNH LÝ BƯỚM KÉP ĐỐI VỚI TỨ GIÁC NGUYỄN NGỌC GIANG (TP HỒ CHÍ MINH) TRỊNH HUY VŨ (THPT CHUYÊN KHTN HÀ NỘI) Tóm tắt Chúng ta khám phá chứng minh định lí bướm đơn và định lí bướm kép cho tứ giác Các kết này là mở rộng các kết [1] tác giả Zvonko Cerin Định lí bướm đơn tứ giác Zvonko Cerin [1] đã chứng minh kết sau gọi là định lí bướm đơn tứ giác Định lý (Định lí bướm đơn tứ giác) Cho A0 B C D0 là tứ giác nội tiếp ABCD Giả sử ABCD và A0 B C D0 cùng chung giao điểm các đường chéo U và V là các giao điểm đường thẳng AC với các đường thẳng D0 A0 và B C Định lí bướm tứ giác, thiết lập AI AU IV = (1) UI V C IC Zevonko Cerin đã mở rộng hệ thức (1) thành định lí tổng quát sau Định lý Gọi A0 B C D0 là tứ giác nội tiếp ABCD E là giao A0 C và B D0 I là giao AC và BD U là giao AC và D0 A0 V là giao AC và B C Nếu E nằm trên đường thẳng AC, thì AU EV AI = (2) UE V C IC 205 (206) Tạp chí Epsilon, Số 03, 06/2015 Cerin chứng minh hệ thức (2) phương pháp tọa độ với trợ giúp phần mềm Maple Cách chứng minh Cerin có ưu điểm là cách chứng minh có tư thuật toán Nhược điểm nó là lời giải dài, tính toán phức tạp Chính vì để khắc phục nhược điểm này, chúng tôi đã nghiên cứu và đưa cách chứng minh túy hình học J C' D H C V F D' A U M I E B' A' B G Chứng minh định lý Đặt B D0 cắt AB F ; A0 C cắt AD G; F G cắt AC H; F G cắt B C J, A0 C cắt BD M Xét ∆AF G và ∆CB C có AC, F B , GC đồng quy E Theo định lí Desargues, suy J, D, B thẳng hàng Nói cách khác là J nằm trên BD Xét tứ giác toàn phần AD0 EA0 F G, ta có (HU, AE) = −1 suy UA HA = HE UE Từ đây ta có (HI, AE) = G(HI, AE) = (JI, DM ) = C (JI, DM ) = (V I, CE) Suy HA IE V C IE AU EV AI = suy = HE IA V E IC UE V C IC 206 (207) Tạp chí Epsilon, Số 03, 06/2015 Đây chính là điều phải chứng minh Định lí bướm kép tứ giác Từ định lí 1, chúng tôi nảy sinh ý tưởng mở rộng định lí bướm đơn tứ giác thành định lí bướm kép sau Định lý (Định lí bướm kép tứ giác) Cho tứ giác ABCD Gọi I là giao điểm hai đường chéo AC và BD Qua I dựng các đường thẳng d1 , d2 , d3 ; d01 , d02 , d03 cắt các cạnh AB, BC, CD, DA M, R, G; N, S, H; P, T, F ; Q, L, J Gọi giao điểm RL, GJ; ST, HF với AC là U2 , U3 ; V2 , V3 Gọi giao điểm M U3 với AD là W ; QU2 với AB là X Gọi giao điểm N V3 với DC là Z; P V2 với BC là Y Gọi giao điểm XW với AC là U ; Y Z với AC là V Chứng minh AU IV AI = (3) UI V C IC D F P Q Z W T J L A U3 U V3 U2 V I C V2 X R S M H G N B 207 Y (208) Tạp chí Epsilon, Số 03, 06/2015 Chúng tôi mở rộng định lý thành định lý tổng quát sau Định lý Cho tứ giác ABCD Gọi I là giao điểm hai đường chéo AC và BD Gọi E là điểm bất kì nằm trên AC Qua E dựng các đường thẳng d1 , d2 , d3 ; d01 , d02 , d03 cắt các cạnh AB, BC, CD, DA M, R, G; N, S, H; P, T, F ; Q, L, J Gọi giao điểm RL, GJ; ST, HF với AC là U2 , U3 ; V2 , V3 Gọi giao điểm M U3 với AD là W ; QU2 với AB là X Gọi giao điểm N V3 với DC là Z; P V2 với BC là Y Gọi giao điểm XW với AC là U ; Y Z với AC là V Chứng minh AI AU EV = (4) UE V C IC D F P Q Z T W J L A X1 U U3 U2 V2 I E V3 V X R M S G W1 Y H N B Chứng minh định lý Gọi giao điểm W E và BC là W1 ; XE cắt DC X1 ; X1 W1 cắt Y Z V Áp dụng định lí cho tứ giác GHF J nội tiếp tứ giác ABCD ta có AU3 EV3 AI = , (5) U3 E V3 C IC 208 C (209) Tạp chí Epsilon, Số 03, 06/2015 Áp dụng định lí cho tứ giác M W1 P W nội tiếp tứ giác ABCD và kết hợp với (5), ta suy P, W1 , V3 thẳng hàng Hoàn toàn tương tự ta chứng minh N, X1 , V2 thẳng hàng Áp dụng định lí Papus cho cặp điểm thẳng hàng là P, Z, X1 và N, W1 , Y có P W1 cắt ZN V3 ; ZY cắt X1 W1 V ; P Y cắt X1 N V2 nên V , V2 , V3 thẳng hàng Nói cách khác là V nằm trên AC Do đó V chính là giao điểm ZY với AC Suy V trùng V hay V thuộc X1 W1 Áp dụng định lí cho tứ giác XW1 X1 W nội tiếp tứ giác ABCD ta thu AI AU EV = UE V C IC Đây là điều phải chứng minh Nhận xét Khi E = I thì định lí trở thành định lí Tài liệu tham khảo [1] Zvonko Cerin, On Butterflies inscribed in a quadrilateral, Forum Geom, 6(2006), 241-246 209 (210) Tạp chí Epsilon, Số 03, 06/2015 210 (211) ĐÊM TRƯỚC NHỮNG BẢN THẢO SẮP IN NGUYỄN QUỐC KHÁNH (Hà Nội) Trong bài điểm sách “Toán học mắt ai” đăng trên Epsilon số ngày 13/04/2015, tôi đã nhắc tới câu chuyện có tính “bệnh thành tích”, đó là câu chuyện số lượng giải Fields tính theo quốc gia Trong bài viết đó, tôi đã nói là nhờ có giải Fields năm 2010, mà Việt Nam đã quân bình thành tích với cường quốc toán học là nước Đức Viết vậy, là có ba thiếu sót Thứ nhất, Đức không là cường quốc toán học, mà còn là triết học, âm nhạc, văn chương, công nghệ, kĩ thuật, kinh tế, quân Thứ hai, có số thời kì, toán học nước Đức không là cường quốc, mà còn là đế quốc thực (theo nghĩa đen) Có hai thứ phải kể tới nước Đức đầu kỉ 19, và đầu kỉ 20, là tạp chí Crelle, và Đại học Göttingen Tờ báo (độc lập) nghiên cứu toán học cao cấp “hạng nặng” đầu tiên trên giới có thể tính là tờ Crelle nước Đức Ra đời năm 1826 ông Crelle, không biết cách nào, tạp chí Crelle đã trở thành tượng độc vô nhị lịch sử toán học Ngay số đầu tiên, Crelle đã phát “sự tồn Abel” cách cho đăng tải loạt bài báo nhà toán học trẻ này Trong năm tiếp theo, Crelle tiếp tục trở thành nơi “trình làng” hàng loạt nhà toán học từ kiệt xuất tới kiệt xuất trở lên Rất nhiều nhà toán học kiệt xuất, mà giới chưa có biết đến họ, đã đăng bài báo đầu tay đây Đó là trường hợp Dirichlet, Eisenstein, Grassmann, Hesse, Jacobi, Kummer, Lobachevsky, Möbius, Pücker, Weierstrass, vân vân Ở góc độ khác, Đại học Göttingen “của Klein và Hilbert” đã thừa nhận cách rộng rãi là thánh đường toán học thời kì cuối kỉ 19, đầu kỉ 20 Đây chính là nơi “tầm sư 211 (212) Tạp chí Epsilon, Số 03, 06/2015 Crelle, Crelle’s Journal, Klein, Göttingen, Hilbert: năm bảo vật toán học Đức học đạo” hầu hết các trưởng khoa toán đầu tiên các đại học trên toàn nước Mỹ, nói cách đúng hơn, là nước Mỹ, với mục tiêu xây dựng toán học cao cấp và đại, là động lực để hướng tới phát triển quốc gia, đã gửi hàng loạt các nghiên cứu sinh sang học tập Göttingen hướng dẫn trực tiếp Klein và Hilbert Tất người đó sau này đã trở thành nhà lãnh đạo khoa học xuất sắc nước Mỹ đại kỉ 20, và cùng lúc, họ đã trở thành người thầy trưởng khoa toán các đại học khác trên khắp giới Vai trò Göttingen với nước Mỹ cách trực tiếp, và với phần còn lại toàn giới cách gián tiếp, là chưa thực đo đếm cách rõ ràng Có mốc thời gian đáng nhớ là, vào thời giờ, Pháp và Đức đã có phát triển đáng ngưỡng mộ toán học, thì nước Mỹ còn hoang vu Khi mà Hilbert chuẩn bị bài nói chuyện bất hủ mình 23 bài toán hướng tới kỉ Paris, thì cùng lúc, tận cuối kỉ 19, với ủng hộ tài chính mạnh mẽ nhà tài phiệt Rockefeller, Đại học Chicago thành lập Trải qua 100 năm phát triển, khoa toán Đại học Chicago ngày đã là trung tâm hùng mạnh trên giới Còn thân Đại học Chicago thì từ ngày đầu, đã trở thành nơi khai sinh tiêu chuẩn lĩnh vực xây dựng và kĩ thuật toàn nước Mỹ Tôi lại mắc cái tật ăn nói lan man, mà trì hoãn để chưa phải tới với cái thiếu sót thứ ba bài viết trước Cái sai sót thứ ba, là sai sót “lớn nhất”, là thực Việt Nam chưa thể quân bình thành tích giải Fields với Đức, vì Đức thì có giải, 212 (213) Tạp chí Epsilon, Số 03, 06/2015 còn Việt Nam thì có nửa, phải chia đôi với Pháp Nhưng nói tới đây, các bạn đọc đừng buồn vì chuyện Việt Nam là ít thành tích Đức, vì lẽ chúng ta phải buồn, thì thân nước Đức phải buồn gấp vài lần, vì chính họ phải “cam chịu” xếp sau nước Châu Á khác câu chuyện thành tích giải Fields này Quốc gia Châu Á mà tôi muốn nói tới, quốc gia mà họ đã âm thầm, lặng lẽ giật tới tận giải Fields, đó là nước Nhật Điều thú vị là, ba giải Fields nước Nhật thuộc lĩnh vực hình học đại số, đó là Kodaira năm 1954 với các công trình dẫn tới hiểu biết tảng các đa tạp Kähler và các cấu trúc Hodge, Hironaka năm 1970 với định lý giải kì dị tiếng, và Mori năm 1990 với chương trình Mori việc phân loại các đa tạp đại số số chiều Tất nhiên, có thể vài chục năm tới, thành tích Việt Nam mảng giật giải Fields hoàn toàn có thể cân với Đức, tâm hơn, thì là Nhật Nhưng có câu chuyện khác “bệnh thành tích” không kém Đức và Nhật mà chúng ta nên để ý tới, đó là câu chuyện sách và dịch thuật Kodaira, Hironaka, Mori: ba nhà toán học Nhật Bản giành huy chương Fields Trên giới có đại dịch thuật mà không có thể quên Cuộc đại dịch thuật thứ là Châu Âu vào kỉ 11 và kỉ 12 phần lớn các học giả Ả-rập thực các đại học đầu tiên Châu Âu hình thành, chính đại dịch thuật này trở thành tiền đề để Châu Âu có phát triển không tưởng khoa học và giáo dục trên quy mô toàn châu lục Và đại dịch thuật thứ hai chính là nước 213 (214) Tạp chí Epsilon, Số 03, 06/2015 Nhật vào hai kỉ 16 và 17 với hàng ngàn sách đã chuyển ngữ chính giới học giả nước(1) , chính đại dịch thuật này là tiền đề, cột trụ cho phát triển quốc gia nước Nhật sau này Ngày nay, người ta nói vui rằng, Đại học Tokyo, các sinh viên có thể không cần dùng tới sách chuyên khảo tiếng nước ngoài, vì thứ gì đã đưa vào sách nước ngoài, thì là nó đã cũ, hai là sách đã dịch tiếng Nhật tức thì Sinh viên Todai có thể làm việc và nghiên cứu trực tiếp lĩnh vực thời thượng trên tiếng mẹ đẻ Theo các thông tin mà GS Nguyễn Xuân Xanh đã dẫn bài viết “Tại người Nhật mê đọc sách?”(1) thì từ dân số còn dừng mức 30 triệu, Nhật đã có sách phát hành tới triệu bản, và số sách in ấn chục vạn là phổ biến Còn theo thông tin trích lục blog “Bản báo cáo Émile Vayrac”(2) mà Blog Quachhien đã đăng tải, thì đầu kỉ 20 Việt Nam đã xuất ấn phẩm phát hành tới chục vạn bản, và tiêu biểu là “Thơ ngụ ngôn La Fontaine” học giả Nguyễn Văn Vĩnh dịch đã bán 100.000 lần xuất đầu tiên, và “Tứ dân văn uyển” tới nửa triệu phát hành đó có 360.000 số là phát không Nhưng đáng tiếc, đó là trường hợp hãn hữu, và tới bây thì chuyện đã khác hoàn toàn Nếu Nhật, việc đọc sách đã trở thành yếu tố truyền thống cấu thành nên văn hóa xã hội, thì Việt Nam, chuyện đọc sách và văn hóa đọc lại trở thành đề tài nóng bỏng Hiện nay, Nhật Bản và Việt Nam có cùng mức dân số xấp xỉ 100 triệu dân, diện tích ngang ngửa (khoảng 300 nghìn km2 ), số lượng sách phát hành hai quốc gia thực là trời vực Nếu Việt Nam sách phát hành tới vạn đã là best-seller, thì Nhật là tương đối tủn mủn Nếu Nhật có các tựa sách in tái lại tới triệu bản, các nhà văn và người sống nghề viết Nhật vô cùng đông đảo và có chế độ đãi ngộ đặc thù và nhân văn, thì Việt Nam, có lẽ có vài tác giả có tới vài chục sách phát hành, và nghề viết, kể người viết chuyên nghiệp, thường là “nghề tay trái”, mà “cơm áo gạo tiền không đùa với khách thơ” 214 (215) Tạp chí Epsilon, Số 03, 06/2015 Cùng mức dân số và diện tích với Nhật Bản và Việt Nam, và nước Nhật, nước Đức ngày thực là đế quốc sách, so với Anh, Pháp, Mỹ, thì Đức là khổng lồ không cần bàn cãi Người ta hay nói vui rằng, bên Đức, đường, bạn gặp người đàn ông cầm chai bia và lang thang, thì 90% là tay kĩ sư, tối thiểu là nhà toán học, còn tới thăm nhà người bạn, và bạn nhận là có phòng riêng bí mật, thì 90% là viết sách nào đó, và có lẽ thực là nhà văn chưa biết chừng Trên thực tế, nhiều gia đình người Đức thực có công ty sách phòng làm việc cá nhân Một số các công ty sách gia đình là Springer, hiệu sách nhỏ, họ đã phát triển ngóc ngách giới Ngày nhắc tới Springer nghĩa là nhắc tới sách khoa học kĩ thuật, sách chuyên khảo, và sách chất lượng cao nhất, và ngược lại, nhắc tới sách khoa học kĩ thuật chất lượng cao, thì nghĩa là nhắc tới Springer Nhớ lại cách đây chừng 1-2 năm, cộng đồng toán học Việt Nam đã tỏ vô cùng vui mừng hai tạp chí nghiên cứu toán học nước đã Springer phát hành toàn cầu, đó là “Acta Mathematica Vietnamica” và “Vietnam Journal of Mathematics”, riêng việc này thôi đã đủ để thấy uy tín Springer giới khoa học mà nói thật là đáng nể Nói nhiều tới Đức và Nhật thế, là âu để chúng ta 215 (216) Tạp chí Epsilon, Số 03, 06/2015 có dịp tự nhìn lại mình Rõ ràng so với hai cường quốc sách thế, thì chúng ta là Epsilon, Epsilon, chắn khác Khác 0, vì lúc này đây, chúng ta có “tiểu dịch thuật”, mà cá nhân tôi, thì đây thực công âm thầm và lặng lẽ thực bởi, cùng lúc, nhiều cá nhân và các nhà sách tư nhân, người dũng cảm và tử tế Có sách đã bắt đầu cách đây vài năm, có vừa mua xong tác quyền Một số sách toán chuẩn bị có dịch tiếng Việt Cuốn “KINH THÁNH Toán" hay nhất, bán chạy Springer suốt mười năm qua là Proof from THE BOOK hai tác giả Martin Aigner và Günter M Ziegler viết dựa trên hồi ức Paul Erdös đã công ty sách Long Minh mua tác 216 (217) Tạp chí Epsilon, Số 03, 06/2015 quyền và tổ chức dịch theo gợi ý và kết nối GS Lê Tuấn Hoa Đây có lẽ là sách toán đầu tiên Springer dịch tiếng Việt thời kì Với gần 50 chủ đề chia nhóm kiến thức số học, đại số, hình học, giải tích, tổ hợp, và lý thuyết đồ thị, THE BOOK chứa đựng cách dày đặc các định lý, tính chất, và các chứng minh toán học đẹp đến ngỡ ngàng Từ định lý đại số, tới tính vô hạn số nguyên tố, các bài toán đếm cây, ứng dụng công thức Euler, bài toán xáo bài, các bài toán đồ thị, phương pháp thống kê các bài toán đếm, bài toán Kedlaya, số π và cây kim Buffon, tới hàng loạt các “hiện tượng” toán học đẹp đẽ, thú vị với ý tưởng chứng minh bất ngờ “không thể tưởng tượng được” đã các tác giả liên tục cập nhật qua lần tái Trong nhóm dịch thuật, có học trò trực tiếp tác giả thứ hai là ông Ziegler Có chuyện khá thú vị là, quá trình dịch thuật, sách đã chuẩn bị đời, đó, nhóm dịch thuật công ty Long Minh, lúc đó làm việc trên 4, đã chuyển sang làm việc trên thảo Có lẽ, sau “Mèo Penrose” và tủ sách Theoni Pappas, cùng hàng loạt sách DK, dịch sách coóng này Springer chắn là tiếp tục tuyệt vời cho Tủ sách “Đam mê toán học” mà Long Minh tâm theo đuổi Với cách tiếp cận khác, sách Kiselev’s Geometry gồm hai tập hình học phẳng và hình học không gian A P Kiselev chủ biên đã GS Nguyễn Tiến Zũng “chỉ đạo” dịch tiếng Việt Sputnik Education Đã lâu người ta có thể lại thấy sách giáo khoa bài bản, kiến thức chính xác, hệ thống bài tập lớp lang, và chứa đựng đầy tính mô phạm các tác giả người Nga xuất trên kệ sách nước, kể từ sau thời sách kinh điển Prasolov và Sharygin Sách hình học Kiselev, cùng với “Tổ hợp quy nạp” Vilenkin dịch GS Hà Huy Khoái, và “Nhập môn đại xác suất & thống kê” soạn GS Nguyễn Tiến Zũng và GS Đỗ Đức Thái, tất khiến cho Sputnik Education bước đặt dấu ấn rõ rệt và vững mình với việc phát triển hệ thống sách giáo khoa toán học nước Các sách Sputnik Education thiên chất lượng, nội dung rành rọt, chữ nghĩa sắc sảo, nhiên là chưa thực đẹp Dẫu hay chưa có đẹp, e đón nhận 217 (218) Tạp chí Epsilon, Số 03, 06/2015 các độc giả phần nào đó có hạn chế Cùng lúc với tiến lên tủ sách “Đam mê toán học” Long Minh, tủ sách khoa giáo Sputnik Education, thì tủ sách “Khoa học khám phá” nhóm dịch giả Phạm Văn Thiều tiếp tục thể suất làm việc đáng ngưỡng mộ mình Lần này sách Love and Math người cộng gần gũi với GS Ngô Bảo Châu là GS Edward Frenkel dịch và phát hành công ty sách Nhã Nam Cuốn sách này là tiếp nối tuyệt vời sách hay trước đó Perelman mà chính nhóm dịch giả này đã dịch là “Perfect Rigor – Thiên tài kì dị và đột phá toán học kỷ” Hai sách hai vế gạch nối, gạch nối biến chuyển kì lạ lòng hệ thống giáo dục nước Nga Xô Viết Thông qua hành trình toán học Perelman và Frenkel, bạn độc giả, là người làm giáo dục và giáo dục toán học nước, chắn rút nhiều bài học quý giá Người ta nói rằng, trình độ dịch giả, đầu tiên thể qua việc chọn sách Sách cốt tinh, không cần nhiều Nhóm dịch giả TS Phạm Văn Thiều lặng lẽ xây dựng thực đã phần nào làm điều đó Perelman, Frenkel, hành trình toán học kì lạ và câu chuyện giáo dục toán học nước Nga Xô Viết Hướng tới thể nghiệm với xã hội, bắt đầu dòng sách “Toán học thân thiện”, nhà sách (giấu tên) đã tiến hành giao dịch tác quyền và đã bắt đầu chuyển ngữ cùng lúc gần chục đầu sách toán các loại, chủ yếu hướng tới đối 218 (219) Tạp chí Epsilon, Số 03, 06/2015 tượng học sinh cấp và cấp hai, đó, tiêu biểu có lẽ là The cartoon guide to statistics Larry Gonick Cuốn “Truyện tranh để học xác suất thống kê” này là cú debut cho tủ sách có tên dự kiến là “Em không sợ toán.” Đây là mạo hiểm, vì xác suất thống kê là mảng bị bỏ trống gần hoàn toàn cấp độ trung học, điều mà các nước phương Tây, đã trở thành điều gì đó hiển nhiên hệ thống tảng kiến thức Một sách khác đã gần hoàn thành, là sách “Cơ sở” Euclid (Euclid’s Element) GS Đàm Thanh Sơn, Blogger 5xu Nguyễn Phương Văn và nhóm cộng tổ chức biên dịch Theo dự kiến, NXB Kim Đồng là đơn vị phát hành dịch này Tuy đây là sách kinh điển, và nhóm dịch giả chất lượng Nhưng thực có nhiều câu hỏi đặt khả thành công dự án này Thứ là việc sách có quá nhiều người tham gia dịch thuật liệu tới đâu, tính đồng và cân đối các phần khác nào Thứ hai là thực thì liệu kiến thức sách này bây có giá trị các hệ bạn đọc tiếp theo, sử dụng số tư kiến thức mức độ nào đó, hay đơn là hồi tưởng dĩ vãng Và thứ ba, là thường sách này, có giá trị nhiều chính bình chú người biên dịch, đây lại điều mà có lẽ không nhóm biên dịch thực Và xét trên ba câu hỏi vậy, thì chuyện phát hành dịch này, có thể không hẳn đạt lại kì vọng lúc ban đầu Tất nhiên, tới tận thời điểm này thì NXB Kim Đồng chưa có động thái cụ thể gì hơn, mặc dù dịch có lẽ đã gần hoàn thiện Như vậy, còn quá sớm để nói tương lai dịch Nhưng dù nào, thì nhiệt tâm tất các đơn vị tham gia công tiểu dịch thuật các sách toán học này là đáng trân trọng! Còn bây giờ, thời gian chờ đợi, lại mời độc giả Epsilon đọc lại dòng giới thiệu đây GS Ngô Bảo Châu “Cơ sở” Euclid cách đây nhiều ngày 219 (220) Tạp chí Epsilon, Số 03, 06/2015 Tác phẩm Cơ sở Euclid là phận cái gọi là phép mầu Hy Lạp Ở xứ này, lần đầu tiên người có ý thức khả chứng minh dựa trên có sở logic hình thức, và đó không thể bắt bẻ Nó là bước tiến xa so với các văn minh cùng thời nơi toán học có thể phát triển phát triển công cụ đo đạc trực tiếp phục vụ sống hàng ngày Thực ra, định lý Pythagore đã chứng minh Ấn độ từ 600 năm, Trung Quốc từ 500 năm trước công nguyên, tức là 200 năm trước sách Cơ sở đời Những khám phá thú vị này không làm lung lay lý thuyết phép mầu nhiệm Hy Lạp Trong Á Đông đặt trọng tâm vào Nghĩa và coi thường Ngữ (pháp), Hy Lạp người ta đã có ý thức sức mạnh ngôn ngữ GS Ngô Bảo Châu(3) Viết chứng minh là thay đổi chất: cái trực quan riêng bạn đã trở thành khách thể tồn độc lập với bạn Về mặt nguyên tắc, người khác có thể đọc lại và hiểu, không cần phải mò mẫm lại đường bạn qua, không cần phải trải nghiệm lại khó nhọc mà bạn đã gặp Con đường gắn bó với bao buồn vui ta, người khác là phương tiện để từ điểm A đến điểm B Nhưng là cái giá phải trả để ta có quyền hy vọng người khác xa Deligne viết này chứng minh lý tưởng mà không khi, không đạt tới: ”nothing should remain visible of the efforts it cost to reach an understanding.” GS Ngô Bảo Châu(3) Giới thiệu với độc giả Epsilon số số thảo chuẩn bị đem in nhiều các nhà sách, đặt bối cảnh lịch sử chuyện sách Đức và Nhật, là để hi vọng các bạn có thông tin và cách nhìn khoáng đạt chuyện sách toán Việt Nam nói riêng và chuyện sách nói chung Ngưỡng mộ nước bạn đã đành, phải nhìn lại rằng, thực ra, sách toán Việt Nam không phải là ít, không phải là không hỗn loạn Có nhiều sách chất lượng, bên cạnh đó có không ít sách thị trường, chuyện âu không thể tránh khỏi Cho 220 (221) Tạp chí Epsilon, Số 03, 06/2015 nên, người làm sách trước đem in sách, mong sách tới tay độc giả, và tới thật sớm Nhưng là khâu, vì các nhà sách mong mỏi phần, thì các tác giả đã viết sách, mong mỏi sách tới tay bạn đọc và bạn đọc cùng chia sẻ điều sách bao lần nhiều Tôi hay nói vui rằng, có sách in tới triệu bản, tới tay triệu người, không đọc, nên đem phép nhân số lượng với số tỉ lệ đọc sách lại thì có lẽ chẳng còn bao nhiêu Cho nên, là điều mà thân người làm sách nên lưu tâm, để tránh lại bị rơi vào bệnh thành tích mà thực chất Rằng không in sách và bán hết đã là thành công, mà cái chủ yếu, là độc giả đã đọc và đạt gì, là cái điều cần hướng tới chung <Bài viết này đã cho phép các đơn vị sở hữu tác quyền các thảo chuẩn bị đem in> Trí Ngủ 10/06/2015 Hà Nội Trích dẫn: Tại người Nhật mê đọc sách? – Nguyễn Xuân Xanh Bản báo cáo Émile Vayrac – Blog Quachhien Yêu ngôn – Ngô Bảo Châu 221 (222) Tạp chí Epsilon, Số 03, 06/2015 222 (223) CÁC VẤN ĐỀ CỔ ĐIỂN VÀ HIỆN ĐẠI TRẦN NAM DŨNG (Đại học Khoa học Tự nhiên, ĐHQG TP HCM) Giới thiệu Chuyên mục này dành cho các vấn đề cổ điển và đại trình bày dạng các bài toán xâu chuỗi Đó có thể là chuỗi các bài để giải bài toán đẳng chu, chứng minh đẳng thức Euler kỳ diệu + 212 + 312 + · · · = π6 , chuỗi bài toán vận trù Cách trình bày xuất phát từ vấn đề đơn giản, dễ hiểu, khái niệm định nghĩa luôn bài để có thể đọc tương đối độc lập Và chuỗi bài nêu vấn đề định, có thể là giải bài toán kinh điển hay nêu giả thuyết mới, vấn đề Ban Biên tập khuyến khích các độc giả gửi lời giải (toàn phần phần) cho Ban Biên tập Các lời giải hay chọn đăng các số (N+3) Thư điện tử xin gửi theo địa trannamdung@yahoo.com, tiêu đề bắt đầu bởi: "[Epsilon] [Lời giải]" Trong số này chúng tôi chọn đăng chuỗi bài toán Khối vuông Rubik Alexey Kanel-Belov, Ilya Ivanov-Pogodaev, Alexey Malistov, Dmitriy Baranov, Ivan Mitrofanov đề xuất cho Hội nghị mùa hè Cuộc thi toán các thành phố, năm 2008 Đặc biệt, dành cho các học sinh lớp lớn tiểu học, các học sinh THCS, THPT và các thầy cô giáo phổ thông chúng tôi giới thiệu chuỗi bài toán Phát triển tư tổ hợp qua các bài toán đếm Phát triển tư tổ hợp qua các bài toán đếm Đếm số đoạn thẳng, số tam giác, số hình vuông không phải vui mà thực hình thành các tư tổ hợp quan 223 (224) Tạp chí Epsilon, Số 03, 06/2015 trọng: biết chia trường hợp, biết phân loại, biết dự đoán quy luật, biết tổng quát hóa, biết loại trừ Dưới đây là chuỗi các bài toán đếm dành cho các học sinh lớp lớn tiểu học và các lớp Trung học sở Với tư tổ hợp hình thành mức độ trực giác, trang bị các công cụ tổ hợp, học sinh dễ dàng hiểu nhanh, hiểu sâu các khái niệm và có khả áp dụng Bài toán Có điểm nằm trên đường thẳng Hỏi có bao nhiêu đoạn thẳng tạo thành? Bài toán Có điểm nằm trên đường thẳng và điểm nằm ngoài đường thẳng đó Hỏi có bao nhiêu tam giác tạo thành? Bài toán Có điểm trên đường thẳng và điểm trên đường thẳng khác Hỏi có bao nhiêu tam giác có đỉnh các điểm đó? 224 (225) Tạp chí Epsilon, Số 03, 06/2015 Bài toán Trên cạnh tam giác có tương ứng 3, 4, điểm Hỏi có bao nhiêu tam giác có đỉnh các điểm đó Bài toán Có điểm tạo thành lưới vuông x Hỏi có bao nhiêu tam giác có đỉnh các điểm đã cho? Bài toán Có bao nhiêu tam giác hình sau? 225 (226) Tạp chí Epsilon, Số 03, 06/2015 Bài toán Trong hình trên có bao nhiêu hình vuông? Bài toán Trong các hình sau có tương ứng bao nhiêu hình chữ nhật? Bài toán Thử dự đoán xem hình bài có bao nhiêu hình chữ nhật? Kiểm tra dự đoán bạn cách đếm trực tiếp 226 (227) Tạp chí Epsilon, Số 03, 06/2015 Bài toán 10 Có bao nhiêu hình bình hành hình sau Bài toán 11 Vẽ điểm trên mặt phẳng cho có đúng 74 tam giác có đỉnh các điểm này Khối vuông Rubik và bài toán Higman Đối tượng nghiên cứu chúng ta là khối vuông Rubik và các trò chơi gần với nó Trước bắt đầu nghiên cứu khối vuông Rubik, ta hãy làm số bài toán chuẩn bị Các bài toán chuẩn bị: Bài toán Có 12 khối lập phương xếp thành dãy, đánh số theo thứ tự từ đến 12 từ trái sang phải Khi trống thần gõ tiếng, hai hình lập phương nào đó đổi chỗ cho Sau 333 tiếng trống, khối lập phương nhảy lên vì hạnh phúc thấy bên phải nó có số lẻ các hình lập phương với số nhỏ số nó Hỏi có thể có đúng hình lập phương nhảy lên vì hạnh phúc? Bài toán Có 42 khối lập phương xếp thành hàng, đánh số theo thứ tự tăng dần Một nước cho phép đổi chỗ hai khối lập phương Có thể xảy tình sau đúng 2008 nước thì hai khối lập phương đầu đổi chỗ cho nhau, còn tất các khối lập phương khác chỗ cũ? 227 (228) Tạp chí Epsilon, Số 03, 06/2015 Bài toán Có 20 khối lập phương có màu khác xếp thành vòng tròn Với nước có thể nhấc khối lập phương lên và xếp khối lập phương thứ vào chỗ khối thứ hai, khối thứ hai vào chỗ khối thứ ba và khối thứ ba vào chỗ khối thứ Có thể xảy khả sau số bước thì tất các khối lập phương dịch sang bước so với cũ theo chiều kim đồng hồ? Bài toán Các mặt khối lập phương sơn các màu khác Từ số các khối lập phương ta xếp thành hình chữ nhật m × n Ta có thể chọn dãy các khối nhỏ (dọc hay ngang) và xoay đồng thời các khối nhỏ theo chiều ngang (hay chiều dọc) Chứng minh ta có thể xoay tất các khối nhỏ để các mặt trên có màu giống 2.1 Chuỗi A - Khối vuông (lập phương) Rubik Để rõ ràng ta gọi khối lập phương là khối lập phương lớn, còn khối nhỏ là các khối lập phương nhỏ tạo thành khối lập phương lớn Mỗi mặt khối lập phương, tạo thành từ khối nhỏ, có thể quay theo chiều kim đồng hồ và ngược chiều kim đồng hồ Có thể thực vài phép quay liên tiếp và ta gọi là tổ hợp các phép quay, hay đơn giản là tổ hợp Các mặt khối lập phương ký hiệu các chữ in hoa, ví dụ A, B, C Phép quay theo chiều kim đồng hồ mặt nào đó ký hiệu đơn giản đúng chữ cái đó luôn, ví dụ A Phép quay ngược chiều kim đồng hồ ký hiệu là A−1 Tổ hợp các phép quay viết dãy các chữ cái: ví dụ ký pháp ABA−1 C nghĩa là đầu tiên ta quay mặt A theo chiều kim đồng hồ, sau đó quay mặt B theo chiều kim đồng hồ, sau đó mặt A ngược chiều kim đồng hồ và cuối cùng là mặt C theo chiều kim đồng hồ Với các phép quay (hay các tổ hợp) X và Y ta gọi giao hoán tử chúng là tổ hợp XY X −1 Y −1 Các khối nhỏ có loại: loại trung tâm - các mặt, góc các góc khối lập phương và - các cạnh khối lập phương Ta tưởng tượng các khối góc và không gắn với gì cả, tức là ta có thể thỏa mái gỡ và chuyển 228 (229) Tạp chí Epsilon, Số 03, 06/2015 Ở đây các khối góc đổi chỗ cho và các khối đổi chỗ cho nhau, và các mặt các khối nhỏ nhìn thấy từ bên ngoài trước đổi là các mặt ngoài sau đổi (Các khối trung tâm ta không đổi chỗ) Mọi vị trí có thể thu cách đổi chỗ ta gọi là trạng thái Ta nói khối lập phương mà tất các mặt đơn sắc (cùng màu) là trạng thái đúng Ta nói khối nhỏ vị trí đúng màu các mặt nó giống màu trạng thái đúng khối lập phương Nếu từ trạng thái ta có thể tổ hợp các bước thu trạng thái khác thì ta nói các trạng thái này liên thông với Trạng thái giải là trạng thái liên thông với trạng thái đúng A1 Một tổ hợp các phép quay đưa khối lập phương khỏi trạng thái ban đầu Chứng minh ta lặp lại số lần thì ta lại thu trạng thái ban đầu A2 Tồn hay không tổ hợp toàn các phép quay mà áp dụng nó số khác lần, có thể chuyển khối lập phương trạng thái đúng từ trạng thái giải được? A3 Hãy nghĩ tổ hợp các phép quay cho phép đổi chỗ các khối nhỏ 1, 2, và giữ các khối khác chỗ (hình 15.1) Hình 15.1: Hãy nghĩ tổ hợp các phép quay cho phép đổi chỗ các khối nhỏ 1, 2, và giữ các khối khác chỗ A4 Chứng minh tổ hợp A−1 C −1 B −1 A−1 BAC đổi chỗ các 229 (230) Tạp chí Epsilon, Số 03, 06/2015 khối nhỏ và và giữ các khối khác chỗ (hình 15.2, bên trái) Hình 15.2: Minh họa cho bài toán A4 (bên trái) và A5 (bên phải) A5 Hãy tìm tổ hợp cho phép các khối nhỏ và đồng thời xoay chỗ còn các khối khác thì chỗ cũ và cũ (hình 15.2, bên phải)? A6 Chứng minh không tồn tổ hợp cho phép khối nhỏ xoay chỗ còn các khối khác thì chỗ cũ và cũ (hình 15.2, bên phải) A7 Giả sử trạng thái khối lập phương là giải Hãy mô tả cách đặt tất các khối vào chỗ mình Giả sử trạng thái khối lập phương không thiết giải Hãy xét tất các trạng thái các khối và mô tả tất các trạng thái liên thông với trạng thái đó A8 Hãy tìm tổ hợp không tầm thường các phép quay mà áp dụng lần, nó không tạo thay đổi nào A9 Hãy tìm tổ hợp các phép quay, cho phép đổi chỗ vòng tròn các khối nhỏ 1, 2, (hình 15.3, bên trái), và không làm thay đổi vị trí các khối góc và giữ tất các khối giữa khác chỗ cũ và cũ A10 Giả sử trạng thái khối lập phương là giải Hãy mô tả cách xếp các khối góc vào vị trí mình mà không làm 230 (231) Tạp chí Epsilon, Số 03, 06/2015 Hình 15.3: Minh họa cho bài toán A9 (bên trái) và A12 (bên phải) thay đổi vị trí các khối Giả sử trạng thái khối lập phương không thiết giải Hãy xét tất các trạng thái các khối góc và mô tả tất các trạng thái liên thông với trạng thái đó A11 Chứng minh không tồn tổ chức các phép quay cho phép xoay chỗ khối góc và giữ tất các khối khác chỗ đó và đó A12 Hãy tìm tổ hợp các phép quay cho phép đồng thời xoay các khối nhỏ 1, 2, (hình 15.3, bên phải) sang góc 120 độ theo chiều kim đồng hồ và không thay đổi vị trí các khối nhỏ khác A13 Hãy mô tả cách xoay khối Rubik, biết nó trạng thái giải A14 Làm theo các khối góc xác định ta có thể xếp thành khối Rubik không, chúng đúng chỗ chúng, còn các khối thì vị trí đúng? A15 Hãy tìm số tối đa các trạng thái đôi không liên thông khối lập phương A16 Hãy tính số trạng thái giải khối vuông Rubik 231 (232) Tạp chí Epsilon, Số 03, 06/2015 2.2 Chuỗi B Trong các bài toán chuỗi này, ta xem xét số các trò chơi nát óc tương tự Ta nói hai tổ hợp phép quay là khác nhau, như, cùng áp dụng vào trạng thái, ta thu các kết khác B1 Trên bàn cờ ta xếp các số từ đến 64, ô số Cho phép chọn hình vuông × và đổi chỗ các số đó theo chiều kim đồng hồ Chứng minh sử dụng các phép toán này, ta có thể thu cách xếp B2 Xét khối × × Hãy mô tả tất các trạng thái giải Có bao nhiêu trạng thái vậy? Hình 15.4: “Chiếc vòng Hungary" B3 Xét trò chơi “Chiếc vòng Hungary” (xem hình 15.4) Đây là trò chơi nát óc phẳng, tạo thành hay vài vòng tròn dính với nhau, trên đó có số ô tròn đánh số Một số số các ô tròn này có thể thuộc vào vòng tròn Mỗi bước đi, ta di chuyển vòng tròn theo hay vài nấc cùng với tất các ô nó Khoảng cách các ô tròn là Ô tròn thuộc nhiều vòng tròn có thể chạy theo vòng tròn nào chứa nó Để đơn giản, ta xét trò 232 (233) Tạp chí Epsilon, Số 03, 06/2015 chơi với hai vòng tròn, vòng chứa ô tròn Các vòng và có di chuyển cùng vòng nào Chú ý nước tương ứng với hoán vị trên tập hợp các số {1, 2, , 10} Hãy mô tả tất các trạng thái có thể chuyển đến từ trạng thái ban đầu B4 Xét trò chơi “Xích đạo” (xem hình 15.5) Trò chơi nát óc này cấu tạo từ mặt cầu, bao bọc đai, chia thành 12 phần, có dạng phần mặt cầu Hai đai giao góc vuông và có mảnh chung, ta gọi là các nút Có tất nút Cho phép di chuyển các đai cho mảnh này chuyển thành mảnh Tổng số các mảnh có thể di chuyển là × 12 − = 30 Hãy mô tả tất các trạng thái có thể chuyển đến từ trạng thái ban đầu Hình 15.5: Trò chơi “Xích đạo" B5 Xét khối vuông × × Hãy mô tả tất các trạng thái đến 233 (234) Tạp chí Epsilon, Số 03, 06/2015 B6 Xét trò chơi “15” Trong hình vuông × có 15 bìa đánh số từ đến 15 Có ô trống Mỗi nước cho phép chọn bìa kề cạnh với ô trống và chuyển bìa đó vào ô trống Hãy mô tả tất các trạng thái đến B7 Có tối đa bao nhiêu trạng thái đôi không liên thông khối vuông × × 4? B8 Hãy tìm hệ các bất biến cho khối vuông × × B9 Hãy tìm hệ các bất biến cho khối vuông n × n × n 2.3 Chuỗi C C1 Xét tứ diện Ta có thể xoay tứ diện nào cho nó biến thành chính nó số đỉnh có thể đổi chỗ Có bao nhiêu cách chuyển vậy? C2 Câu hỏi tương tự cho hình lập phương Các đường chéo hình lập phương nào các di chuyển này? Giả sử ta có khối đa diện Cũng tương tự phép quay khối vuông Rubik và các trò chơi nát óc khác, ta có thể thực tổ hợp các phép di chuyển Rõ ràng tổ hợp này là phép di chuyển biến khối đa diện thành chính nó Ta gọi tổ hợp hai di chuyển là tích hai phép di chuyển Ta gọi di chuyển mà đa diện nói chung không di chuyển gì và tất các đỉnh đứng yên chỗ là đơn vị Nếu có di chuyển nào đó “nhân” với di chuyển đơn vị thì di chuyển đó không thay đổi C3 Giả sử có n phần tử Xét tất các phép biến đổi xếp các phần tử này theo thứ tự khác Ví dụ với n = thì có xếp vậy: Đổi chỗ và 2, để yên 3; Đổi chỗ và 3, để yên 1; Đổi chỗ và 3, để yên 3; Đổi sang chỗ 2, sang chỗ 3, sang chỗ (xích độ dài 3); 234 (235) Tạp chí Epsilon, Số 03, 06/2015 Đổi sang chỗ 3, sang chỗ 2, sang chỗ (xíc độ dài 3); Để tất các phần tử chỗ Một cách tương tự có thể viết tất các xếp (ta gọi là hoán vị) với n Tích hai hoán vị là hoán vị thu cách áp dụng hoán vị thứ nhất, sau đó hoán vị thứ hai Hoán vị nào là đơn vị? Hãy kiểm tra với di chuyển A, tồn di chuyển A−1 mà AA−1 = A−1 A = E, đó E – di chuyển đơn vị Hãy kiểm tra quy tắc (AB)C = A(BC) thỏa mãn C4 Xét tập hợp A – tập hợp tất các di chuyển biến khối lập phương vào chính nó và B là tập hợp tất các hoán vị phần tử Hãy tương ứng hai tập hợp để tích tập hợp này tương ứng với tích tập hợp Định nghĩa Tập hợp G mà các điều kiện sau đây thỏa mãn: Với hai phần tử xác định tích chúng thỏa mãn luật kết hợp (AB)C = A(BC)(A, B, C ∈ G); Tồn phần tử đơn vị E ∈ G : AE = EA = A với A ∈ G; Với phần tử A ∈ G tồn phần tử nghịch đảo A−1 ∈ G : AA−1 = A−1 A = E; gọi là nhóm Nhóm các hoán vị từ bài toán C3 ký hiệu là Sn C5 Chứng minh các tập hợp sau cùng các phép toán là nhóm: Tập hợp các số nguyên theo phép cộng; Tập hợp các số hữu tỷ dương theo phép nhân; Tập hợp các dãy các phép quay khối vuông Rubik theo phép nối (tổ hợp) C6 Các tập hợp sau có phải là nhóm không: Tập hợp các số hữu tỷ theo phép nhân; 235 (236) Tạp chí Epsilon, Số 03, 06/2015 Tập hợp các từ bảng chữ cái hữu hạn (có từ rỗng) phép nối hai từ; Tập hợp các từ từ các chữ cái {a, b, c} (kể từ rỗng), với điều kiện có thể thay đổi các từ XabcY , XbcaY , XcabY thành XY với từ X và Y (tức là xóa abc, bca, cab từ từ), thực phép toán ngược (viết thêm các từ tương ứng); Tập hợp các chuyển vị phần tử (12)(34), (13)(24), (14)(23) và phép biến đổi đồng nhất? Ghi chú: ký hiệu (123)(4567) có nghĩa là dấu ngoặc, các phần tử thay đổi vòng tròn: → → → / → → → → Định nghĩa Giả sử G là nhóm, H là tập G Nếu H chứa phần tử đơn vị nhóm G và H chứa tất các tích các phần tử H và các nghịch đảo chúng thì H gọi là nhóm C7 Giả sử H là nhóm G Chứng minh H là nhóm C8 Tìm tất các nhóm S3 C9 Định lý Lagrange Chứng minh số phần tử nhóm chia hết cho số phần tử nhóm nó C10 Tìm nhóm gồm n!/2 phần tử Sn , đó n > Ta gọi nhóm bài toán trước là An C11 Chứng minh phần tử An là tích các xích độ dài Định nghĩa Phần tử aba−1 b−1 gọi là giao hoán tử các phần tử a và b Định nghĩa Giao hoán tử nhóm G là nhóm gồm tất các tích các giao hoán tử C12 Tìm giao hoán tử các nhóm S3 , A3 , A4 , Sn , An 236 (237) Tạp chí Epsilon, Số 03, 06/2015 Ta chọn nhóm G nào đó phần tử a Mỗi phần tử nhóm ta cho tương ứng với phần tử a−1 ga Phần tử này gọi là liên hợp với g qua a, hay đơn giản là liên hợp C13 Giả sử H là nhóm G Chứng minh a−1 Ha - tập hợp các phần tử liên hợp với H là nhóm Các nhóm H và a−1 Ha gọi là liên hợp Định nghĩa Nhóm mà qua phép liên hợp biến thành chính nó gọi là chuẩn tắc C14 Chứng minh giao hoán tử nhóm, phần tử đơn vị và nhóm là các nhóm chuẩn tắc Trong nhóm có hai nhóm chuẩn tắc tầm thường là phần tử đơn vị và nhóm Các nhóm chuẩn tắc khác, có, gọi là không tầm thường Các nhóm không có nhóm chuẩn tắc không tầm thường gọi là nhóm đơn C15 Với giá trị nào n thì Sn là nhóm đơn C16 Chứng minh An là nhóm đơn với n > C17 Nhóm nào các nhóm Sn An tương ứng với các nhóm dịch chuyển khối lập phương và khối tứ diện C18 Chứng minh nhóm các phép dịch chuyển biến đổi icosahedron tương ứng với A5 C19 Hãy tìm nhóm có phần tử mà đó tồn hai phần tử a và b cho ab 6= ba (còn tiếp phần 2) 237 (238)

Ngày đăng: 14/09/2021, 20:42

Tài liệu cùng người dùng

Tài liệu liên quan